Hand Nerves 01-22, 24 Flashcards

1
Q

A 15-year-old boy presents with a history of a traumatic laceration to the dominant right wrist after punching a window 4 weeks ago. He has numbness and weakness in the median nerve distribution. On operative exploration of the wound, there is a complete transection of the median nerve and a 4-cm gap between the proximal and distal ends. Which of the following is the best method of nerve reconstruction?

A) Polyglycolic acid nerve tube placement
B) Saphenous vein grafting
C) Sural nerve grafting
D) Transposition of the median nerve and primary nerve coaptation

A

The correct response is Option C.

The best method of nerve reconstruction is sural nerve grafting. An autologous sural nerve graft is the best choice for a nerve gap of 3 cm or greater. Multiple cables would likely be required to reconstruct the entire diameter of the median nerve and restore sensory and motor function.

Transposition of the median nerve and primary nerve coaptation is incorrect since the nerve gap is too large for a tension-free nerve repair, even with transposition of the median nerve at the wrist.

Saphenous vein grafting is incorrect since the size of the nerve defect warrants nerve grafting rather than vein grafting. Vein grafting could be used for an associated vascular injury of this size to the radial or ulnar artery.

Polyglycolic acid (PGA) nerve tube placement would be more appropriate for a nerve gap less than 3 cm. This serves as a scaffold for nerve regeneration. The indications for nerve allograft reconstruction are expanding the subject of ongoing research.

How well did you know this?
1
Not at all
2
3
4
5
Perfectly
2
Q

A 29-year-old man who is a laborer presents with significant pain of the nondominant left hand. Medical history includes a laceration of the ring and small fingers palmarly over the proximal phalanx 1 year ago. At the time of the injury, he underwent repair of the flexor digitorum profundus to the small finger and direct repair of the ulnar digital nerve to the ring finger. All other nerves were intact. On examination, the patient reports pain over the nerve repair site that interferes with work-related tasks. There is no sensation distal to the repair. Positive Tinel sign over the ulnar extent of the scar on the ring finger is noted. Which of the following surgical techniques is most appropriate in this patient?

A) Excision of neuroma alone
B) Excision of neuroma and nerve implantation into bone
C) Excision of neuroma and nerve repair with allograft
D) External and internal neurolysis of the neuroma in continuity
E) Targeted muscle reinnervation or nerve transfer to nearby motor nerve

A

The correct response is Option C.

Injuries to the extremities are extremely common, and with the close proximity of the digital nerve to the palmar surface of the digits, these nerves are commonly lacerated. Initial treatment includes primary repair or grafting, depending on the extent of injury and the ability to perform direct repair with minimal tension. Despite repair, patients may develop neuroma in continuity and fail to recover meaningful sensation distal to the level of injury.

In this scenario, the patient has developed a neuroma in continuity at the site of digital nerve repair. Many treatment options exist for the treatment of neuroma in the extremities where the distal nerve target is present. These include: direct repair, the use of a nerve conduit, allograft, or autograft. Where the distal nerve target is absent, as with amputation, techniques to limit or treat neuroma include: excision alone, excision with amputation into muscle or bone, nerve graft to nowhere, nerve capping, centro-central coaptation, end to side neurorrhaphy, targeted muscle reinnervation (nerve transfer), or regenerative peripheral nerve interface (RPNI).

In the study by Moran et al., where authors retrospectively reviewed 127 patients following surgery for symptomatic neuroma, they found that patients who underwent neuroma excision followed by repair had significantly lower DASH scores, compared with implantation into muscle or bone or with simple excision alone (p = 0.03).

Lans et al. demonstrated that patients who underwent neuroma excision followed by repair/reconstruction had lower numeric rating pain scores, lower PROMIS pain interference scores, and higher PROMIS upper extremity scores.

While targeted muscle reinnvervation is a useful technique to prevent or treat neuroma pain, it is more commonly performed in the setting of amputation.

How well did you know this?
1
Not at all
2
3
4
5
Perfectly
3
Q

Which of the following structures is a potential site for compression of the ulnar nerve?

A) Anconeus epitrochlearis
B) Lacertus fibrosis
C) Ligament of Struthers
D) Pronator teres
E) Supinator

A

The correct response is Option A.

Ulnar nerve compression at the elbow is the second most common compression neuropathy of the upper extremity (after carpal tunnel syndrome). There are several sites in this region that can contribute to ulnar nerve compression, including the arcade of Struthers, medial intermuscular septum, Osborne ligament, fascia between the two heads of the flexor carpi ulnaris (FCU), and fascial bands within the FCU distally. The anconeus epitrochlearis is an anomalous accessory muscle that can cause compression of the underlying ulnar nerve at the elbow. The anconeus epitrochlearis, present in 4 to 34% of individuals, originates from the medial epicondyle of the humerus and inserts into the olecranon. If identified during cubital tunnel surgery, this structure is typically released or excised.

The proximal leading edge of the supinator (arcade of Fröhse) can compress the posterior interosseous nerve.

The pronator teres muscle is a potential compression site of the median nerve in the proximal forearm.

The lacertus fibrosis (bicipital aponeurosis) is a potential compression site of the median nerve at the antecubital region.

The superficial radial nerve can be compressed between the tendons of the brachioradialis and extensor carpi radialis longus in the forearm.

The ligament of Struthers is a fibrous band that runs from the tip of the supracondylar process to the medial epicondyle. The median nerve and brachial artery can run through the passage created by the ligament and can be compressed when running through this passage. The arcade of Struthers differs from the ligament of Struthers in that the arcade is a fascial band that runs from the triceps fascia to the medial intermuscular septum, and it is a potential site of compression of the ulnar nerve.

How well did you know this?
1
Not at all
2
3
4
5
Perfectly
4
Q

A 30-year-old man presents with an injury to the ulnar nerve at the elbow. Examination shows an 8-cm nerve gap. Reconstruction with distal nerve transfers at the level of the wrist (motor) and palm (sensory) is planned to restore intrinsic function and finger sensation. If the proximal nerve gap is not repaired, which of the following deficits will most likely persist?

A) Decreased elbow flexion strength
B) Decreased finger extension strength
C) Decreased sensation in the first web space
D) Decreased sensation on the dorsal hand

A

The correct response is Option D.

Distal to the elbow, the ulnar nerve supplies motor innervation to the flexor carpi ulnaris as well as the flexor digitorum profundus to the ring and small fingers, before heading toward the hand, where it supplies most of the intrinsic muscles of the hand (hypothenar muscles, ulnar two lumbricals, interossei, adductor pollicis, deep head of the flexor pollicis brevis). Along the way, it sends off a sensory branch to the dorsal hand; this branch arises about 7 cm proximal to the wrist crease. The most common nerve transfer for motor restoration is transfer of the nerve to pronator quadratus (distal anterior interosseous nerve) to the ulnar motor branch at the wrist. Sensory nerve transfers within the palm can restore sensation to the ulnar digits. However, these distal reconstructions will not restore sensation to the dorsal ulnar hand.

How well did you know this?
1
Not at all
2
3
4
5
Perfectly
5
Q

A 35-year-old, right-hand–dominant man presents with a 4-month history of right upper extremity weakness. The patient reports a history of sudden onset shoulder pain that began without trauma. The pain improved, but he has developed difficulty abducting the shoulder, externally rotating the shoulder, and flexing the elbow. MRI shows hourglass constrictions of the brachial plexus. Electrodiagnostic studies demonstrate the involvement of multiple peripheral nerves. Which of the following is the most likely diagnosis?

A) Brachial plexus avulsion

B) Guillain-Barré syndrome

C) Parsonage-Turner syndrome

D) Rotator cuff injury

E) Transverse myelitis

A

The correct response is Option C.

Acute brachial neuritis, also known as neuralgic amyotrophy or Parsonage-Turner syndrome, is an uncommon but well-recognized clinical entity. Patients characteristically present with acute-onset unilateral severe shoulder and/or arm pain that radiates, followed by progressive weakness and atrophy of the muscles in the shoulder girdle and arm. Pain is the initial symptom for 90% of patients. The etiology of the syndrome is thought to be idiopathic; however, it has been reported in various clinical situations, including post surgery, post infection, during pregnancy, during vigorous exercise, post trauma, and post vaccination. Patients exhibit significant variability in presentation regarding nerve involvement, muscle involvement, the extent of recovery, and recurrence. Diagnosis is made by clinical examination and supported by electrodiagnostic studies and imaging. Electrodiagnostic studies show multiple peripheral nerve lesions, rather than a brachial plexus localization. Parsonage-Turner syndrome is considered self-limited and is often managed conservatively; however, recovery may be protracted and incomplete, and a large percentage of patients never fully recover (30%). Surgical exploration has become more common for patients who do not show clinical or electrodiagnostic signs of recovery after several months. Nerve imaging will demonstrate hourglass constrictions, which are neurolysed at the time of exploration, sometimes in conjunction with nerve transfer.

These patients usually have full shoulder range of motion; additionally, imaging does not correlate with rotator cuff injury.

Avulsion of the brachial plexus would have a prodrome of trauma, and diagnosis would point to root involvement as opposed to more distal nerve pathology.

Guillain-Barré syndrome is usually preceded by infection or other immune stimulation that induces an aberrant autoimmune response targeting peripheral nerves and their spinal roots. These patients traditionally present with symmetric acute flaccid paralysis that can progress to respiratory failure.

Transverse myelitis describes a heterogeneous group of inflammatory disorders that are characterized by acute or subacute motor, sensory, and autonomic (bladder, bowel, and sexual) spinal cord dysfunction (traditionally bilateral), which does not correlate with this patient’s presentation.

How well did you know this?
1
Not at all
2
3
4
5
Perfectly
6
Q

A Froment sign is created by retained function of which of the following muscles?

A) Adductor pollicis
B) Extensor pollicis brevis
C) Extensor pollicis longus
D) Flexor pollicis brevis
E) Flexor pollicis longus

A

The correct response is Option E.

Ulnar nerve injuries are particularly devastating and have significant sensory and motor functional consequences. Proximal to the elbow, the expected motor deficits include absence of the flexor carpi ulnaris, the flexor digitorum profundus to the middle/ring/small fingers, the hypothenar muscles (opponens digiti minimi, abductor digiti minimi, flexor digiti minimi brevis), the third and fourth lumbrical muscles, the dorsal and palmar interosseous muscles, the adductor pollicis, and the deep head of the flexor pollicis brevis. A Froment sign is observed when the patient attempts to pinch but is unable to activate the adductor pollicis (ulnar innervated) and compensates by activating the flexor pollicis longus by flexing the interphalangeal joint (median nerve innervated). Extensor pollicis longus (EPL), extensor pollicis brevis (EPB), and flexor pollicis brevis (FPB) are not involved in the thumb interphalangeal joint flexion that produces Froment sign.

How well did you know this?
1
Not at all
2
3
4
5
Perfectly
7
Q

Targeted muscle reinnervation allows for increased type of which of the following prosthetic controls for an above-elbow amputee?

A) Cable
B) Electromyographic
C) Passive
D) Switch

A

The correct response is Option B.

Upper limb amputation is a devastating loss, more so than lower limb amputation. The level of amputation portends increased functional loss, i.e., forearm-level amputees maintain more functional capacity than above-elbow amputees. Many patients with upper extremity amputations rely on their prosthetic for daily functional demands. Patients with an above-elbow amputation have less muscles to exert control of a prosthetic, which makes multiple simple movements difficult and compound movements especially challenging.

There are multiple types of prostheses for the upper extremity. There are passive prosthetics, which are frequently cosmetic but may allow for a stable post for the other extremity. Functional prosthetics are body-powered (via cable controls) or externally powered (myoelectric, switch control). Body-powered prostheses use the remnant body motions through a harness (i.e., scapular and humeral motion for a transhumeral amputee) via cable to control motion and the force of a terminal device. These are simple devices that do not require as much maintenance and have more longevity, but they are limited in terms of functionality.

Myoelectric prostheses use the remnant muscle contractions via electromyographic (EMG) capture for prosthetic control. The more independent discrete electromyographic signals there are, the more potential functional options of the prosthetic there are. If there are minimal/limited EMG sensor options, then switch control can be used. Switch control is when small switches are turned on/off to increase control for separate components of a prosthetic. Switch control can be combined with myoelectric control to increase functionality for higher level amputees. For example, a transhumeral-level amputee can use switch control to position the elbow, while EMG signals from the biceps and triceps control wrist/hand motion.

Targeted muscle reinnervation (TMR) uses nerve transfers to “hyperreinnervate” muscles to allow for increased distinct electromyographic signals as well as increased amplification of signals, increasing control sites for prosthetic functionality. A good example is using residual peripheral nerves of transhumeral amputees, such as the median, radial, and ulnar nerves, to increase independent control of each biceps and triceps head as well as brachialis. TMR therefore allows for increased EMG control of a myoelectric prosthetic.

How well did you know this?
1
Not at all
2
3
4
5
Perfectly
8
Q

An upper trunk (C5-C6) brachial plexus injury is most likely to result in dysfunction of which of the following muscles?

A) Biceps
B) Flexor carpi ulnaris
C) Pectoralis minor
D) Trapezius
E) Triceps

A

The correct response is Option C.

Upper trunk brachial plexus injuries are associated with the classic “waiter’s tip” posture: the shoulder is adducted and internally rotated at rest. The elbow is extended, the forearm is pronated, and the wrist and digits are held in flexion. Impacted nerves include the axillary, musculocutaneous, and suprascapular nerves. The musculocutaneous nerve innervates the biceps and brachialis muscle. Loss of function of these muscles results in deficits of elbow flexion and forearm supination.

The flexor carpi ulnaris is innervated by the ulnar nerve, which originates from the medial cord of the brachial plexus and carries innervation from C8-T1.

The pectoralis minor is innervated by the medial pectoral nerve from the medial cord of the plexus and carries nerve fibers from C8 and T1.

The trapezius is innervated by the spinal accessory nerve (cranial nerve XI).

The triceps is innervated by the radial nerve, which originates from the posterior cord of the brachial plexus and has contributions from C5-T1.

How well did you know this?
1
Not at all
2
3
4
5
Perfectly
9
Q

A 55-year-old woman who is an administrative assistant is referred because of a 2-month history of numbness of the middle finger on the dominant hand. Splinting has not improved the symptoms. Electrodiagnostic studies show borderline mild carpal tunnel syndrome. Which of the following is the most appropriate next step in management?

A) Carpal tunnel release
B) Corticosteroid injection
C) Exercises with tendon and nerve gliding
D) Guyon canal release
E) Transcutaneous electrical nerve stimulation (TENS)

A

The correct response is Option B.

The answer is corticosteroid injection. A 3-month history of persistent symptoms is one indication for surgical intervention; the patient in this scenario has a history of 2 months. The patient has not had a trial of nonsurgical management. The electrodiagnostic studies show borderline or mild positive carpal tunnel syndrome. Corticosteroid injection has shown improvement in 32% of affected patients who did not have to go on to surgery. Corticosteroid injection should therefore be considered prior to surgical intervention. Transcutaneous electrical nerve stimulation as well as exercise has not been shown to be beneficial in the management of carpal tunnel syndrome. Besides corticosteroid injections, nonsurgical splinting by an occupational therapist and ultrasound have also been shown to improve symptoms of carpal tunnel syndrome.

How well did you know this?
1
Not at all
2
3
4
5
Perfectly
10
Q

A 29-year-old man is evaluated 15 months after a motorcycle collision in which he sustained a left humerus fracture and a complete left brachial plexus avulsion. He has not recovered any motor or sensory function of the left arm but desires the ability to actively flex the elbow. Which of the following procedures is most likely to restore active elbow flexion in this patient?

A) Free functioning gracilis muscle transfer
B) Pedicled bipolar latissimus dorsi muscle transfer
C) Proximal transfer of the flexor-pronator mass onto the humerus
D) Triceps-to-biceps transfer
E) Ulnar nerve to musculocutaneous nerve transfer

A

The correct response is Option A.

Adult traumatic brachial plexus injuries are devastating and life-altering injuries. Evaluation includes a detailed physical examination and radiologic and electrodiagnostic studies. Knowledge of injury patterns, timing of surgery, prioritization in restoration of function, and management of patient expectations are key components of management. In general, options for treatment of brachial plexus injuries include neurolysis, nerve grafting, or nerve transfers and should be performed within 6 months of injury. Free functioning muscle transfers (FFMT) and tendon transfers are typically used in patients who present late (greater than 12 months from injury), because the time for the nerve to regenerate after reconstruction is greater than the survival time of the motor end plates after denervation.

FFMT is the transplantation of a muscle and its neurovascular pedicle to a new location to assume a new function. The muscle is innervated by transferring an intact uninjured donor motor nerve; circulation is restored to the muscle through microsurgical anastomosis of the artery and vein to donor vessels (typically the thoracoacromial artery and cephalic vein). Within 6 to 9 months, the transferred muscle reinnervates, eventually gaining independent function. Although initially indicated in patients who presented late or as a salvage procedure with failed previous nerve reconstruction, the success with FFMT has led to use in early reconstruction to obtain elbow flexion and rudimentary grasp in patients with pan-plexus injuries. The gracilis muscle is the most commonly used donor because of its proximally based neurovascular pedicle and its long tendon length (which can reach distally into the forearm). Spinal accessory or intercostal nerves could be used as donor nerves for a free gracilis transfer.

Steindler flexorplasty, pedicled latissimus dorsi muscle transfer, triceps-to-biceps transfer, and Oberlin transfer each require remaining function of the brachial plexus and would be contraindicated in this patient with long-standing total brachial plexus palsy.

How well did you know this?
1
Not at all
2
3
4
5
Perfectly
11
Q

A 7-year-old boy falls and sustains the fracture in the x-ray study shown. He undergoes closed reduction and percutaneous pinning of the fracture that night. Postoperatively, he cannot flex his thumb at the interphalangeal joint or his index finger at the distal phalangeal joint. He has normal sensation of his thumb and other fingers. Which of the following nerves was most likely injured in the fall?

A) Anterior interosseous
B) Musculocutaneous
C) Posterior interosseous
D) Radial
E) Ulnar

A

The correct response is Option A.

The median nerve travels ulnarly to the brachial artery at the distal humerus. Because of the close proximity of the median and anterior interosseous nerves, they can be injured in pediatric supracondylar humerus fractures. This is often the result of strain on the brachial artery and median nerve at the time of injury, and nerve injuries occur in 11 to 16% of these fractures. Contusions of the median and anterior interosseous nerves often recover over time with conservative management. Patients can present with pure median nerve injury, pure anterior interosseous nerve injury, or injury to both nerves. In some cases of pulseless supracondylar humerus fractures, exploration is warranted. In this patient, motor deficits are indicative of an anterior interosseous nerve contusion.

Given the maintained sensation of the hand, the median nerve would be incorrect. The ulnar nerve does not provide motor innervation for thumb IP or index DIP flexion.The radial and posterior interosseous nerves do not innervate the flexor pollicis longus or index finger (flexor digitorum profundus) muscles and do not cross over the volar humerus. Also, the musculocutaneous nerve has no motor innervation distal to the elbow and is only sensory to the forearm.

How well did you know this?
1
Not at all
2
3
4
5
Perfectly
12
Q

A patient presents with an unresectable sarcoma of the proximal forearm. An elective transhumeral amputation is planned. Which of the following are the main advantages of performing targeted muscle reinnervation at the time of amputation in this patient?

A) Decreased pain in residual limb and improved control of myoelectric prosthesis
B) Greater length of residual limb and improved soft-tissue coverage over bone
C) Improved sensation in residual limb and greater range of motion in native elbow
D) Improved shape of residual limb and greater range of motion in native shoulder
E) Increased muscle bulk of residual limb and improved control of myoelectric prosthesis

A

The correct response is Option A.

Targeted muscle reinnervation (TMR) has been a major advance in the care of amputees and involves nerve transfers in the residual limb. Benefits of targeted muscle reinnervation include improved control of myoelectric prostheses for transhumeral amputees and an improvement in residual limb pain for amputees. There appears to be greater effect when TMR is performed acutely at the time of amputation. With this and other contemporary techniques for amputees, the nerves are provided a functional destination. TMR does not substantially alter the range of motion in the shoulder or elbow, and does not provide additional length of the limb. It does not appear to increase the muscle bulk in the residual limb.

How well did you know this?
1
Not at all
2
3
4
5
Perfectly
13
Q

Which of the following diagnostic findings is most consistent with a recovering motor nerve injury?

A) Decreased motor unit potential amplitude
B) Decreased motor unit recruitment
C) Fibrillation potentials
D) Nascent potentials
E) Positive sharp waves

A

The correct response is Option D.

Electrical studies of a recovering nerve injury would show nascent potentials. This finding usually precedes the onset of clinically evident voluntary movement in the muscles innervated by the injured nerve. Nascent potentials appear several months after injury and result from axonal regeneration. Decreased motor unit potential amplitude, fibrillation potentials, positive sharp waves, and decreased motor unit recruitment are possible diagnostic findings in the setting of a nerve injury, but they are not indicative of recovery. Nerve lesions that spontaneously recover are usually treated nonsurgically, whereas those without recovery are explored and reconstructed. As a general rule, nerve regrowth occurs at approximately 1 inch per month or 1 millimeter per day. Motor endplates degrade at about 1% per week, hence the maximum length that a nerve can grow to restore motor function is approximately 13 to 18 inches. Repairs at the brachial plexus level rarely result in the recovery of any intrinsic muscle function. Sensory end organs, however, remain viable and can be reinnervated even after many years.

How well did you know this?
1
Not at all
2
3
4
5
Perfectly
14
Q

A 20-year-old woman presents for repair of a significant degloving injury to her dominant right thumb. During the soft-tissue repair, a 4-cm defect is noted in the radial proper digital nerve. Which of the following is the most appropriate technique for nerve repair?

A) Autologous nerve graft
B) Autologous vein graft
C) Collagen conduit
D) Nerve transposition

A

The correct response is Option A.

For nerve gaps 3 cm or longer that are not amenable to repair, peripheral nerve grafts are the most reliable choice. Proper microsurgical nerve repair requires meticulous, atraumatic technique with appropriate magnification, instrumentation, and sutures. Primary repair is recommended when a tension-free anastomosis is possible. In this case, primary repair is not possible, even with nerve mobilization or transposition. Use of these autologous grafts is limited by donor site supply and morbidity, loss of sensation, and possible neuroma formation.

Various nerve conduits such as silicone tubes, biologic tubes such as vein grafts, and biodegradable conduits such as collagen or polyglycolic acid are limited to small diameter nerves with shorter nerve gaps (2 to 3 cm). There appears to be no significant regeneration beyond 3 cm. Nerve transpositions are not performed for digital nerve repair.

How well did you know this?
1
Not at all
2
3
4
5
Perfectly
15
Q

A male newborn is evaluated in the newborn nursery because of limited movement of the right arm. Maternal history includes gestational diabetes, but routine prenatal monitoring and ultrasound examinations were normal. The patient’s delivery was difficult, and he weighed 10.5 lb (4.8 kg) at birth. He was noted to have no spontaneous movement of the right arm. The right upper extremity was warm, pink, and supple. Pulsations of the radial and ulnar arteries were palpable at the wrist. X-ray studies of the affected shoulder show no obvious fractures. A photograph is shown. Which of the following is the most appropriate next step in management?

A) Angiography of the extremity
B) Anticoagulation
C) CT scanning of the extremity
D) Measurement of compartment pressures
E) Observation

A

The correct response is Option E.

This patient has an obstetrical palsy of the right upper extremity. The likelihood of recovery depends on the severity of the injury, but statistically over 70% of patients have complete or near complete recovery of upper extremity function without any surgical intervention. Thus the most appropriate next step for this newborn is observation. MRI of the shoulder and neck are helpful to discern evidence of anatomical injury to the cervical roots and/or portions of the brachial plexus, but CT scanning is unnecessary and of little use in this regard. Angiography and anticoagulation would be appropriate steps if there was clinical evidence of subclavian or brachial artery thrombosis, but the physical examination does not support this diagnosis. Similarly, the examination is inconsistent with neonatal compartment syndrome, a rare condition that usually presents with arm swelling, immobility, reduced arm perfusion, and purple cutaneous areas. Therefore, measurement of compartment pressures is unnecessary.

How well did you know this?
1
Not at all
2
3
4
5
Perfectly
16
Q

A 65-year-old man presents to the office with symptoms and examination findings consistent with bilateral carpal tunnel syndrome. Medical history includes recent rupture of the left biceps tendon and several trigger fingers. Biopsy of the tenosynovium is performed during carpal tunnel release and the pathology report shows a positive Congo red stain. Which of the following diagnoses is most likely and should prompt further consultation?

A) Amyloidosis
B) Diabetes mellitus
C) Fibromyalgia
D) Malingering
E) Rheumatoid arthritis

A

The correct response is Option A.

A recent study showed that 10% of men over 50 years old and women over 60 years old with bilateral carpal tunnel syndrome had positive tenosynovial biopsies for amyloid. This can be a devastating disease if left to affect the heart or other organs and can be diagnosed with a simple biopsy during carpal tunnel release. Other findings suggestive of amyloidosis include a spontaneous biceps rupture, trigger finger, and spinal stenosis.

Patients with diabetes and rheumatoid disease have a higher incidence of carpal tunnel syndrome than the general population but the mention of biceps rupture points to the diagnosis of amyloidosis. Fibromyalgia and malingering have not been shown to be related to carpal tunnel syndrome.

How well did you know this?
1
Not at all
2
3
4
5
Perfectly
17
Q

A 32-year-old man presents to the emergency department for evaluation of a laceration of the right wrist sustained when he punched a glass window 1 hour ago. Physical examination shows a 2-cm transverse laceration of the volar ulnar wrist crease. Wound exploration shows complete laceration of the ulnar nerve. On physical examination of motor function, LOSS of which of the following functions is most likely in this patient?

A) Adduction of the thumb carpometacarpal joint
B) Extension of the metacarpophalangeal joint of the ring and small fingers
C) Extension of the thumb interphalangeal joint
D) Flexion of the interphalangeal joint of the index and middle fingers
E) Flexion of the interphalangeal joint of the ring and small fingers

A

The correct response is Option A.

The ulnar nerve is the terminal branch of the medial cord of the brachial plexus. It enters the forearm between the two heads of the flexor carpi ulnaris (FCU). In the forearm, the ulnar nerve innervates the FCU and flexor digitorum profundus of the small and ring fingers. It courses distally under the FCU to enter Guyon’s canal at the wrist. The dorsal cutaneous nerve, which gives sensation to the dorsoulnar hand, arises approximately 5 to 7 cm proximal to the ulnar styloid. In Guyon’s canal, the ulnar nerve splits into a deep motor and a superficial sensory branch. The deep motor branch innervates the hypothenar muscles (abductor digiti minimi, opponens digiti minimi, and flexor digiti minimi), as well as the lumbricals to the ring/small fingers, dorsal and palmar interossei, flexor pollicis brevis (deep head), palmaris brevis, and adductor pollicis. The superficial sensory branch in the palm innervates the small finger and the ulnar aspect of the ring finger.

Adduction of the thumb is controlled through activation of the ulnar-innervated adductor pollicis muscle. In the small and ring fingers, extension of the metacarpophalangeal (MCP) joint is performed through activation of the radially innervated extensor digitorum communis and extensor digiti minimi muscles. In the index, middle, ring, and small fingers, flexion of the proximal interphalangeal (PIP) joint is performed through activation of the median-innervated flexor digitorum superficialis.

While flexion of the ring and small finger distal interphalangeal (DIP) joints is produced by the ulnar-nerve innervated FDP tendons to the ring and small finger, the ulnar nerve provides branches to this muscle proximal to this patient’s injury. Extension of the thumb MCP joint is via the radial nerve innervated extensor pollicis brevis muscle.

How well did you know this?
1
Not at all
2
3
4
5
Perfectly
18
Q

In a transhumeral amputee, targeted muscle reinnervation can be utilized to improve control in a myoelectric prosthesis. Which of the following nerve transfers can be performed to provide intuitive prosthetic control for hand closure?

A) Median nerve to short head of biceps
B) Musculocutaneous nerve to long head of biceps
C) Radial nerve to lateral head of triceps
D) Radial nerve to long head of triceps
E) Ulnar nerve to lateral head of triceps

A

The correct response is Option A.

Targeted muscle reinnervation (TMR) utilizes a set of nerve transfers in order to allow intuitive prosthetic control for upper extremity amputees. Functioning nerves that no longer have their distal muscle target can be transferred to intact proximal muscles and generate a novel electrical signal that can be picked up by a myoelectric prosthesis. Another benefit of TMR is the potential to prevent or treat painful neuromas.

In the case of a transhumeral amputee, elbow flexion myoelectric prosthetic control is maintained by preserving musculocutaneous innervation to the long head of the biceps muscle. The distal remnant of the median nerve is transferred to the motor nerve of the biceps short head to create a signal for prosthesis hand closure. Elbow extension signals are maintained with radial innervation of the long head of the triceps. Signals for prosthesis hand opening are created with transfer of the distal radial nerve to the motor nerve of the triceps lateral head.

How well did you know this?
1
Not at all
2
3
4
5
Perfectly
19
Q

Resistance to which of the following maneuvers is most likely present in a digit that has intrinsic tightness?

A) Passive extension of the metacarpophalangeal (MCP) joint with the proximal interphalangeal (PIP) joint held in hyperextension
B) Passive extension of the PIP joint with the MCP joint held in hyperflexion
C) Passive flexion of the DIP joint with the PIP joint held in hyperextension
D) Passive flexion of the PIP joint with the MCP joint held in hyperextension
E) Passive flexion of the PIP joint with the MCP joint held in hyperflexion

A

The correct response is Option D.

The intrinsic muscles (dorsal/palmar interossei and lumbricals) are responsible for much of the fine motor function of the hand. Contractures of these muscles lead to a loss of the delicate and complex balance of the intrinsic and extrinsic muscles and typically results in the clinical picture of an intrinsic-plus hand. The intrinsics attach to the extensor mechanism through the lateral bands and facilitate force transmission from the muscles to the extensor mechanism on the proximal and distal phalanges. Because of their line of pull, the intrinsics are responsible for metacarpophalangeal (MCP) joint flexion and proximal interphalangeal (PIP) joint extension. The intrinsic tightness test (i.e. Bunnell test) requires one to assess passive PIP joint flexion with the MCP joint extended. This is compared with passive PIP joint flexion with the MCP joint in flexion which assesses for extrinsic tightness. If there is a substantial increase in resistance to PIP joint flexion with the MCP joint in extension, then the test is considered positive and indicative of intrinsic tightness or adhesions of the lateral bands.

Trauma is the most common cause of intrinsic muscle contracture. Spasticity from an upper motor neuron lesion (e.g. traumatic brain injury, cerebrovascular accident, cerebral palsy) may also lead to intrinsic contracture. Arthritis may also lead to intrinsic contracture resulting from joint deviation or dislocation.

In trauma, initial treatment is directed at edema prevention and aggressive hand therapy. Patients with spasticity from an upper motor neuron disorder are also initially managed with therapy and splinting. If these modalities are insufficient, surgical release of the intrinsic muscles or tendons (proximal or distal depending on extent of involvement) may improve posture and function. Ulnar motor neurectomy is another option in severely affected individuals to decrease intrinsic muscle tone and improve posture and function, but is only effective in the absence of a fixed MCP joint contracture.

How well did you know this?
1
Not at all
2
3
4
5
Perfectly
20
Q

A 37-year-old woman presents for evaluation of a laceration to the mid humerus that she sustained in a motor vehicle collision. On examination, the patient is unable to extend the wrist, fingers, or thumb. Surgical exploration shows complete radial nerve transection; the median/ulnar nerves are intact. Direct neurorrhaphy is performed after debridement and mobilization of the nerve ends. Which of the following is the last muscle to be reinnervated during nerve recovery?

A) Abductor pollicis brevis
B) Abductor pollicis longus
C) Brachioradialis
D) Extensor carpi radialis brevis
E) Extensor indicis proprius

A

The correct response is Option E.

Radial nerve injuries may occur in the setting of humeral fractures, and transection is most common in the setting of an open injury. The most important components of functional recovery following radial nerve injury include wrist, finger, and thumb extension. The order of reinnervation of the radial-innervated muscles is most commonly brachioradialis, extensor carpi radialis longus, supinator, extensor carpi radialis brevis, extensor digitorum communis, extensor carpi ulnaris, extensor digiti quinti, abductor pollicis longus, extensor pollicis longus, extensor pollicis brevis, and extensor indicis proprius. The abductor pollicis brevis is innervated by the median nerve via the thenar motor branch.

How well did you know this?
1
Not at all
2
3
4
5
Perfectly
21
Q

A 37-year-old woman presents with sharp lateral elbow pain sustained when lifting a garbage bag out of a can at work. Which of the following muscles is the most likely cause of this patient’s pain?

A) Anconeus
B) Brachioradialis
C) Extensor carpi radialis brevis
D) Extensor carpi radialis longus
E) Extensor carpi ulnaris

A

The correct response is Option C.

The common extensor tendon attaches the extensor carpi radialis brevis (ECRB), extensor digitorum communis (EDC), extensor digiti minimi, and extensor carpi ulnaris to the lateral epicondyle. Among these forearm extensor muscles, pathology found in the attachment of ECRB and EDC at the lateral epicondyle (LE) is commonly cited as a reason for pain at the LE. While the extensor carpi radialis longus (ECRL) is a wrist extensor and may be a source of lateral elbow pain with strain, it does not originate from the lateral epicondyle. The ECRL muscle was observed to originate from the distal aspect of the supracondylar ridge mainly as a muscular attachment. The brachioradialis is an elbow flexor and originates off of the lateral column of the distal humerus. It is not involved with lateral epicondylitis. The anconeus is a small muscle which originates off of the lateral epicondylitis and its main function is to assist with elbow extension.

How well did you know this?
1
Not at all
2
3
4
5
Perfectly
22
Q

A 36-year-old man presents with weakness of the left wrist and limited finger extension 2 days after he underwent surgery for a closed fracture of the right humerus that he sustained in a motor vehicle collision. At the time of surgery, the radial nerve was found to be intact without any significant signs of trauma. On examination, the patient is unable to extend his wrist or digits but elbow extension is intact. Which of the following histopathologic features seen in neurapraxia (Sunderland Type 1) injury is most likely in this patient?

A) Disruption of perineurium
B) Endoneurial fibrosis
C) Increased axonal transport
D) Segmental demyelination
E) Wallerian degeneration

A

The correct response is Option D.

Peripheral nerve injuries can be classified into three main categories: Neurapraxia, axonotmesis, and neurotmesis. These groups have been further classified by several different classification systems, which include those by Seddon and Sunderland, among others.

Neurapraxia (Sunderland type 1) is an injury to the myelin sheath only, while axons are preserved. In trauma, these injuries are most frequently caused by compression or stretching. Although segmental demyelination occurs (leading to conduction block), there is no Wallerian degeneration of the nerve, and a full recovery can be expected within days to weeks. Axonotmesis (Sunderland type 2, 3 or 4) involves damage to axons, and is characterized by Wallerian degeneration. Sunderland type 2 injury involves only the axons, and usually there will be a full recovery without intervention, while types 3 and 4 involve injury to the endoneurium and perineurium respectively and fail conservative management. Neurotmesis (Sunderland type 5) is a complete disruption of a peripheral nerve. MacKinnon and Dellon described a type 6 injury that involves mixed Sunderland type injuries along the length of a damaged nerve. Axonal transport is not a relevant histopathological feature.

How well did you know this?
1
Not at all
2
3
4
5
Perfectly
23
Q

A 50-year-old woman previously diagnosed with left forearm compression neuropathy of the superficial radial nerve comes to the office for examination. The patient has not responded to 7 months of conservative management which consisted of NSAID therapy, steroid injection, a trial of splinting, and activity modification. Operative treatment is planned. Fascial release between which of the following two tendons is most appropriate in this patient?

A) Abductor pollicis longus and extensor pollicis brevis
B) Brachioradialis and extensor carpi radialis longus
C) Extensor carpi radialis longus and extensor carpi radialis brevis
D) Flexor carpi radialis and abductor pollicis longus
E) Flexor carpi radialis and brachioradialis

A

The correct response is Option B.

Superficial radial nerve compression of the forearm occurs most frequently at the posterior border of the brachioradialis where the nerve transitions from a deeper, subfascial position to a more superficial, subcutaneous location. Also known as Wartenberg syndrome, patients may present with pain, numbness, or tingling over the dorsal radial hand radiating to the dorsal thumb and index finger.

Symptoms of superficial radial nerve compression may be confused with symptoms of de Quervain’s tenosynovitis. In addition, both conditions may coexist simultaneously.

Patients diagnosed with superficial radial nerve compression are initially treated conservatively since this approach is successful in relieving symptoms in the majority of cases. Conservative management consists of rest, splinting, removal of external compression source (such as a tight wristwatch band, bracelet, or handcuffs), and nonsteroidal anti-inflammatory medications.

Surgery is indicated when conservative measures fail. Surgical decompression involves release of the fascia between the brachioradialis and extensor carpi radialis longus tendons. It is at this interval that the nerve transitions from deep to superficial and prone to compression. The other responses do not reflect the correct surgical anatomy of this condition.

How well did you know this?
1
Not at all
2
3
4
5
Perfectly
24
Q

A 45-year-old man who is a cyclist comes to the office because of tingling of the left ring and small fingers. Normal sensibility dorsally is noted. Guyon canal release is planned. Which of the following structures is a border of the Guyon canal?

A) Capitate
B) Dorsal distal radio-ulnar ligament
C) Lunate
D) Pisohamate ligament
E) Volar distal radio-ulnar ligament

A

The correct response is Option D.

Guyon canal, also known as ulnar canal and ulnar tunnel, allows passage of the ulna nerve and artery into the hand. It is a semi-rigid fibrosseous longitudinal tunnel, approximately 4 cm in length, beginning at the proximal edge of the transverse carpal ligament and ending at the hypothenar aponeurotic arch. The roof comprises the volar carpal ligament. The medial wall is the pisiform and pisohamate ligament. The lateral wall is the hook of the hamate. The floor is the flexor retinaculum and hypothenar muscles.

The volar and dorsal distal radio-ulnar ligaments are the thicker portions of the triangular fibrocartilage complex (TFCC) and do not contribute to Guyon canal.

The capitate and lunate are part of the floor of the carpal tunnel and also do not contribute to Guyon canal.

How well did you know this?
1
Not at all
2
3
4
5
Perfectly
25
Q

A 52-year-old man presents for evaluation of a claw deformity of the right ring and small fingers. Medical history includes an unrepaired low ulnar nerve injury sustained 30 years ago. Which of the following is the most likely pathophysiology of this patient’s deformity?

A) Unbalanced abductor digit minimi muscle
B) Unbalanced median and ulnar innervated intrinsic muscles
C) Weak thenar muscles
D) Weak ulnar innervated extrinsic flexor muscles
E) Weak ulnar innervated intrinsic muscles

A

The correct response is Option E.

Clawing after ulnar nerve injury includes hyperextension of the metacarpophalangeal (MCP) joints and flexion of the interphalangeal (IP) joints. The pathophysiology includes paralysis of the interossei and third and fourth lumbricals. Unopposed long extensors cause the metacarpophalangeal joints to fall into extension while the long flexors pull the proximal interphalangeal joints into flexion. This posture is the classical ‘claw hand.’

How well did you know this?
1
Not at all
2
3
4
5
Perfectly
26
Q

Myoelectric prostheses offer which of the following advantages over body-powered prostheses?

A) Higher durability
B) Lower cost
C) Lower frequency of adjustment
D) More complex motions performed
E) Shorter training time

A

The correct response is Option D.

Body-powered prostheses have been shown to have advantages in durability, training time, frequency of adjustment, maintenance, and feedback; however, they could still benefit from improvements of control. Myoelectric prostheses have been shown to provide greater range of motion including more complex movements involving multiple joints moving at the same time. Currently, evidence is insufficient to conclude that either system provides a significant general advantage. Prosthetic selection should be based on a patient’s individual needs and include personal preferences, prosthetic experience, and functional needs.

How well did you know this?
1
Not at all
2
3
4
5
Perfectly
27
Q

A 26-year-old man comes to the office 4 weeks after injuring his left shoulder while snowboarding. Physical examination shows limited abduction and forward flexion of the shoulder to 30 degrees. No additional abnormalities are noted. From which of the following areas of the brachial plexus does the affected nerve most likely arise?

A) Lateral cord
B) Lower trunk
C) Medial cord
D) Posterior cord

A

The correct response is Option D.

The axillary nerve (ventral rami of C5 and C6) arises from the posterior cord of the brachial plexus, giving off muscular branches to teres minor and deltoid. It also innervates the shoulder joint and the skin over the deltoid. Its close proximity to the inferior shoulder capsule as it courses on the anteroinferior border of the subscapularis and then through the quadrangular space, puts it at risk for injury.

The axillary nerve is most commonly injured during orthopedic surgeries such as shoulder arthroscopy, and open reduction and internal fixation (ORIF) of the proximal humerus, in which case, it is most commonly the result of closed traction injury. It can also be seen in the setting of an anterior glenohumeral joint dislocation or proximal humerus fracture, or as the result of a direct blow to the superior aspect of the shoulder. The majority of nerve injuries are temporary neurapraxias, which typically resolve within 6 to 12 months of injury; however, permanent nerve deficit can occur, requiring surgical intervention in the form of decompression, or reconstruction with nerve graft or nerve transfer from the radial nerve.

The lateral cord receives contribution from C5, C6, and C7 roots, and contributes to the musculocutaneous and median nerves. The medial cord receives contribution from C8 and T1 roots, and contributes to the median and ulnar nerves. There are no superior or anterior cords within the brachial plexus.

How well did you know this?
1
Not at all
2
3
4
5
Perfectly
28
Q

A 57-year-old woman comes to the office because of burning pain and stiffness of the right hand 8 weeks after closed treatment of a distal radius fracture. The patient reports that she has had difficulty sleeping and continues to have discomfort despite taking narcotics. On physical examination, the hand is shiny, swollen, and warm, and finger range of motion is decreased. There is hypersensitivity to light touch. X-ray studies show good alignment of the fracture. Electrodiagnostic testing shows no abnormalities. Bone scan shows increased periarticular uptake. Which of the following is the most appropriate diagnosis?

A) Complex regional pain syndrome
B) Factitious disorder
C) Midpalmar space abscess
D) Opioid addiction
E) Pain catastrophizing

A

The correct response is Option A.

The most appropriate diagnosis is reflex sympathetic dystrophy, or complex regional pain syndrome (CRPS) type I. This patient exhibits symptoms consistent with CRPS, which is a form of severe neuropathic pain. The diagnosis of CRPS involves history, physical examination, and diagnostic testing. In addition to pain out of proportion, other features must be present. These can include changes in blood flow, altered temperature perceptions, sudomotor activity, edema, and pigmentation changes. Although no specific test is pathognomonic, triple-phase bone scans are helpful in adding credence to the diagnosis. First- and second-phase bone scans may show asymmetric flow and autonomic dysfunction, while the third phase demonstrates increased periarticular uptake in multiple joints of the affected extremity.

CRPS is divided into two types. Type I occurs without identifiable nerve involvement (also known as reflex sympathetic dystrophy), and Type II has identifiable nerve involvement (causalgia). It is more common in smokers and in women. Pain in CRPS can be either sympathetically mediated or sympathetically independent. This condition is characterized by persistent pain, cold intolerance, autonomic dysfunction, and trophic changes. Patients may show swelling, stiffness, difficulty sleeping, and persistent pain out of proportion to the normal postoperative/post-injury course that may be incompletely relieved by narcotics.

A variety of treatment modalities have been employed in addressing CRPS. These range from therapy modalities such as range of motion, stress loading, and desensitization to pharmacologic interventions with anticonvulsants or antidepressants. Stellate ganglion blocks or autonomic nerve blocks may be helpful in sympathetically mediated pain, and nerve stimulation (either transcutaneous or at the spinal cord level) can also be employed. Often multiple modalities are used concurrently and in sequence. Peripheral nerve decompression may be helpful in resolving symptoms related to CRPS type II.

Factitious disorder can occur when there is potential for secondary gain, but would not present with physiological symptoms.

Although opioid addiction can be a source of pain complaints in an attempt to acquire additional narcotics, the patient exhibits physiological changes that are unable to be mimicked.

Pain catastrophizing is a maladaptive behavioral response to pain that can be a risk factor for prolonged pain after trauma.

A midpalmar space abscess would be unlikely after a closed distal radius fracture. It would also not be likely to have trophic skin changes or changes in a bone scan as seen in this patient.

How well did you know this?
1
Not at all
2
3
4
5
Perfectly
29
Q

The Jones transfer for radial nerve palsy, specifically flexor carpi ulnaris to extensor digitorum communis III-V, is most likely to have which of the following significant disadvantages?

A) Difficulty of dissection
B) Inappropriate excursion of donor tendon
C) Indirect line of pull requiring pulley creation
D) Poor synergy
E) Unacceptable postoperative rupture rate

A

The correct response is Option B.

Although the Jones transfers were practiced for years, there were significant disadvantages that led to them falling out of favor. Among the disadvantages are loss of flexor carpi ulnaris (FCU) as an important ulnar wrist stabilizer and weakness in flexion/ulnar deviation, which is a very important wrist motion. Additionally, the short excursion of FCU is inadequate to fully extend the fingers when transferred to extensor digitorum communis (EDC).

Subsequent modifications to the Jones technique by Boyes and others found better alternatives to the use of FCU. Standard Boyes transfers are:

PT to ECRL and ECRB
FCR to EPL and ABL
FDS-III to EDC (via interosseous membrane)
FDS-IV to EPL and EIP (via interosseous membrane)

How well did you know this?
1
Not at all
2
3
4
5
Perfectly
30
Q

An 18-year-old man presents for follow-up evaluation 8 weeks after he sustained a penetrating injury to the posterior medial right elbow and a complete transection of the ulnar nerve in the cubital tunnel. At this time, which of the following are the most likely Sunderland/Mackinnon injury grade and electromyogram/nerve conduction findings in this patient?

A

The correct response is Option D.

Nerve injuries are graded using the Sunderland/Mackinnon classification. Grade I injuries involve neurapraxia and are expected to recover completely; grades II to IV injuries involve increasing disruption of the perineurium and endoneurium (with expectation for a variable degree of spontaneous recovery); grade V injuries represent neurotmesis, or complete transection of the nerve. Grade VI injuries represent combined injuries in which more than one grade of injury exists within the same segment of damaged nerve. After a complete nerve transection and progression of Wallerian degeneration, patients develop fibrillations and sharp waves and progressive decrease in the compound motor action potential.

How well did you know this?
1
Not at all
2
3
4
5
Perfectly
31
Q

A 30-year-old man is brought to the emergency department after a rollover motor vehicle collision. Physical examination shows significant soft-tissue loss and a median nerve injury. At the time of surgical repair, there is a 5-cm gap in the nerve. Which of the following treatment options is most likely to provide the best long-term functional outcome for this patient?

A) Acellular autograft
B) Autologous vein graft
C) Collagen conduit
D) Peripheral nerve autograft
E) Polyglycolic acid conduit

A

The correct response is Option D.

For bridging long nerve gaps not amenable to primary repair (greater than 3 cm), peripheral nerve autografts are the most reliable choice. Their use is limited by supply and donor site morbidity from additional incisions, loss of sensation, and possible neuromas. Common donor sites include sural nerve, medial antebrachial cutaneous nerve, lateral antebrachial cutaneous nerve, dorsal cutaneous branch of the ulnar nerve, superficial peroneal nerve, and posterior and lateral cutaneous nerves of the thigh.

Nerve conduits, such as silicone tubes, synthetic biodegradable tubes (such as polyglycolic acid conduit or collagen conduit), and biologic tubes (such as autologous vein grafts) are limited to use in short gaps (less than 3 cm). Beyond 3 cm, there is no clinically meaningful regeneration. Acellular autografts have recently been used with good success in short nerve gaps (less than 3 cm). The advantage of these products is that they provide the extracellular matrix molecules, which may enhance nerve regeneration.

How well did you know this?
1
Not at all
2
3
4
5
Perfectly
32
Q

A 50-year-old male construction worker is evaluated for weakness of grip and pinch with inability to touch index finger to thumb. The patient is referred from a neurologist with a diagnosis of anterior interosseous nerve syndrome. Which of the following muscles is most likely to be spared?

A) Flexor digitorum profundus to index
B) Flexor digitorum profundus to middle
C) Flexor pollicis longus
D) Pronator quadratus
E) Pronator teres

A

The correct response is Option E.

The pronator teres is innervated by the median nerve prior to its take off of the anterior interosseous nerve (AIN), which is why it cannot be affected by AIN syndrome. Pronation generated by the pronator teres or the pronator quadratus would be indistinguishable clinically. The AIN is a terminal branch off the median nerve that innervates the flexor digitorum profundus to the index and middle fingers, flexor pollicis longus (FPL) and pronator quadratus. The AIN arises from the median nerve approximately 4 to 6 cm distal to the medial epicondyle. It travels between the flexor digitorum profundus (FDP) and flexor digitorum superficialis (FDS) initially, and then between the FPL and FDP. Then it lies on the volar surface of the interosseous membrane and travels with the anterior interosseous artery, terminating in the pronator quadratus and then the wrist joint capsule and the intercarpal, radiocarpal, and radioulnar joints. The nerve originates from C5-T12, becoming the medial and lateral cords of the brachial plexus and then becoming the median nerve.

How well did you know this?
1
Not at all
2
3
4
5
Perfectly
33
Q

Which of the following is the type of axon fiber (neuron) that is primarily involved with the autonomic changes that occur with complex regional pain syndrome (CRPS) type 1?

A) A delta sensory
B) Alpha motor
C) C sensory
D) Gamma motor
E) Ia sensory

A

C sensory fibers are responsible for a deeper, more non-localizable pain. C fibers can react to various stimuli, including thermal, mechanical, or chemical. C fibers respond to physiologic changes in the body, such as hypoxia, hypoglycemia, hypo-osmolarity, the presence of muscle metabolic products, and light or sensitive touch.

Paul Sudeck noticed that CRPS demonstrates classic inflammatory signs such as pain, swelling, erythema, hyperthermia, and impaired function. However, clinical chemistry markers of inflammation are not elevated. These findings imply a neurogenic inflammation. C fibers have an afferent function in the mediation of pain (and itch), but also an efferent neurosecretory function. They release neuropeptides such as substance P and calcitonin-gene-related peptide (CGRP). The presence of these neuropeptides might explain trophic and autonomic symptoms such as swelling, erythema, and hyperhidrosis. Elevated CGRP levels are also associated with autonomic disturbances, mainly with hyperhidrosis. Also, a role for CGRP in hair growth is suggested, and substance P seems to be involved in the regulation of osteoclastic activity.

Alpha motoneurons innervate muscle fibers of skeletal muscle and are directly responsible for initiating their contraction. When the central nervous system sends out signals to alpha neurons to fire, signals are also sent to gamma motoneurons to do the same. This process maintains the tautness of muscle spindles and is called alpha gamma co-activation. Without gamma motoneurons, muscle spindles would be very loose as the muscle contracts. Unrestricted alpha activity would not allow for muscle spindles to detect a precise amount of stretch and would not allow for optimization of muscle function.

Ia sensory fibers are a type of proprioceptor that is found inside the muscle itself. They lie parallel to the contractile fibers, and give them the ability to precisely monitor muscle length.

A delta fiber is a type of sensory nerve fiber. A delta fibers carry cold, pressure and some pain signals. Because A delta fibers have a higher conduction velocity, and are responsible for quick, shallow pain to a specific area. They are activated by a stimulus of weaker intensity, and are not responsible for the autonomic changes seen with CRPS.

How well did you know this?
1
Not at all
2
3
4
5
Perfectly
34
Q

A 24-year-old man comes to the office because of a brachial plexus injury sustained in a motorcycle accident. Nerve transfer to the biceps for restoration of elbow flexion is planned. Which of the following fascicles or nerves is the most appropriate donor for the transfer?

A) Distal spinal accessory nerve
B) Flexor carpi ulnaris fascicle of the ulnar nerve
C) Medial pectoral nerve
D) Palmaris longus fascicle of the median nerve
E) Thoracodorsal nerve

A

The correct response is Option B.

Adult upper trunk brachial plexus injuries result in significant disability. Several surgical treatment strategies exist, including nerve grafting, nerve transfers, and a combination of both approaches.

The flexor carpi ulnaris (FCU) fascicle of the ulnar nerve to biceps transfer was first described by Oberlin et al in 1994. Generally, the donor nerve with the largest caliber and the greatest number of motor axons should be used for elbow flexion. The other suggested nerve transfer options are also possibilities, but are not as preferable as the FCU fascicle transfer. MacKinnon has advocated transfer of the FCU fascicle of the ulnar nerve to the biceps and FCR fascicle of the median nerve to the brachialis to maximize recovery of elbow flexion.

How well did you know this?
1
Not at all
2
3
4
5
Perfectly
35
Q

A 22-year-old man comes to the emergency department after sustaining a laceration to the dorsal thumb by punching a glass door. Radial nerve block is planned during surgical repair. Which of the following is the approximate distance proximal to the radial styloid in which the superficial branch of the radial nerve pierces the deep fascia?

A) 0 to 4 cm
B) 5 to 9 cm
C) 10 to 14 cm
D) 15 to 19 cm

A

The correct response is Option B.

The superficial branch of the radial nerve runs below the brachioradialis muscle in the mid-forearm, later becoming sub-fascial between the brachioradialis and extensor carpi radialis longus (ECRL) muscles. Approximately 8 to 9 cm proximal to the radial styloid, the superficial branch of the radial nerve (SBRN) becomes subcutaneous, piercing the fascia. The ideal location of infiltration for a radial nerve block is at the sub-fascial location just before the nerve becomes subcutaneous.

How well did you know this?
1
Not at all
2
3
4
5
Perfectly
36
Q

A 53-year-old woman is evaluated for a 1-year history of numbness and tingling of the thumb and the index and long fingers of the right hand. She has been unresponsive to conservative treatment. An increase in which of the following is most likely suggestive of carpal tunnel syndrome in this patient?

A) Abductor digiti minimi fibrillations
B) Adductor pollicis fibrillations
C) Motor nerve conduction velocity
D) Sensory distal latency
E) Sensory nerve conduction velocity

A

The correct response is Option D.

The diagnosis of carpal tunnel syndrome is primarily a clinical diagnosis; however, electrodiagnostic studies (EDX) may be helpful in confirming the diagnosis. While these EDX studies are commonly referred to as “EMGs,”’ they are actually two separate studies: the nerve conduction studies (NCS) and the electromyography (EMG). NCS examine both the sensory and the motor nerve fibers. Sensory nerve conduction studies measure sensory nerve action potential, and the motor nerve conduction studies evaluate a compound muscle action potential. The NCS also measures the amplitude of both the compound muscle action potential and sensory nerve action potential. Nerve conduction velocity (NCV), the velocity of the nerve’s action potential between two points, is also measured by the nerve conduction studies.

The EMG tests the muscle itself. The needle electrode examination can measure motor unit potential (MUP). MUP is measured in regards to its amplitude, duration, wave shape, and firing pattern. In the diagnosis of carpal tunnel syndrome, particular attention is given to the MUP of the abductor pollicis brevis muscle, which is uniquely innervated by the median nerve after it passes through the carpal tunnel.

In the diagnosis of carpal tunnel syndrome, changes in the sensory nerve are detected earlier in the carpal tunnel process than motor changes. Early NCS changes (as compared to standardized normal values) include prolonged or increased sensory distal latencies. Prolonged motor latencies (also abnormal) are detected less frequently than the sensory latency changes, and detected in only 35 to 50% of patients with carpal tunnel syndrome. Motor amplitude change, found in carpal tunnel syndrome, is detected even less commonly. A conduction block, or slowing of the nerve’s action potential velocity (NCV) between two points, can be seen with carpal tunnel syndrome.

The needle electrode examination (EMG) is normal in more than 60% of patients with the diagnosis of carpal tunnel syndrome. Fibrillations in the abductor pollicis brevis occurs in generally less than 20% of patients with carpal tunnel syndrome. The adductor pollicis brevis and abductor digiti minimi muscles are innervated by the ulnar nerve, and would not show any electrodiagnostic evidence of muscle instability in isolated carpal tunnel syndrome.

How well did you know this?
1
Not at all
2
3
4
5
Perfectly
37
Q

A 47-year-old man is brought to the emergency department after sustaining a stab wound injury to the left shoulder. Physical examination shows isolated loss of deltoid function. The injured nerve is supplied by which of the following nerve roots?

A) C5 through C6
B) C6 through C7
C) C7 only
D) C7 through T1
E) T1 only

A

The correct response is Option A.

The deltoid muscle receives motor innervation from the axillary nerve. The axillary nerve receives its contributions from C5 and C6 roots. These roots come together to form the superior trunk, which splits into anterior and posterior divisions. The axons heading to the axillary nerve travel in the posterior division, which joins the other posterior divisions from middle and inferior trunks to form the posterior cord. The axillary nerve arises from the posterior cord and travels laterally to innervate the deltoid muscle.

C7 is the primary innervation to the latissimus dorsi and triceps, and contributes to digital extension as well. C8 and T1 primarily serve the hand, providing intrinsic muscle innervation.

How well did you know this?
1
Not at all
2
3
4
5
Perfectly
38
Q

A patient with severe traumatic brachial plexus root avulsion injury is scheduled to undergo functioning free muscle transfer for simultaneous restoration of both elbow flexion and finger flexion, in addition to other reconstructive procedures. Use of which of the following muscles is most appropriate for this purpose?

A) Gracilis
B) Pectoralis major
C) Rectus abdominus
D) Serratus anterior
E) Trapezius

A

The correct response is Option A.

The gracilis muscle is the most commonly described muscle for use as a free functioning muscle in reconstruction of upper extremity function following brachial plexus injury. Common options for use in these reconstructions include the gracilis, latissimus dorsi, rectus femoris, and vastus lateralis. The gracilis muscle has good excursion, size, and length, but does lack strength compared with some other muscle options.

The rectus abdominis, serratus anterior, and trapezius muscles have not been described for free functioning muscle transfer in the upper extremity. Though the pectoralis major muscle was described as a free functioning muscle transfer by Manktelow and McKee in 1978, it has not been a commonly used muscle.

How well did you know this?
1
Not at all
2
3
4
5
Perfectly
39
Q

An 8-year-old boy is brought to the emergency department after sustaining injury to the right upper extremity, 3-cm proximal to the antecubital fossa. Which of the following factors is associated with improved functional outcomes following peripheral nerve repair?

A) Fewer suture strands used in the nerve repair
B) Higher-tension nerve repair
C) Increasing time between nerve injury and repair
D) More proximal nerve injury
E) Younger patient age

A

The correct response is Option E.

The repair of peripheral nerve injuries can be affected by several factors. Younger patients tend to have improved outcomes compared with older patients. Although there is no consensus on the optimal timing for nerve repair, earlier repairs have been shown to have better outcomes than those attempted at later time points. The level at which the injury has occurred can also affect the outcome. The more proximal the injury, the worse the prognosis in terms of motor and sensory return. Moreover, more complete and rapid regain of function occurs in more proximally innervated muscles. Finally, technical aspects of the nerve repair can also affect outcomes. Minimal tension and an increasing number of suture strands crossing the repair site are both associated with improved function.

How well did you know this?
1
Not at all
2
3
4
5
Perfectly
40
Q

A 25-year-old man comes to the office after sustaining a deep laceration to the elbow. Physical examination shows decreased function of the ulnar nerve, and the patient is taken for operative exploration and repair. Following proximal and distal dissection, a 1-cm gap between the proximal and distal nerve ends persists. Which of the following is the most appropriate next step in management?

A) Nerve transfer
B) Nerve transposition
C) Polyglycolic acid nerve conduit
D) Primary repair
E) Sural nerve grafting

A

The correct response is Option B.

Principles of microsurgical nerve repair include the use of meticulous and atraumatic technique with adequate magnification, microsurgical instruments, and sutures. A primary repair is performed whenever possible, provided that the repair is tension-free in order to maximize perfusion to the repair site.

In this patient, a 1-cm nerve gap in the ulnar nerve was present even after mobilizing the proximal and distal nerve ends. In this situation, the ulnar nerve may be transposed anteriorly, which would shorten the distance between the nerve ends and allow for primary repair.

Nerve transfers are indicated in very proximal nerve injuries where a proximal stump is unavailable for primary repair or grafting, or when a very long nerve gap is present where there would be a concern that target muscle denervation might occur prior to nerve regeneration.

Polyglycolic acid nerve conduits are bioabsorbable tubes through which nerve regeneration occurs. They represent an option for nerve reconstruction without any associated donor site morbidity when a nerve gap is present in order to achieve a tension-free repair.

Although primary nerve repair is preferable to the use of a graft/conduit, doing so in the setting of this patient’s 1-cm nerve gap would not result in a tension-free repair.

Autologous nerve grafting, such as with the sural nerve, is an option for nerve reconstruction when a nerve gap is present in order to achieve a tension-free repair.

How well did you know this?
1
Not at all
2
3
4
5
Perfectly
41
Q

A 33-year-old woman comes to the office because of volar numbness of the right thumb, index, long finger, and palm for the past 4 months. Medical history includes no abnormalities. Physical examination shows weakness of the palmar abduction of the thumb and interphalangeal joint flexion of the thumb. Percussion tenderness over the volar aspect of the wrist does not reproduce symptoms. Passive flexion of the wrist does not reproduce symptoms. Which of the following is the most likely diagnosis?

A) Anterior interosseous syndrome
B) Carpal tunnel syndrome
C) Cubital tunnel syndrome
D) Posterior interosseous syndrome
E) Pronator syndrome

A

The correct response is Option E.

Pronator syndrome is a compression neuropathy of the median nerve in the proximal forearm. Paresthesias in the palm and the median nerve–innervated fingers of the thumb along with weakness of the flexor pollicis longus muscle are classic findings in pronator syndrome. Pronator syndrome can be differentiated from carpal tunnel syndrome by sensory symptoms and numbness occurring in the palm, which is innervated by the palmar cutaneous branch of the median nerve. This branch comes off the proper median nerve proximal to the carpal tunnel. Findings of weakness of the flexor pollicis longus muscle and often the flexor digitorum profundus muscle to the index finger are often seen in pronator syndrome. Anterior interosseous syndrome is a motor-only compression of the median nerve in the proximal forearm; sensory findings are not present. Cubital tunnel syndrome or ulnar nerve compression at the elbow presents with sensory findings in the small and ring fingers and may also affect the interosseous muscles. Posterior interosseous syndrome affects the extensor muscles in the forearm and is not the pathologic lesion described in the scenario.

How well did you know this?
1
Not at all
2
3
4
5
Perfectly
42
Q

A 30-year-old woman comes to the office because of burning pain in the left wrist 5 months after discharge from the hospital. During her visit, a catheter was inserted in the left radial artery for arterial line monitoring in the intensive care unit. On examination, point tenderness and Tinel sign are noted over the volar radial aspect of the forearm, just ulnar to the radial artery, and overlying the flexor carpi radialis tendon. Which of the following is the most likely nerve of origin for the suspected condition?

A) Anterior interosseous
B) Median
C) Musculocutaneous
D) Radial
E) Ulnar

A

The correct response is Option C.

The neuroma is of the lateral antebrachial cutaneous nerve, which innervates the area in question, over the volar forearm, including the skin overlying the flexor carpi radialis (FCR) tendon. The lateral antebrachial cutaneous nerve is the continuation of the musculocutaneous nerve in the forearm.

The dorsal radial sensory nerve innervates the skin overlying the dorsal and radial aspects of the wrist and does not innervate the skin overlying the FCR tendon. Therefore, the radial nerve is not appropriate.

The ulnar and median nerves are not appropriate, as they give off sensory contributions in the palm and fingers. The anterior interosseous nerve is mainly a motor nerve in the forearm, finally sending off branches to the volar capsule of the wrist.

How well did you know this?
1
Not at all
2
3
4
5
Perfectly
43
Q

A 29-year-old man is brought to the emergency department because of a deep laceration of the medial right arm at the elbow. Examination shows complete loss of ulnar nerve function to the right hand. During exploration in the operating room, complete transection of the ulnar nerve at the elbow with a 3-cm nerve gap after debridement is noted. Which of the following procedures is most likely to result in the earliest recovery of intrinsic muscle function in this patient’s hand?

A) Anterior interosseous nerve transfer
B) Cable grafting with nerve allografting
C) Cable grafting with sural nerve grafting
D) Primary repair with the elbow in flexion
E) Ulnar nerve transposition and primary repair

A

The correct response is Option A.

The most appropriate answer is transfer of the anterior interosseous nerve to the motor branch of the ulnar nerve in the distal forearm. Ulnar nerve injuries are especially debilitating with loss of grip and claw hand deformity. High ulnar nerve injuries are considered to be anything proximal to the innervation of the flexor carpi ulnaris and flexor digitorum profundus muscles at or near the elbow. This includes the elbow, upper arm, and brachial plexus.

The prognosis for recovery of intrinsic hand function is poor in high ulnar nerve injuries 30 to 35 cm proximal to the hypothenar eminence. This is due to the length of time required for nerve regeneration, and the motor endplate degeneration that occurs during this time. High ulnar nerve injuries will exceed the approximately 18-month window for regeneration in order to achieve meaningful muscle recovery.

The theory behind nerve transfer surgery is to take an expendable donor nerve and use the fascicles to restore function to a more critically injured nerve. The anastamosis for an AIN-to-ulnar nerve transfer is 8 to 10 cm proximal to the wrist crease and greatly decreases the amount of distance and therefore time required for reinnervation of intrinsic hand muscles. Although the anterior interosseous nerve contains 75% of the axons of the deep motor branch of the ulnar nerve, meaningful recovery of intrinsic muscle function can be expected. The anastamosis for the transfer can be done in an end:end or end:side technique. End-to-side nerve transfer is indicated in partial nerve injuries or lower nerve injuries where primary repair of the injured nerve is possible and one can expect some contribution of nerve fibers from the native nerve.

Primary repair of any nerve injury under tension or that requires extreme joint flexion is not indicated. This results in tension at the repair site, internal scarring, possible flexion contracture, and a poor result. Anterior transposition of the ulnar nerve may be performed in some instances but is reported to gain only 1 cm of length from the proximal nerve. This is not enough to bridge the gap in this scenario.

Any attempt at primary repair of a nerve injury with a 3-cm gap would require nerve grafting. In a mixed nerve, multiple cables of nerve graft are recommended in an attempt to topographically reconnect the sensory and motor fascicular bundles. The current gold standard for nerve repair in adults is autograft. The most common donor nerves are the sural nerve and medial antebrachial cutaneous nerve.

Processed nerve allograft has become a viable alternative to autograft nerve. The allograft nerve is processed and decellularized but maintains the microstructure of the nerve tissue including the fascicular anatomy and microvasculature. The allograft is rapidly revascularized without the donor site morbidity associated with autograft. The RANGER study has demonstrated S3 and M4 or above recovery in 86% of repairs using allograft nerve in gaps up to 5 cm. This question specifically asks about the most rapid recovery of motor function, which should occur with a nerve transfer.

How well did you know this?
1
Not at all
2
3
4
5
Perfectly
44
Q

A 30-year-old man undergoes evaluation of a stab wound to the left upper extremity. A photograph is shown. Examination shows that the injured nerve is completely transected. Supination is weak, and he cannot flex his elbow in a supinated position. Which of the following areas is most likely to be insensate?

A) Dorsal little finger
B) Index finger pad
C) Posterior arm
D) Radial forearm
E) Thumb pad

A

The correct response is Option D.

The patient has an injury to the musculocutaneous nerve. This nerve provides motor axons to the brachialis, biceps brachii, and coracobrachialis. Patients with a musculocutaneous nerve transection cannot flex the elbow when supinated; the brachioradialis would provide some elbow flexion in a pronated position. The biceps brachii is the strongest supinator of the forearm, so patients with this injury have weak supination. The musculocutaneous nerve also provides sensory axons to the lateral brachial and lateral antebrachial cutaneous nerves. Patients with transection of this nerve would be insensate on the lateral arm and the radial side of the forearm.

Sensation to the posterior arm is provided by the posterior brachial cutaneous nerve, a branch of the radial nerve.

Sensation to the thumb pad is provided by the median nerve.

Sensation to the dorsal little finger proximally is provided by the dorsal sensory branch of the ulnar nerve and distally by the ulnar and radial proper digital nerves, branches of the ulnar nerve.

Sensation to the index finger pad is provided by the median nerve.

How well did you know this?
1
Not at all
2
3
4
5
Perfectly
45
Q

A 25-year-old woman comes to the office because of nerve compression of the right upper extremity. Electromyography and nerve conduction studies are planned. Which of the following is the most likely indicator of motor axon loss in this patient?

A) Absent polyphasic waveforms
B) Decreased distal motor latency
C) Fibrillation potentials
D) Increased amplitude
E) Increased conduction velocity

A

The correct response is Option C.

Specific electrodiagnostic criteria indicate axonal loss: nerve conduction study amplitudes are decreased, conduction velocity is slowed, distal latency is prolonged, and fibrillation potentials and polyphasic waveforms are present.

How well did you know this?
1
Not at all
2
3
4
5
Perfectly
46
Q

A 50-year-old man who is homeless is brought by ambulance to the emergency department. His blood alcohol concentration is 325 mg/dL. Examination of the right hand and forearm shows absent palpable pulses at the radial and ulnar arteries. Compartment pressure is 55 mmHg. Which of the following nerves is most likely irreversibly affected in this patient?

A) Lateral antebrachial
B) Medial antebrachial
C) Median
D) Radial
E) Axillary

A

The correct response is Option C.

The most appropriate answer is median. Pathophysiology of Volkmann’s contracture begins with the deep and central muscles, which include flexor digitorum profundus and flexor pollicis longus. The next affected is the middle layer, which includes flexor digitorum superficialis and pronator teres and then the wrist flexors. Lastly, the extensor forearm is affected.

In terms of nerve sensitivities, beginning at 30 mmHg, there are decreased conduction velocities. At 50 mmHg, there is no conduction. After 8 hours, there is irreversible damage.

The median nerve is affected before the ulnar nerve. The radial nerve is dorsal and not in the deep compartment. Both antebrachial nerves are superficial. The axillary nerve does not go to the forearm.

Alcohol is a clear comorbidity in this patient and therefore the timing is unknown. The pulselessness in this case indicates a late finding of compartment syndrome. Pain out of proportion along with paraesthesias and pressure is an early sign. Other late signs include pallor and paralysis.

How well did you know this?
1
Not at all
2
3
4
5
Perfectly
47
Q

A 24-year-old man is brought to the emergency department 5 hours after being involved in a motorcycle collision. The patient reports worsening pain of the left forearm despite previous opioid administration. On physical examination, the left radial pulse is easily palpable. Hypoesthesia in a median nerve distribution is noted in the left hand. Active motion of wrist and fingers is present but is minimal and limited by pain. Passive wrist motion produces intense pain in the left forearm. X-ray study of the left upper extremity shows no fracture, and remaining trauma workup is negative. A photograph is shown. Which of the following is the most appropriate next step in management?

A) Carpal tunnel release
B) Forearm fasciotomy
C) Local wound care until demarcation of tissue loss
D) Splinting and observation
E) Wound debridement and skin grafting

A

The correct response is Option B.

This patient presents with acute compartment syndrome (ACS) in the left forearm after blunt soft tissue trauma. The most appropriate next step in management is emergent forearm fasciotomy for decompression.

The diagnosis of ACS in the extremities should be based on clinical presentation and progress. Signs and symptoms commonly evolve over a few hours after the initial traumatic event, especially after fluid resuscitation. Occasionally, the patient’s initial presentation may raise strong suspicion and warrant immediate decompression. ACS in the extremities may develop from soft tissue injury without a fracture in up to 30% of cases.

The commonly accepted clinical findings suggestive of ACS (rest pain, pain on passive motion, paresthesia, and paresis) have been shown to have high specificity but low sensitivity, rendering them a poor predictive value. Paresis/paralysis and other signs of arterial obstruction (pulselessness, pallor, pain out of proportion) are thought to be particularly late findings. Therefore, physicians should have a high level of suspicion for ACS in any case of limb trauma associated with excessive pain and paresthesia. Measurement of compartment pressures may be of diagnostic assistance in equivocal cases, especially when the patient is unable to communicate.

Carpal tunnel release is an important component of a forearm fasciotomy; however, it is not sufficient decompression alone. Emergent decompressive fasciotomy should precede wound debridement, demarcation of tissue loss, and splinting of the extremity. Skin grafting of the wounds (including the skin incisions for fasciotomy) may or may not be needed after resolution of limb edema.

How well did you know this?
1
Not at all
2
3
4
5
Perfectly
48
Q

A 45-year-old man sustains a laceration of the ulnar nerve proximal to the elbow. He has loss of intrinsic hand function. At the time of surgical repair, there is a 1-cm gap in the nerve. Which of the following is the most appropriate management of this injury?

A) Interposition nerve conduit
B) Interposition nerve grafting
C) Primary repair only
D) Primary repair with anterior transposition
E) Primary repair with distal nerve transfer

A

The correct response is Option E.

The most appropriate management of this injury is to repair the ulnar nerve primarily and perform a distal nerve transfer as well.

The ulnar nerve is one of the two upper extremity nerves that supply motor input to the intrinsic muscles of the hand (the other being the median nerve). It supplies the interossei, hypothenar muscles, ulnar lumbricals, and the adductor pollicis muscles. These small muscles of the hand are vital for proper thumb and finger function. Because of their small size and delicate nature, they are very sensitive to denervation, and over a period of 9 to 12 months atrophy beyond repair. The aim of nerve repair is to reestablish nerve signals to the end-organ (i.e., muscle) prior to irreversible denervated muscle. The injury described is a good 18 inches or so away from the hand, and regeneration of the motor fibers to the intrinsic muscles would only occur after at a rate of an inch a month, by which time permanent atrophy would have already occurred. Primary repair alone can lead to suboptimal intrinsic muscle function despite excellent technical repairs. Distal nerve transfers help prevent the denervational atrophy by “babysitting” the muscles during the time it takes for the ulnar nerve to regenerate its motor fibers to the end-organ/muscles. The terminal branch of the anterior interosseous nerve (AIN) is most commonly used as the donor nerve. An end-side neurorrhaphy is performed to the ulnar motor fascicles in the distal forearm, a distance which results in reinnervation of the intrinsic muscles well before the 9- to 12-month mark.

The known topography of the ulnar nerve allows the surgeon to coapt the donor nerves to the appropriate motor recipient site of the ulnar nerve. An end-end coaptation of the terminal AIN to the motor fascicle of the ulnar nerve is also an option.

Nerve grafts or conduits are not required to repair the nerve injury in this clinical scenario. A 1-cm gap can usually be primarily repaired after dissecting the nerve and freeing it up proximally and distally. If further length is needed for tension-free repair, the ulnar nerve can be transposed anteriorly out of its natural position, giving another few centimeters of length.

How well did you know this?
1
Not at all
2
3
4
5
Perfectly
49
Q

A 35-year-old woman comes for evaluation of a 6-month history of increasing numbness of the right long finger of the dominant hand. She delivered a healthy newborn 6 months ago. She reports that the numbness awakens her from sleep and resolves after she shakes her hand. Physical examination shows a two-point discrimination of 5 mm in all digits. Which of the following is the most likely abnormal electrodiagnostic finding in this patient?

A) Motor action potential amplitude of 25 ?V
B) Motor latency of 4 ms
C) Occasional fasciculations
D) Positive sharp waves
E) Sensory action potential latency of 4 ms

A

The correct response is Option E.

The patient has early carpal tunnel syndrome. She only exhibits sensory findings with numbness that resolves. Two-point discrimination is abnormal in late carpal tunnel syndrome when there is irreversible nerve damage. There are no motor symptoms at this point.

Electrodiagnostic studies are a two-part examination consisting of sensory action potentials (SAP), also referred to as nerve conduction studies (NCS) and electromyography (EMG). Findings in EMG latency for muscle abnormality are >4.0 ms. Normal muscles show occasional fasciculations with high fibrillations 5 weeks after denervation. Normal motor nerve shows no sharp waves and no fasciculations. Mild motor latencies on NCS/EMG are >4.0 ms. Motor latencies of >6.0 are considered severe. Because this is early carpal tunnel syndrome with no evidence of motor weakness, a motor latency of 4.0 is unlikely. Similarly, a motor action potential of 25 ?V is unlikely.

Findings in SAP distal latency for sensory abnormality are >3.5 ms and an amplitude <15 ?V (normal is 15-25 ?V). Therefore, sensory action potential latency of 4.0 ms is correct.

How well did you know this?
1
Not at all
2
3
4
5
Perfectly
50
Q

A 42-year-old man with carpal tunnel syndrome is evaluated for symptoms that are progressively worsening despite conservative management. Surgical release using an open, short scar technique is planned. Which of the following is the most accurate statement when comparing this technique with endoscopic release?

A) Both techniques are equivalent in long-term symptom relief and recovery
B) Endoscopic release is far superior because of superior long-term symptom relief
C) Endoscopic release is only used for bilateral cases
D) Open, short scar technique requires regional block, whereas endoscopic release does not
E) Open technique has a higher association with recurrent median nerve injury

A

The correct response is Option A.

Open, short incision and endoscopic carpal tunnel release are equivalent in long-term symptom relief and recovery. Carpal tunnel syndrome is a condition caused by compression of the median nerve at the wrist. It is characterized by pain and numbness of the fingers within the median nerve distribution: the thumb, index, and long fingers, as well as the radial aspect of the ring fingers. With progressive compression, thenar atrophy can occur as well as weakness of thumb opposition. Conservative treatment includes splinting, avoidance of repetitive activities or positions that elicit symptoms, and occasionally steroid injection. With progression of symptoms, surgical release is indicated.

Open release provides transcutaneous access to the transverse carpal ligament. Traditionally, a long incision had been used extending from the proximal palm across the wrist and onto the proximal forearm. The incision has become progressively shorter, such that most surgeons employ a short scar confined to the proximal palm. Endoscopic release uses two small incisions for port access and provides transection of the transverse carpal ligament without division of the palmar aponeurosis.

Debate has existed regarding the superiority or inferiority of one technique over the others. Clearly, the endoscopic technique causes less pain and less alteration in early grip strength, when compared with the more classic, longer incision open techniques. However, when specifically comparing the open, limited scar technique to the endoscopic technique, studies have shown essentially the same outcome data regarding strength, return to work, symptom relief, and reoperation.

Virtually all studies have shown that open and endoscopic release have the same long-term symptom relief, measured at multiple points in time up to one year.

The open technique is often thought to be associated with a lower association with recurrent median nerve injury.

Bilaterality does not preclude open or endoscopic release.

Both techniques can be done during local or regional anesthesia.

How well did you know this?
1
Not at all
2
3
4
5
Perfectly
51
Q

A 68-year-old woman is evaluated because of numbness and paresthesias of the right hand. The diagnosis of carpal tunnel syndrome is confirmed by electromyography and nerve conduction velocity studies. The patient is interested in a trial of nonsurgical management. Nighttime splint immobilization and corticosteroid injection therapy to the carpal tunnel are planned. Which of the following best describes the most likely long-term outcome of this management strategy?

A) Complete resolution of the patient’s symptoms
B) No effect on the patient’s symptoms
C) No initial effect on the patient’s symptoms followed by gradual improvement over 6 to 12 months
D) Short-term improvement of the patient’s symptoms followed by a recurrence in 6 to 12 months
E) Worsening of the patient’s sensory and motor symptoms

A

The correct response is Option D.

Nonsurgical management of carpal tunnel syndrome has been shown to be effective for symptomatic relief of carpal tunnel syndrome for up to 3 to 6 months, depending on the type of intervention that is chosen. Most patients who choose nonsurgical management will ultimately require surgery for carpal tunnel syndrome. Studies suggest that up to 70% of patients will go on to surgical carpal tunnel release at 1 year.

Various nonsurgical interventions have been recommended for carpal tunnel syndrome. These include splint immobilization, oral corticosteroid therapy, local corticosteroid injection, ultrasonography, acupuncture, nonsteroidal anti-inflammatory drug (NSAID) therapy, and more. Many of the interventions are not supported by high-level evidence.

Studies have shown improvement in carpal tunnel syndrome with splint immobilization, oral corticosteroid therapy, local corticosteroid injections, and ultrasonography when compared with a placebo or no treatment. Local corticosteroid injection into the carpal tunnel is more effective than oral corticosteroid therapy at 1 and 3 months without the potential risks of systemic steroid therapy. Splint immobilization plus a cortisone injection is more effective than splint immobilization only at 6 months. Full-time splint immobilization has not shown any benefit when compared with nighttime splint immobilization only.

In patients who have mild to moderate carpal tunnel syndrome without static numbness, weakness, or thenar atrophy, short-term improvement or resolution of symptoms can be expected but long-term resolution is not likely.

Patients with severe disease and signs of longstanding nerve compression including numbness and loss of abductor pollicis brevis strength may not respond to nonsurgical interventions. Worsening of symptoms as a result of nonsurgical treatment is unlikely.

Carpal tunnel release has been shown to be more effective than splint immobilization and corticosteroid injections for symptomatic relief at 3, 6, and 12 months. However, many surgeons will offer a trial of nonsurgical management based on patient preferences. Complete resolution of symptoms following nonsurgical intervention is thought to be a good prognostic indicator for the success of surgery.

How well did you know this?
1
Not at all
2
3
4
5
Perfectly
52
Q

A 26-year-old man is referred for evaluation 9 months after sustaining a lower trunk brachial plexus traction injury after being hit by a motor vehicle. At the time of injury, he had paresthesia in the ring and small fingers and weakness of grip. Today, physical examination shows intrinsic atrophy and mild ulnar claw hand. A photograph is shown. He demonstrates increased sensation in the ring and small fingers since the time of the accident, but he has no clinical or electrodiagnostic improvement in motor function. Which of the following is most likely to improve the claw hand posture?

A) Anterior interosseous nerve branch of median nerve to ulnar nerve transfer in forearm
B) Brachialis branch of musculocutaneous nerve to ulnar nerve transfer in upper arm
C) Exploration and neurolysis of the lower trunk of the brachial plexus
D) Posterior interosseous nerve branch of radial nerve to ulnar nerve transfer in forearm

A

The correct response is Option A.

Lower trunk brachial plexus injuries can lead to deficits in hand function. Some can cause global hand dysfunction (median and ulnar nerve), while others can present as isolated ulnar nerve dysfunction. The signs and symptoms of isolated ulnar nerve dysfunction include numbness and tingling in the ring and small fingers, as well as weakness of the intrinsics. Ulnar nerve dysfunction may or may not present with clawing of the ulnar two digits, depending upon whether the flexor digitorum profundus (FDP) tendon to the ring and small fingers is involved. This patient has signs and symptoms consistent with low ulnar nerve palsy, and would benefit from an anterior interosseous nerve (AIN) to ulnar motor nerve transfer in the forearm.

The brachialis branch of the musculocutaneous nerve is classically used to reinnervate the AIN in the upper arm. This transfer is employed in cases of AIN palsy. The extensor carpi radialis brevis branch of the posterior interosseous nerve is most commonly used to reinnervate the AIN (median nerve), but can also be transferred to the ulnar nerve. This transfer occurs more proximally in the forearm and, therefore, would require a greater distance to travel to reach its endpoint (intrinsics of hand). The better and more appropriate transfer is the AIN to ulnar motor in this case.

Internal neurolysis can be used for neuromas in situ or neuropraxic nerves, but it would not be indicated for deficit in the hand. The length needed to recover function is too great, and motor end plate death is likely to occur before meaningful recovery can be seen.

How well did you know this?
1
Not at all
2
3
4
5
Perfectly
53
Q

A 6-month-old boy who sustained a brachial plexus injury during delivery is brought for evaluation. On examination, the left elbow is held in extension and arm is positioned in internal rotation. He has active flexion of the fingers and wrist. Extension of the fingers and wrist is weak; anti-gravity test of the elbow extension discloses weakness. There is no active elbow flexion or shoulder abduction; Horner’s sign is absent. Which of the following structures is most likely injured?

A) C7 roots
B) C8-T1 roots
C) Lateral cord
D) Posterior cord
E) Upper trunk

A

The correct response is Option E.

The infant has the classic manifestations of an upper trunk (C5-6), or Erb’s, palsy (weak or absent elbow flexion, shoulder abduction and external rotation, relatively preserved elbow extension and distal wrist/hand flexion). The “waiter’s tip” posture of the affected extremity indicates relative sparing of lower root (C8-T1) function. Isolated injury to the C7 root is uncommon and would primarily affect radial nerve innervated muscles such as the wrist and finger extensors, and forearm pronation. Injury to the posterior cord also effects radial nerve innervated muscles and would not alter biceps or deltoid function as observed in this child. Lateral cord damage would result in weak or absent biceps and pectoral function, but would not diminish deltoid or periscapular muscle activity.

How well did you know this?
1
Not at all
2
3
4
5
Perfectly
54
Q

A 14-year-old boy sustains a laceration of the distal forearm. Physical examination and wound exploration suggest flexor carpi ulnaris tendon and ulnar nerve involvement. The patient undergoes immediate microsurgical nerve repair with a nerve graft. Which of the following factors is most likely to predict a satisfactory outcome in this patient?

A) Age
B) Gender
C) Immediate repair
D) Ulnar nerve involvement
E) Use of nerve graft

A

The correct response is Option A.

Multiple studies have evaluated outcomes of median and ulnar nerve repair after transection injury. A meta-analysis by Ruijs et al. confirmed that younger age, specifically under 16 years old, was associated with the highest chance of satisfactory recovery of motor function in both median and ulnar nerve injuries. Patient gender was not found to be a statistically significant factor in outcome. In the same analysis, median motor nerve injuries were found to have a better chance of recovery than ulnar motor nerve injuries. Timing influenced outcome, with delay of repair adversely affecting prognosis, and although the ideal window for repair was not able to be defined by this review, there is some evidence that immediate repair may result in worse outcomes. The use of nerve grafts did not significantly predict motor recovery in these injuries.

How well did you know this?
1
Not at all
2
3
4
5
Perfectly
55
Q

A 65-year-old woman undergoes open reduction and internal fixation for the injury displayed in the x-ray study shown. Six weeks postoperatively, she has diffuse swelling, stiffness, and pain of the right upper extremity. Her skin appears shiny, and she has changes in the patterns of both hair and perspiration in the right upper extremity when compared with the left upper extremity. After injury, treatment with which of the following would most likely have decreased her risk for this complication?

A) Amitriptyline
B) Ascorbic acid
C) Gabapentin
D) Prednisone
E) Pregabalin

A

The correct response is Option B.

Complex regional pain syndrome (CRPS) is chronic pain that persists in the absence of ongoing cellular damage and is characterized by autonomic dysfunction, trophic changes, and impaired function. In the perioperative period, the physiologic consequences of CRPS in the upper extremity contribute to or create one or more of the following: clinically significant osteopenia, delayed bony healing or nonunion, joint stiffness, tendon adhesions, arthrofibrosis, pseudo-Dupuytren palmar fibrosis, swelling, and atrophy. The reported incidence of CRPS is 5.5 to 26.2 per 100,000 person-years, and the prevalence is reported as 20.7 per 100,000 person-years. Women are more frequently affected than men, with a ratio of 3:1 to 4:1; the upper extremity is involved more frequently than the lower extremity; and fracture is the most common causative event. Incidence of CRPS after distal radius fracture has been reported to be between 22 and 39%.

In a double-blind, prospective, multicenter trial by Zollinger et al., 416 patients with 427 wrist fractures were studied for the effects of prophylactic vitamin C (ascorbic acid) on the risk of subsequent development of CRPS. Administration of 500 mg vitamin C daily was found to significantly decrease the incidence of CRPS in patients with distal radius fracture. The authors recommend treatment for 50 days.

The other four medications listed have all been reported for the treatment of patients diagnosed with CRPS. None have been reported to decrease the incidence of CRPS when used prophylactically.

How well did you know this?
1
Not at all
2
3
4
5
Perfectly
56
Q

A 30-year-old man is evaluated after sustaining a laceration of the left ulnar nerve at the level of the mid humerus. Exploration and direct repair of the ulnar nerve are performed immediately. Which of the following interventions is most likely to result in optimal recovery of hand function?

A) Anterior interosseus nerve transfer to the ulnar nerve
B) Dynamic splinting
C) Electrical stimulation of the ulnar nerve
D) Flexor digitorum superficialis tendon transfer to the A2 pulley
E) Wrist extensor tendon transfer to the lumbricals

A

The correct response is Option A.

High ulnar nerve injuries are plagued by poor recovery of intrinsic function even when the nerves are repaired under optimal conditions. To maximize recovery and restore intrinsic function in the hand, one of the most effective interventions is transfer of the motor branch of the anterior interosseus nerve to the motor branch of the ulnar nerve at the wrist. Recently, this has been shown to also be useful as a “babysitter” transfer, potentially retaining motor endplates in the small muscles of the hand while a primarily repaired ulnar nerve regenerates.

Dynamic splinting has played a role in radial palsies and is frequently employed to overcome joint contractures that are not fixed (with a hard endpoint). This would not assist in hand recovery in this setting.

Electrical stimulation performed by a hand therapist helps in retraining innervated muscles but will not improve function in the denervated small muscles of the hand.

Tendon transfers have been the mainstay of functional restoration when nerve repair is not possible and/or the timeframe that has elapsed is great enough to make successful reinnervation of the target muscles unlikely. Both the wrist extensor-to-lumbrical transfer and the flexor digitorum superficialis transfer have been used to correct the “clawing” associated with ulnar palsies.

How well did you know this?
1
Not at all
2
3
4
5
Perfectly
57
Q

A 45-year-old right-hand–dominant man who is a tennis player is evaluated because of a 6-month history of pain in the right lateral elbow. He has pain when lifting objects, and the pain radiates to the forearm. Physical examination shows tenderness just distal and anterior to the lateral epicondyle. Which of the following muscles is most likely affected?

A) Brachioradialis
B) Extensor carpi radialis brevis
C) Extensor carpi radialis longus
D) Extensor carpi ulnaris
E) Extensor digiti minimi

A

The correct response is Option B.

The extensor carpi radialis brevis (ECRB) origin is the primary muscle involved in lateral epicondylitis. The undersurface is avascular, making it a potential site for degeneration and partial tears. The ECRB shares a common origin with the extensor carpi ulnaris, extensor carpi radialis longus, and brachioradialis. The extensor digiti minimi also originates from the lateral epicondyle and has been involved in some cases of lateral epicondylitis, but not as commonly as the ECRB.

How well did you know this?
1
Not at all
2
3
4
5
Perfectly
58
Q

Stellate ganglion blocks help alleviate the symptoms of complex regional pain syndrome in which of the following ways?

A) Helping repair nerve injuries
B) Increasing parasympathetic tone
C) Reducing sympathetic tone
D) Relaxing the muscles of the upper extremity
E) Releasing the body’s natural endorphins

A

The correct response is Option C.

Complex regional pain syndrome (CRPS) is a long-term pain condition that is believed to result from dysfunction in the central or peripheral nervous systems. CRPS is characterized by pain, swelling, or stiffness in the affected hand or extremity. The pain may be out of proportion to the injury that triggers it.

There are two types of CRPS. In type I, there is no identifiable nerve injury. In type II, there is an identifiable nerve injury. The name of this disorder was changed from reflex sympathetic dystrophy to CRPS because not all patients have increased sympathetic tone.

Stellate ganglion blocks may be used to anesthetize the stellate ganglion, which is a cluster of sympathetic nerves at the base of the neck, in an effort to decrease the overactivity of the sympathetic nerves seen in CRPS. The sympathetic nervous system has been implicated in the pathophysiology of CRPS, and consequently, sympathetic nervous system blockade is widely used to treat CRPS.

Sympathetic nervous system dysfunction is presumed to be an essential component of the syndrome, and sympathetic blockade has been recommended as early as possible to interrupt and reverse the process. The treatment effectively cuts the vicious cycle of pain, immobilization, and decreased joint motion.

Stellate ganglion blocks do not act to increase parasympathetic tone, relax muscles, release endorphins, or repair injured nerves.

How well did you know this?
1
Not at all
2
3
4
5
Perfectly
59
Q

A 35-year-old woman who underwent surgical release of the first dorsal compartment 8 months ago is evaluated because of severe pain over the anatomical snuffbox of the right hand since the surgery. Physical examination shows focal tenderness over the region of the first dorsal compartment, a Tinel sign at the surgical scar, and numbness distal to the surgical scar. Medication includes gabapentin since the pain began, and the patient has attempted desensitization in hand therapy. Which of the following is the most appropriate next step in management?

A) Exploration silicone capping of the palmar cutaneous nerve
B) Initiation of oral pregabalin therapy
C) Iontophoresis with dexamethasone
D) Neurolysis and intramuscular placement of the superficial radial nerve
E) Phenol injection into the scar and compression glove therapy

A

The correct response is Option D.

One potential complication from surgical treatment of de Quervain disease (first dorsal compartment release) is injury to branches of the radial sensory nerve. When such injury is suspected, treatment should consist of exploration and, if possible, repair of the injured nerve branch. When this is not possible, multiple surgical options are available; transposition of the injured radial sensory nerve into the brachioradialis has been one of the commonly employed treatment strategies. Alternatives include transposition of the nerve into bone or a vein, nerve stripping, and coverage of the injured nerve with vascularized tissue.

Both gabapentin and pregabalin are useful medical interventions for nerve pain. General guidelines suggest proceeding with surgical treatment for suspected neuromas if no improvement occurs after 6 months of oral therapy.

Iontophoresis with corticosteroids has no proven efficacy for neuroma therapy. Phenol has been employed for stump neuromas and for Morton’s neuromas; however, there is no evidence to support its use in non-amputation neuroma treatment.

Silicone capping, once advocated for treatment of end-neuromas, has fallen out of favor because of poor pain relief results and displacement/migration of the caps.

How well did you know this?
1
Not at all
2
3
4
5
Perfectly
60
Q

A 43-year-old woman is evaluated for intrinsic wasting and paresthesias of the little and ring fingers. Which of the following is the most likely site of nerve entrapment?

A) Anconeus epitrochlearis
B) Arcade of Frohse
C) Lacertus fibrosis
D) Ligament of Struthers
E) Pronator teres

A

The correct response is Option A.

Nerve entrapment is caused by the anconeous epitrochlearis. The scenario describes both motor and sensory signs and symptoms attributable to ulnar nerve compression. Ulnar nerve entrapment at the elbow is the second most common nerve entrapment neuropathy in the upper limb other than carpal tunnel syndrome in young adults. The most common cause is the ligament of Osborne. The anconeus epitrochlearis muscle, which is a congenital accessory muscle, arises from the medial epicondyle of the humerus and inserts at the olecranon process of the ulna. It can be found in normal elbows with an incidence of between 4 and 34%. It has been associated with ulnar nerve compression at the elbow.

The arcade of Frohse has been associated with posterior interosseous nerve entrapment. The lacertus fibrosis, pronator teres, and ligament of Struthers have been associated with median nerve compression. Note that the arcade of Struthers may be associated with ulnar nerve compression, but this option is not provided.

How well did you know this?
1
Not at all
2
3
4
5
Perfectly
61
Q

A 36-year-old man comes to the office because of a 16-month history of diffuse pain of the posterior right shoulder. There is no history of trauma. Results of x-ray study are negative for an osseous injury. Physical examination shows atrophy isolated to the posterior scapular muscles. Motor and sensory examination of the right upper extremity shows weakness in shoulder external rotation with the arm adducted; no other abnormalities are noted. Which of the following nerves is most likely injured?

A) Axillary
B) Long thoracic
C) Spinal accessory
D) Suprascapular
E) Thoracodorsal

A

The correct response is Option D.

The patient has an isolated palsy of the suprascapular nerve, the first branch off of the upper truck (C5, C6) of the brachial plexus. Causes can include trauma, ganglion cyst (supraspinous fossa), or direct compression of the nerve as it passes under the transverse scapular ligament. Symptoms include diffuse posterior shoulder pain, atrophy of the supraspinatus and infraspinatus muscles, and weakness in shoulder external rotation.

The axillary nerve comes off of the posterior cord and innervates the deltoid muscle and teres minor muscle; palsy would primarily impair shoulder abduction. The long thoracic nerve is composed of contributions from the C5-C7 roots and innervates the serratus anterior muscle; a deficit would lead to scapular winging. The thoracodorsal nerve (C6-C8), a branch of the posterior cord, innervates the latissimus dorsi muscle; a deficit would impair shoulder extension, adduction, and internal rotation. The spinal accessory nerve innervates the trapezius and sternocleidomastoid muscles. Injury to this nerve does not affect the periscapular musculature.

How well did you know this?
1
Not at all
2
3
4
5
Perfectly
62
Q

A 44-year-old woman is evaluated because of a 6-month history of pain in her right upper chest and back, intermittent coolness in her right hand, and numbness and tingling of her right ring and little fingers. Results of the Adson test show a decreased radial pulse on the affected side, and the Roos test reproduces the patient’s symptoms on the affected side. Which of the following is the most appropriate next step?

A) Decompression of ulnar nerve at elbow
B) Noninvasive vascular study and electrodiagnostics
C) Resection of anterior and middle scalene muscles
D) Transaxillary resection of first rib
E) Observation

A

The correct response is Option B.

Three kinds of surgical procedures are employed to treat thoracic outlet compression syndrome (TOCS): transaxillary resection of the first rib, transcervical anterior and medial scalenectomies, and combined transaxillary first rib resection with immediate anterior and medial scalenectomies. This is the most complete procedure for total decompression of the thoracic outlet region. Because 70% of cases have soft-tissue involvement as the etiology of TOCS, current treatment includes transcervical anterior and middle scalenectomy in most TOCS cases.

Prior to any surgery, patients are treated conservatively with an exercise program for TOCS involving scalene stretching, first rib intercostal relaxation, nerve gliding, muscle relaxants, and pain patches for painful myofascial trigger points. Unfortunately, these conservative treatment modalities may yield only limited temporary help.

TOCS is usually classified in two groups. A neurogenic group comprises nearly 90% of all cases. This group usually has upper extremity pain, numbness, and tingling. A true vascular group comprises 10% of cases. Approximately 50% of patients still complain of coldness in the extremity. Approximately 40 to 50% of TOCS cases have concomitant peripheral nerve compression symptoms. Simple distal decompression of nerves will not usually lead to near-complete resolution of symptoms in cases of true TOCS.

There are two tissue groups that cause TOCS: soft tissue and osseous structures. The soft-tissue group includes anterior and middle scalene and their sheath, ligaments, and bands. This group comprises at least 70% of all TOCS cases because of congenital and acquired changes in the soft tissues. The osseous group comprises 30% or less of all TOCS cases and includes cervical rib, changes in the first rib, and clavicle due to injury.

Because TOCS can present with several different findings (including vascular and neurological compromise), it is advisable to work up these findings prior to committing to a treatment course. Noninvasive vascular studies and electrodiagnostics is the most reasonable first step in working up and treating these patients.

How well did you know this?
1
Not at all
2
3
4
5
Perfectly
63
Q

A 51-year-old woman is evaluated because of numbness and tingling of the dorsal and palmar aspects of the left hand, extending to the ring and little fingers, with worsening symptoms at night. Physical examination shows weakness of finger abduction in the hand. Which of the following is the most likely electrodiagnostic finding?

A) Decreased median conduction velocity from above elbow to wrist
B) Decreased ulnar conduction velocity from above elbow to wrist
C) Decreased ulnar conduction velocity from below elbow to wrist
D) Prolonged median sensory latency from wrist to digit
E) Prolonged ulnar sensory latency from wrist to digit

A

The correct response is Option B.

This patient exhibits symptoms of ulnar nerve compression at the cubital tunnel. Cubital tunnel syndrome is characterized by numbness and tingling in the ulnar nerve distribution (ulnar side of hand, involving little finger and ulnar half of ring finger) and can lead to intrinsic weakness. Compression occurs at the level of the elbow, with slowing of nerve conduction across the area of compression. Electrodiagnostic findings in ulnar nerve compression consist of decreased ulnar conduction velocity in the segment from above elbow to the wrist. Comparison of conduction velocities between above elbow to wrist and below elbow to wrist may show a difference, with nerve conduction being faster when measured from below the elbow, as the area of compression is not traversed. Conduction velocities from below elbow to wrist should not be affected. Ulnar nerve compression at the cubital tunnel can be distinguished from compression at the Guyon canal, as symptoms in the dorsal hand are not involved during nerve compression at the Guyon canal (at the level of the wrist), because the dorsal sensory branch of the ulnar nerve branches proximal to the wrist.

Prolonged median sensory latency from wrist to digit is seen in carpal tunnel syndrome.

How well did you know this?
1
Not at all
2
3
4
5
Perfectly
64
Q

A 32-year-old woman comes to the office for evaluation because of numbness of the left little finger 3 months after undergoing repair of a laceration of the left wrist sustained during an unsuccessful suicide attempt. Physical examination shows a healed laceration with a dysesthetic scar at the proximal wrist crease. A strong Tinel sign is present at the repair site. There is complete sensory loss of the little finger and no evidence of clawing. Motor function is intact. Wartenberg sign is absent. Two-point discrimination is greater than 15 mm. Which of the following is the most appropriate next step?

A) Microdissect the neuroma and identify motor fascicles with electrostimulation
B) Microdissect the neuroma and sural nerve graft fascicles
C) Resect the neuroma and direct repair with transposition
D) Resect the neuroma and repair with sural nerve grafts

A

The correct response is Option A.

The patient described has an ulnar neuroma-in-continuity with intact motor function and no sensory regeneration. Mackinnon has described an electrostimulation technique where the proximal motor fibers are identified using nerve stimulation.

Resection of the neuroma, with or without transposition, is not appropriate because it would cause damage to intact nerve fascicles. Microdissection without nerve stimulation would also cause damage to intact fascicles.

How well did you know this?
1
Not at all
2
3
4
5
Perfectly
65
Q

A 17-year-old boy is brought to the emergency department 5 hours after he sustained a stab wound to the left dorsal forearm. On physical examination, he is unable to extend the thumb and metacarpophalangeal joints. Exploration of the wound for repair of a presumed nerve injury is planned. Proper exposure of the nerve is between which of the following muscle groups?

A) Brachialis and triceps
B) Brachioradialis and extensor carpi radialis longus (ECRL)
C) Extensor carpi radialis brevis (ECRB) and ECRL
D) Extensor digitorum communis and ECRB
E) Flexor carpi radialis and pronator teres

A

The correct response is Option D.

Inability to extend the thumb and metacarpophalangeal joints generally suggests an injury to the radial nerve. Wrist extension can be preserved because of the preservation of the extensor carpi radialis longus muscle innervation. In the forearm, the radial nerve can be best approached directly between the extensor digitorum communis and the extensor carpi radialis brevis muscles. The approach between the brachialis and triceps muscles can identify the radial nerve in the upper arm. The interval between the flexor carpi radialis and pronator teres approaches the median nerve.

66
Q

A 20-year-old man comes for evaluation 9 months after sustaining a stab wound to the left proximal upper arm. He did not seek medical attention at the time of the injury. Physical examination shows that he is unable to flex the left elbow with the forearm supinated. He is insensate to the lateral aspect of the upper arm and forearm. In addition to the ulnar nerve transfer to the biceps, which of the following nerve transfers is most appropriate to address this patient’s motor deficit?

A) Median nerve to brachialis
B) Musculocutaneous nerve to brachioradialis
C) Posterior interosseous nerve to triceps
D) Radial nerve to pronator teres
E) Ulnar nerve to flexor carpi radialis

A

The correct response is Option A.

The patient described has sustained a laceration of the musculocutaneous nerve. With this injury, the patient would be able to flex the elbow with the forearm in a pronated position using the brachioradialis, which is innervated by the radial nerve.

Due to the proximal level and amount of time that has passed since the injury, repair with grafting of the musculocutaneous nerve may not be advisable as the axons may not reach the motor end plates of the biceps and brachialis muscles before degeneration.

Use of fascicles from the median nerve, ulnar nerve, and both nerves has been described to restore elbow flexion. Fascicles are transferred distally in the upper arm directly to the nerve branch to the brachialis and/or biceps muscle. Distal coaptation allows donor axons to reach the target muscles more rapidly, and is more appropriate for this patient whose status is nearly 1 year post injury. A fascicle from the ulnar nerve was transferred to the nerve to the biceps (blue dot) and a fascicle from the median nerve was transferred to the nerve to the brachialis (green dot).

The musculocutaneous nerve is injured and cannot be used as a donor. The posterior interosseous nerve does not branch off the radial nerve until distal to the elbow; it would not be able to reach proximal enough to coapt to the nerve to the brachialis or biceps. The pronator teres does not flex the elbow; the median nerve is not injured. The flexor carpi radialis does not flex the elbow; the median nerve is not injured.

67
Q

An 18-year-old man comes to the office because he has “drooping” of the right shoulder and inability to abduct it beyond 30 degrees. He underwent exploration and vascular repair 4 months ago after sustaining a stab wound to the right side of the neck. Photographs are shown. Which of the following surgical transfer techniques is most likely to restore deltoid function in this patient?

A) C5 to C6 nerve root
B) Intercostal to musculocutaneous nerve
C) Partial ulnar nerve to musculocutaneous nerve
D) Radial nerve fascicle to axillary nerve
E) Suprascapular to axillary nerve

A

The correct response is Option D.

Of the nerve transfers listed, only the transfer of a triceps branch to the axillary nerve will restore innervation to the deltoid muscle and provide shoulder abduction. Although debate remains over whether nerve graft repair (if possible) or nerve transfer provides better outcomes for shoulder abduction, both techniques are frequently employed in the setting of upper trunk brachial plexus injury.

The location of the injury (based on scar and mechanism) makes the injury likely distal to the nerve roots; in addition, root to root transfers are not typically part of the brachial plexus reconstruction ladder.

The suprascapular nerve is often the recipient nerve for partial transfers from cranial nerve XI (spinal accessory nerve) in order to provide shoulder stability. In this pattern of injury and based on the photos demonstrating atrophy of the supraspinatus and infraspinatus, the suprascapular nerve could not act as a donor nerve.

Intercostal nerve and partial ulnar nerve transfers to the musculocutaneous nerve are both frequently used to restore elbow flexion in the setting of upper trunk injuries.

68
Q

In relation to the pronator teres muscle, which of the following is the most likely location of the median nerve in the proximal third of the forearm?

A) Between the superficial (humeral) and deep (ulnar) heads of the pronator teres muscle
B) Deep to the superficial (humeral) and deep (ulnar) heads of the pronator teres muscle
C) Superficial to the superficial (humeral) and deep (ulnar) heads of the pronator teres muscle
D) Through the deep (ulnar) head of the pronator teres muscle
E) Through the superficial (humeral) head of the pronator teres muscle

A

The correct response is Option A.

The course of the median nerve is relatively consistent. Just proximal to the elbow, at the medial epicondyle, there is a constant relationship of the median nerve, brachial artery, and the biceps tendon. From medial to lateral, the mnemonic, MAT, describes the relationship (Median nerve, brachial Artery, and biceps Tendon). In the cubital fossa, the nerve dives deep to the lacertus fibrosus, lying anterior to the brachialis muscle and medial to the brachial artery. The nerve enters the forearm between the superficial (humeral) and deep (ulnar) heads of the pronator teres muscle. As the nerve passes through the muscle bellies, it crosses the ulnar artery anteriorly, from medial to lateral, separated from the artery by the deep head of the pronator teres. After emerging from the pronator teres, the median nerve passes deep to an arch created by the two heads of the flexor digitorum superficialis. The nerve continues distally in the forearm between the flexor digitorum superficialis and flexor digitorum profundus. The nerve usually becomes superficial approximately 5 cm proximal to the wrist, emerging between the flexor digitorum superficialis and flexor carpi radialis, dorsal and slightly radial to the palmaris longus tendon.

69
Q

A 24-year-old woman comes to the office because of severe thenar atrophy 3 years after failed repair of a low median nerve laceration. Tendon transfer to restore thumb opposition is planned. Optimal transfer would restore which of the following thumb functions?

A) Palmar abduction, extension, supination
B) Palmar abduction, flexion, pronation
C) Palmar abduction, flexion, supination
D) Radial abduction, extension, pronation
E) Radial abduction, flexion, supination

A

The correct response is Option B.

Injury to the median nerve, either by laceration or compression, results in thenar atrophy and loss of thumb opposition. Tendon transfer is the only reliable technique to restore thumb function in the face of severe, long-standing atrophy. Thumb opposition is a composite movement comprised of palmar abduction, flexion, and pronation. Opposition positions the thumb for grasp, but is not synonymous with it. There are numerous tendon transfer procedures described to improve thumb opposition (e.g., palmaris longus, abductor digiti minimi, flexor digitorum superficialis), but the most effective improve each of the three components. Thumb extension, supination, and radial abduction (in the plane of the hand) are not movements involved in opposition.

70
Q

Which of the following is the earliest one might expect to find electromyographic changes after suspected median nerve damage during carpal tunnel release?

A) 1 Week
B) 3 Weeks
C) 5 Weeks
D) 7 Weeks

A

The correct response is Option B.

Patients with nerve injuries can be evaluated by nerve conduction velocities and electromyography (EMG). Abnormal conduction velocities are associated with decreased amplitude, decreased velocity, and increased latency. Sensory nerve latency above 3.5 ms and/or motor nerve latency above 4.5 ms are considered abnormal. Muscle changes assessed by EMG are typically altered later (2 to 3 weeks after injury) in the clinical course and consist of the presence of fibrillation potentials and decreased motor unit potential recruitment. Though the sensitivity of these studies may be somewhat low, at around 66% when using conduction velocity and latency, the specificity has been reproducibly near 95%.

71
Q

An otherwise healthy 58-year-old man comes to the office because of numbness of all fingers of the left hand. He says he first noticed symptoms after a cross-country drive 6 weeks ago. On physical examination, the thumb, index, and long fingers show sensitivity to the 2.83 Semmes-Weinstein monofilament. The ring and little fingers show sensitivity to the 3.22 monofilament. The little finger is held in an abducted position. Abductor pollicis brevis muscle strength is normal. First dorsal interosseous muscle strength is diminished. Which of the following anatomical structures is the most likely cause of these findings?

A) Arcade of Frohse
B) Lacertus fibrosus
C) Ligament of Struthers
D) Osborne ligament
E) Transverse carpal ligament

A

The correct response is Option D.

This scenario depicts a patient with ulnar nerve compression. Often patients who present with compression neuropathies give a history of numbness of all fingers; however, careful physical examination will show sensory abnormalities only in the anatomical location of the compression. The patient has weakness of the ulnar nerve innervated intrinsic muscles, the first dorsal interosseous muscle, but retains strength in the abductor pollicis brevis muscle. The diagnosis of ulnar nerve compression is suggested. The most common sight of ulnar nerve compression is at the elbow. The anatomical causes of all the nerve compression at the elbow are the arcade of Struthers, the medial intramuscular septum, the bony cubital tunnel, Osborne ligament, an anconeus epitrochlearis muscle, and the origin of the flexor carpi ulnaris muscle. The ligament of Struthers, lacertus fibrosus and the transverse carpal ligament are anatomical sites of compression of the median nerve. The arcade of Frohse is a site of compression of the radial nerve.

72
Q

A 48-year-old woman comes to the office because of burning pain and stiffness in the right hand 6 weeks after treatment of a distal radius fracture. She says she has had difficulty sleeping and that she has discomfort despite taking narcotics. Physical examination shows a shiny appearance of the right hand, decreased range of motion of the fingers, and hypersensitivity to light touch. X-ray studies show good alignment of the fracture. Which of the following tests is the most appropriate to evaluate this patient’s condition?

A) Bone scan
B) CT scan
C) Digital subtraction angiography
D) Lymphoscintigraphy
E) Ultrasonography

A

The correct response is Option A.

This patient exhibits symptoms of complex regional pain syndrome (CRPS). The persistence of physiological changes after surgery or injury can lead to debilitating consequences. This condition is characterized by persistent pain, cold intolerance, autonomic dysfunction, and trophic changes. Patients may show swelling, stiffness, difficulty sleeping, and persistent pain out of proportion to the normal postoperative course that may be relieved incompletely by narcotics.

CRPS is a clinical diagnosis without a single definitive test, and is divided into two types: type I, which occurs without identifiable nerve involvement (also known as reflex sympathetic dystrophy); and type II, which has identifiable nerve involvement (causalgia). It is more common in people who smoke and in women. Pain in CRPS can be either sympathetically mediated or sympathetically independent

The diagnosis of CRPS involves history, physical examination, and diagnostic testing. Although no specific test is pathognomonic, triple-phase bone scans are helpful in adding credence to the diagnosis. First- and second-phase bone scans may show asymmetric flow and autonomic dysfunction, while the third phase demonstrates increased periarticular uptake in multiple joints of the affected extremity.

A variety of treatment modalities have been employed in addressing CRPS. These range from therapy modalities such as range of motion, stress loading, and desensitization to pharmacologic interventions with anticonvulsants or antidepressants. Stellate ganglion blocks or autonomic nerve blocks may be helpful in sympathetically mediated pain, and nerve stimulation (either transcutaneous or at the spinal cord level) can also be employed. Often, multiple modalities are used concurrently and in sequence. Peripheral nerve decompression may be helpful in resolving symptoms related to CRPS type II.

CT scans can be used to assess bony alignment in fractures and are helpful in the evaluation of articular anatomy.

Digital subtraction angiography is useful for evaluation of vasculature and circulation

Lymphoscintigraphy is used to analyze lymphatic drainage in cases of lymphedema.

Ultrasonography can be performed to assess venous outflow and look for deep venous thrombosis.

73
Q

A 35-year-old man comes to the emergency department with a humerus fracture. On examination, he is unable to extend his wrist, fingers, and thumb. Which of the following nerves is most likely injured?

A) Axillary
B) Median
C) Musculocutaneous
D) Radial
E) Ulnar

A

The correct response is Option D.

This patient has a radial nerve injury, which can occur with humerus fractures. The radial nerve innervates the wrist extensors, extensor carpi radialis longus (ECRL) and extensor carpi radialis brevis (ECRB); the thumb extensors, extensor pollicis longus (EPL) and extensor pollicis brevis (EPB); and the finger extensors, extensor digitorum communis (EDC). These radial nerve injuries are usually managed with a period of observation and therapy until a potential neurapraxia resolves. Early evidence of muscle reinnervation would be evident with improved function of the ECRL, followed by the ECRB, then the finger and thumb extensors. If the patient does not regain any function by 3 to 6 months, exploration and nerve repair or tendon transfers to restore lost function can be considered. The standard tendon transfers considered for radial nerve function loss include the pronator teres to the ECRB (wrist extension), flexor carpi ulnaris to the EDC (finger extension), and the palmaris longus to the EPL (for thumb extension). A median nerve deficit would result in loss of flexion of wrist, fingers, thumb, and loss of palmar sensation and not typical after humerus fractures. An ulnar nerve injury could present with loss of hand intrinsic function and loss of sensation of the small finger. The musculocutaneous nerve innervates the biceps and would result in loss of elbow flexion. The axillary nerve is not injured with humerus fractures.

74
Q

A 40-year-old, right-hand–dominant man comes to the office because of minimal sensation in the tips of the digits of the right hand and severe pain in the volar aspect of the right wrist. He sustained lacerations of the median and ulnar nerves, radial and ulnar arteries, and all volar flexor tendons 11 months ago when he punched a window. Each of the structures was repaired primarily within hours of injury. On examination, the hand is well perfused. Percussion at the location of the dot in the photograph shown produces severe, painful paresthesia, which radiate distally. Sharp touch sensation is minimally present at the tips of the thumb, index, and long fingers and the radial side of the ring finger; light touch sensation is absent. Which of the following is the most appropriate next step in treatment?

A) Administration of tacrolimus
B) Excision repair with a sural graftbr> C) Neurolysis and conduit wrap
D) Reassessment in 3 months
E) Transfer of the extensor indicis proprius

A

The correct response is Option B.

The patient has a neuroma of the median nerve after laceration and repair. At 11 months out from injury, he would be expected to have improving light touch at the fingertips. The combination of this and the severe pain with percussion of the volar wrist indicates that very few axons have crossed the repair site. Observing the patient for additional time is unlikely to yield improved recovery. At surgical exploration, the patient had a large neuroma at the repair site. A photograph is shown.

Tacrolimus is an immune modulating agent commonly used in solid organ transplants. It has been investigated in animals and clinical trials in humans and shown some effectiveness in improving nerve regeneration across the repair. It is not currently used in humans outside of clinical trials. Also, in all trials of tacrolimus use, the medication was started at the time of repair, not 11 months later.

Nerve conduits have been used to repair short-gap nerve injuries. Although there are reports of successful nerve regeneration of gaps greater than the original indication of 1.5 cm, a gap of 6 cm is well beyond the limits of what a conduit can bridge.

Transfer of the extensor indicis proprius tendon would restore thumb opposition and use a tendon not involved in the original injury. However, this would not address the patient’s principal complaint of pain and lack of sensation.

75
Q

A 30-year-old man is evaluated because he is unable to abduct or externally rotate his shoulder or flex his elbow 4 months after he was involved in a high-speed motor vehicle collision. Physical examination shows numbness of the lateral upper arm and forearm. Which of the following nerve transfers is most appropriate to restore external rotation of the shoulder?

A) C7 ipsilateral root to anterior division of upper trunk
B) Medial pectoral nerve to medial cord
C) Phrenic nerve to long thoracic nerve
D) Radial nerve to axillary nerve
E) Spinal accessory nerve to suprascapular nerve

A

The correct response is Option E.

Examination findings indicate a C5-6 avulsion or very proximal upper trunk injury. External rotation of the shoulder is provided by the supraspinatus and infraspinatus muscles. Multiple techniques for harvest of the distal spinal accessory nerve to transfer to the suprascapular nerve have been described. This transfer would restore external rotation.

Transfer of the radial nerve to the axillary nerve will provide deltoid and possibly teres major innervation. This will improve shoulder function overall but will not restore external rotation. The phrenic nerve is commonly used to provide donor motor axons, but the long thoracic nerve goes to the serratus anterior.

Contralateral C7 transfer can be used to innervate the upper trunk. Ipsilateral C7 would not be used because the C5 and C6 roots were destroyed in the injury. Also, coaptation to the anterior division of the upper trunk would be distal to the origin of the suprascapular nerve.

The medial pectoral nerve can also be used for donor motor axons. It is limited by its relatively short reach. Coaptation to the medial cord of the brachial plexus would innervate the ulnar nerve, which is not injured in this patient.

76
Q

A 53-year-old man is evaluated because of a 5-month history of numbness and tingling of the right ring and little fingers. He says that symptoms are worse at night and when his elbow is flexed while holding a phone to his ear. Physical examination shows weakness of abduction of the fingers of the right hand. Which of the following structures is the most likely cause of this patient’s symptoms?

A ) Arcade of Frohse
B ) Flexor digitorum superficialis
C ) Ligament of Struthers
D ) Osborne band
E ) Transverse carpal ligament

A

The correct response is Option D.

The structure most likely to be responsible for the condition described is Osborne band.

The patient described exhibits symptoms of ulnar nerve compression at the cubital tunnel. Cubital tunnel syndrome is characterized by numbness and tingling of the ulnar nerve distribution (ulnar side of hand, involving little finger and ulnar half of ring finger) and can lead to intrinsic weakness. The ulnar nerve passes posterior to the medial epicondyle and travels between the medial epicondyle and the olecranon at the level of the elbow. In the region of the cubital tunnel, a set of thick fascial fibers known as Osborne band or ligament forms the roof of the tunnel and can compress the ulnar nerve. A number of additional sites can contribute to ulnar nerve compression, including the arcade of Struthers, the medial intermuscular septum, anconeus epitrochlearis, and the fascia of the flexor carpi ulnaris or flexor pronator musculature.

The arcade of Frohse is a fibrous arch that comes from the lateral epicondyle, which can compress the radial nerve.

The flexor digitorum superficialis can cause compression of the median nerve in the forearm.

The ligament of Struthers is an anatomic variant which arises from the supracondylar process of the humerus. It can cause proximal median nerve compression.

The transverse carpal ligament is responsible for median nerve compression at the carpal tunnel.

77
Q

A 50-year-old woman has significant tenderness of the residual tip of the index finger 8 weeks after undergoing amputation of the fingertip. Physical examination shows significant stump tenderness. Which of the following therapy modalities is most appropriate for desensitization of the amputation stump?

A ) Functional electric stimulation
B ) Immobilization
C ) Kinesiology tape
D ) Semmes-Weinstein monofilaments
E ) Vibration

A

The correct response is Option E.

Vibration is a modality that may be used in therapy to desensitize an amputation stump neuroma. The treatment consists of vibratory stimulation applied to the periphery of the sensitive area and then gradually moving toward the center. Some additional methods used in therapy to treat neuroma pain include desensitization, massage, and transcutaneous nerve stimulation. Functional electric stimulation is similarly not a therapeutic modality. Kinesiology taping is used for edema control and comfort. It would not be used on an amputated stump. Semmes-Weinstein monofilaments are used to quantify sensation to fine touch and are not used for desensitization.

78
Q

A 23-year-old man is brought to the emergency department after being stabbed with a knife in a fight. The patient appears alert and in no distress. Physical examination shows a 2-cm puncture wound just above the midclavicular line. Decreased strength with wrist flexion, weakness of index, long, ring, and little finger flexion at the metacarpophalangeal (MCP) and proximal interphalangeal (PIP) joints, and difficulty with palmar abduction against resistance are noted. Finger, thumb, and wrist extension is intact. Elbow flexion and extension are normal. Sharp injury to which of the following structures is most likely in this patient?

A ) Lateral cord
B ) Medial cord
C ) Median nerve
D ) Middle trunk
E ) Ulnar nerve

A

The correct response is Option B.

The patient described demonstrates median and ulnar nerve weakness after a sharp injury to the brachial plexus. The radial nerve function appears to be preserved.

The medial cord gives fibers to the medial pectoral nerve, the medial brachial and antebrachial cutaneous nerves, the median nerve, and the ulnar nerve. Therefore, this is the appropriate response.

The lateral cord is not appropriate because although that structure does give fibers to the median nerve, it does not branch to the ulnar nerve. The lateral cord terminates in the lateral pectoral nerve, musculocutaneous nerve, and the median nerve. Also of note, as the patient has intact elbow flexion, the musculocutaneous nerve is presumably intact in the scenario described.

The median nerve in isolation is not appropriate as well for similar reasons. This patient has median and ulnar nerve deficits.

The middle trunk contains fibers that go to the radial and median nerves, but not the ulnar. This is not appropriate, therefore, as well.

The posterior cord gives off the radial nerve, axillary nerve, subscapular nerves, and the thoracodorsal. It does not contribute to the median or ulnar nerve.

At the midclavicular line, the ulnar nerve has not yet formed.

79
Q

A 25-year-old woman comes for evaluation because she has difficulty gripping things with her dominant right hand. She sustained a stab wound to the left forearm 6 months ago. Physical examination shows a Tinel sign present over the ulnar aspect of the mid volar forearm with a well-healed laceration. A neuroma in-continuity is identified intraoperatively. The neuroma is resected, and a 3-cm nerve gap remains. Which of the following is the most appropriate management?

A ) Bridge the gap with a synthetic nerve conduit
B ) Dissect the nerve both proximally and distally with an end-to-end neurorrhaphy
C ) Proceed to tendon transfers to assist with grip
D ) Reconstruct the nerve defect with a lateral antebrachial cutaneous nerve graft
E ) Sural nerve cable grafting

A

The correct response is Option E.

The sural nerve is a common source of nerve graft material. It is formed from the medial cutaneous sural nerve that originates from the tibial nerve.

Synthetic nerve conduit repair of nerves has not proven effective in gaps larger than 2 cm and is less effective in motor nerve defects.

A tension-free repair is the goal for the nerve anastomoses. When an acute laceration occurs, primary end-to-end neurorrhaphy is indicated. When the treatment is delayed and a neuroma exists, the type of repair is dictated by the size of the nerve gap. Nerve grafting is indicated to bridge a defect when more than 10% elongation of the nerve would be necessary to bridge the gap.

Tendon transfers would not be indicated at this point.

The lateral antebrachial cutaneous nerve is the distal continuation of the musculocutaneous nerve. Given the size of the defect and the need for at least two cable grafts, this nerve would not be appropriate.

80
Q

An 8-year-old girl is brought to the office for evaluation because her right forearm is severely contracted at the wrist and fingers. Upon questioning, the patient’s parents note that she was treated 1 year ago for a humerus fracture. Physical examination shows a pronated forearm, flexed wrist, hyperextended metacarpophalangeal (MCP) joints, and flexed proximal (PIP) and distal interphalangeal (DIP) joints. She can make a full active fist. Which of the following is the most appropriate management?

A ) Exploration at the level of the humerus fracture with neurolysis and/or repair of the radial nerve
B ) Exploration of the forearm, neurolysis of the median and ulnar nerves, and muscle slide technique performed
C ) Exploration of the upper arm with median and ulnar nerve neurolysis
D ) Flexor carpi radialis to extensor carpi radialis brevis tendon transfer

A

The correct response is Option B.

Volkmann described the sequelae of compartment syndrome following a supracondylar fracture of the humerus in a child with the development of a severely contracted and functionless forearm. The forearm is typically fixed in pronation, the wrist is flexed, and the hand is postured in the ?claw? position with the MCP joints hyperextended and the PIP and DIP joints flexed. The hand is usually insensate. Treatment depends on the severity of the deformity. Moderate contractures are treated with exploration and release of both the median and ulnar nerves and a tendon lengthening procedure (muscle slide). This condition is not due to a radial nerve injury; therefore, exploration of the fracture site and neurolysis of the radial nerve are not indicated. Tendon transfer is not appropriate because it does not address the nerve compression. Revascularization with a bypass graft is not needed because it is no longer ischemic.

81
Q

A 36-year-old man is brought to the emergency department 2 hours after being stabbed in the hand with a knife. Physical examination shows a wound over the dorsum of the first web space. Exploration with administration of a local anesthetic agent is planned. Which of the following is the most appropriate landmark for injection of the local anesthetic?

A ) A1 pulley
B ) Flexor carpi radialis
C ) Medial epicondyle
D ) Pisiform
E ) Radial styloid

A

The correct response is Option E.

The most appropriate location for injection of local anesthesia is the radial styloid.

Wrist blocks are useful for providing anesthesia during hand surgery. Sensation is blocked, while preserving movement of the extrinsic musculature. The sensory supply to the hand is provided by the median, ulnar, and radial nerves. The median nerve innervates the volar radial aspect of the hand, including thumb, index, long, and the radial half of the ring finger. The median nerve can be blocked as it travels between the flexor carpi radialis and palmaris longus tendons. Sensation to the thenar eminence is provided by the palmar cutaneous branch, which divides about 5 to 7 cm proximal to the wrist crease. The ulnar nerve innervates the ulnar aspect of the hand, including the ulnar aspect of the ring finger and the little finger. At the level of the wrist, the ulnar nerve lies radial to the flexor carpi ulnaris tendon, and ulnar to the ulnar artery. The dorsal cutaneous branch of the ulnar nerve branches proximal to the wrist crease and travels dorsally to innervate the ulnar aspect of the dorsum of the hand. It can be blocked as it crosses the region of the ulnar styloid. The radial nerve provides sensation to the dorsal radial aspect of the hand, including the first web space. The superficial radial nerve divides into several branches in the region of the radial styloid and can be blocked by subcutaneous infiltration over the radial styloid proximal to the anatomical snuffbox. This patient has a wound in the dorsal first web space, which is innervated by the radial nerve; hence, injection at the radial styloid level will provide anesthesia to the area.

A digital block can be performed with local injection in the region of the A1 pulley but would result in anesthesia too distal to the area of interest in this case.

The median nerve can be anesthetized by injecting local anesthetic just ulnar to the flexor carpi radialis tendon.

Injection just posterior to the medial epicondyle would result in blockade of the ulnar nerve at the elbow.

The ulnar nerve can be blocked at the level of the wrist by injecting radial to the flexor carpi ulnaris tendon and pisiform bone.

82
Q

Consultation is requested for a 7-year-old girl because of intravenous infiltration of a chemotherapeutic agent in the dorsal forearm. Physical examination shows firmness and swelling of the forearm and pain on passive flexion of the wrist. Which of the following is the most appropriate initial management?

A) Administration of an antidote
B) Doppler sonography of the forearm
C) Liposuction and saline flush of the affected area
D) Measurement of compartment pressures
E) Surgical excision and grafting

A

The correct response is Option D.

Extravasation usually remains localized, yet some patients develop necrotic problem wounds. Often initially underestimated, the extent of injury can declare itself widely with time. Compartment syndrome in an extremity extravasation should be initially ruled out either by clinical assessment or direct measurement of compartment pressures. Tissue loss can include skin, muscle, tendon, nerve, vasculature, and/or joint.

Given the variable amount of soft-tissue involvement, early conservative therapy is recommended. Immediate discontinuation of the infusion at the affected site is paramount and should not be overlooked. Aspiration, liposuction, wound excision, debridement, grafts, flaps, and antidote administration have all been described in the management of extravasation injury.

83
Q

A 24-year-old man comes to the office because of numbness and difficulty moving his ring and little fingers 5 months after cutting his upper arm on broken glass. Current physical examination shows inability to abduct and adduct the ring and little fingers. Sensation to light touch is diminished. Following exploration and resection of a painful, traumatic neuroma, there is a 5-cm gap in the ulnar nerve proximal to the elbow. Which of the following is the most appropriate management to restore intrinsic muscle function?

A) Cadaveric nerve allografting
B) Nerve transfer
C) Sural nerve grafting
D) Use of nerve conduit
E) Vascularized nerve grafting

A

The correct response is Option B.

The most appropriate management for restoration of intrinsic muscle function is nerve transfer.

In nerve injuries resulting in complete transection of the nerve, wallerian degeneration occurs at the site of transection, and Schwann cells in the distal nerve segment undergo apoptosis. With prolonged denervation, decreased regenerative ability with limitation in motor recovery is noted. Optimal functional recovery is dependent upon adequate reinnervation of the motor end plates and target muscles by regenerating motor axons. Over time, loss of target motor end plates via degeneration and fibrosis and replacement of muscle fibers by fat cells occur.

Nerve regeneration occurs at a rate of approximately 1 mm daily or 1 inch monthly. In a high injury to the ulnar nerve, the distance from the proximal motor axons to the intrinsic musculature precludes timely reinnervation, and intrinsic recovery is generally poor. Reinnervation of the muscle ideally should be completed within 12 to 18 months following injury to allow for recovery.

In the patient who has had the delayed symptoms and high ulnar nerve injury described, the time to recovery of intrinsic function will be greater than 2 years if the injury is reconstructed directly. This estimate is based on the elapsed time and distance to the target muscles.

Nerve transfer involves the use of a noncritical or expendable donor motor nerve to reinnervate a missing function. The selection of an available motor nerve donor that is closer to the target muscle can decrease the time needed for reinnervation of the muscle and help to ensure recovery before irreversible changes occur. In the scenario described, the distal portion of the anterior interosseous nerve can be used as a donor nerve to reinnervate the ulnar motor branch. Transfer of the distal anterior interosseous nerve to the motor branch of the ulnar nerve will provide motor neurons in a more distal location to reinnervate the intrinsic muscles in the desired time frame.

Nerve grafting is the most appropriate management to bridge a nerve gap when direct repair is not possible. This would be indicated if the circumstances dictated that muscle reinnervation could occur in an adequate or timely fashion, such as in a more recent injury or in a nerve gap that is closer to the target muscles. Nerve grafting may be performed for sensory recovery, but motor reinnervation is unlikely to occur in the scenario described. The sural nerve provides a good source of nerve autograft that is long (up to 40 cm) and of reasonable diameter (2 to 3 mm), with minimal donor site morbidity.

A variety of artificial nerve conduits have been developed to avoid the need to harvest nerve grafts. Nerve conduits of polyglycolic acid and collagen have been developed to bridge nerve gaps; however, recovery is not as effective as compared with autogenous nerve grafts. Typically, conduits are used for sensory nerves in noncritical areas. Gaps of up to 2 to 3 cm can be bridged.

There has been recent interest in processed preserved nerve allografts which are obtained from cadaveric sources. Studies suggest that allograft nerves may regenerate motor neurons better than nerve conduits, but autologous nerve grafting remains the gold standard. Typically, nerve allografts are more useful for short segmental nerve gaps.

A vascularized nerve graft allows transfer of the blood supply along with the nerve. This decreases the need for revascularization from the surrounding tissues and may be useful for grafting long nerve gaps with badly scarred or irradiated beds.

84
Q

A 13-year-old boy is brought to the office because he has difficulty opening his hand and extending his fingers. History includes release of the forearm compartments to treat a pulseless hand following a supracondylar humerus fracture 2 years ago. On physical examination, passive extension of the fingers is restricted when the wrist is fully extended; it improves with full wrist flexion. Which of the following muscles is the most likely cause of the limitation described?

A) Flexor carpi radialis
B) Flexor carpi ulnaris
C) Flexor digitorum profundus
D) Flexor digitorum superficialis
E) Lumbricals

A

The correct response is Option C.

The most likely cause of the restricted finger extension described is fibrosis of the flexor digitorum profundus muscle. The patient exhibits Volkmann ischemic contracture as a complication of late treatment (over 24 hours from the time of initial ischemia) of arterial compromise associated with the fracture. The muscle groups at the greatest risk during these ischemic episodes are within the deep flexor compartment of the forearm. This risk occurs because the arterial supply is relatively distant from the usual site of occlusion and because this compartment is relatively less distensible. In the scenario described, the flexor digitorum profundus and flexor pollicis longus are at the greatest risk.

Superficial muscle groups such as the flexor carpi radialis, flexor carpi ulnaris, and the flexor digitorum superficialis typically recover some function and do not lead to contractures in the forearm. Likewise, the small muscles of the hand, such as the lumbricals, tend to be less severely injured than the deep compartment of the forearm.

85
Q

A 24-year-old man comes to the office because he says the ring and little fingers of his right hand “catch” when he puts his hand in his pocket and that he “pokes” himself in the eye when washing his face. History includes repair of a complete transection of the right ulnar nerve at the wrist 1 year ago. On physical examination, he is unable to extend the interphalangeal joints of the ring and little fingers when the metacarpophalangeal joints are flexed. Photographs are shown. Which of the following tendon transfers is the most appropriate management?

A) Extensor indicis proprius (EIP) to adductor
B) EIP to extensor digiti minimi
C) EIP to first dorsal interosseous
D) Flexor digitorum superficialis (FDS) of the little finger to A2 pulley
E) FDS of the little finger to lateral band

A

The correct response is Option E.

Of the tendon transfer choices offered, only the FDS transfer to the lateral band (of both the ring and little fingers) will correct the loss of interphalangeal joint extension described, thereby diminishing the tendency for the flexed/abducted finger to catch on pocket edges.

The clinical scenario and photographs demonstrate failure of the intrinsic muscle function to return following a low ulnar nerve repair. The deformities demonstrated include ulnar clawing of the little finger primarily, abduction of the little finger (Wartenberg sign), hyperflexion of the interphalangeal joint of the thumb, and atrophy of the intrinsics (especially notable in the hypothenar eminence on the lateral view). Of these deformities, the patient is bothered primarily by the little finger deformity.

Correction of clawing can be achieved actively or passively. Patients who can extend the interphalangeal joints while hyperextension of the metacarpophalangeal joints is blocked (Bouvier test) can achieve correction of clawing with active or passive transfers. Active transfers attempt to re-create the normal function of the intrinsics by directing pull through the lateral bands. Passive transfers re-create the intrinsic function of metacarpophalangeal joint flexion (similar to externally blocking hyperextension) but do not extend the interphalangeal joints.

EIP transfers are useful for correction of the lateral pinch functions of the intrinsic minus hand. An EIP transfer to the adductor tendon re-creates the thumb component of lateral pinch, while the EIP transfer to the first dorsal interosseous tendon would improve the index function in pinch.

EIP transfer to the extensor digiti minimi is one method used to reduce hyperabduction of the little finger. This would not correct the flexion deformity at the level of the proximal interphalangeal joint.

FDS transfer to the A2 pulley provides a passive transfer, which, based on the patient’s inability to extend the interphalangeal joints during the Bouvier test, would not correct the deformity.

86
Q

A 25-year-old right-hand dominant man is brought to the emergency department after sustaining a stab wound to the right arm in a bar fight. Physical examination shows a 2 × 1-cm laceration over the antecubital fossa. He is unable to flex the interphalangeal joint of the thumb and the proximal interphalangeal joint of the index finger. Which of the following nerves is most likely injured?

A) Lateral antebrachial cutaneous
B) Median
C) Musculocutaneous
D) Radial
E) Ulnar

A

The correct response is Option B.

Median nerve palsy is marked by the inability to oppose the thumb or flex the thumb at the interphalangeal joint. The inability to flex the index finger at the proximal interphalangeal joint is also noted. The lateral antebrachial cutaneous nerve provides sensory innervation to the lateral aspect of the arm. The median antebrachial cutaneous nerve innervates the skin of the anterior and middle surfaces of the forearm to the level of the wrist. This nerve does not innervate any muscles. Radial nerve palsy is marked by the inability to extend the fingers, thumb, and wrist. Patients with radial nerve palsies have difficulty grasping objects. The results of tendon transfers to restore function in patients with radial nerve palsies are among the best and most predictable outcomes. Ulnar nerve palsy symptoms include a ?claw? deformity, with flexion deformities of the ring and little fingers. In later stages, profound muscle wasting of the both hypothenar eminence and the first web space is seen.

87
Q

A 55-year-old man with bilateral carpal tunnel syndrome comes to the outpatient surgical unit for elective surgical intervention of the dominant right hand. He will be the tenth procedure of the day for the surgeon performing the operation. The surgeon favors an open technique; he has performed 150 carpal tunnel operations since finishing his hand fellowship 3 years ago. Which of the following is most likely to increase the risk of wrong-site surgery?

A) The elective nature of the procedure
B) Only one surgeon is involved in the operation
C) The procedure will be performed using an open technique
D) The surgeon has a high volume of cases scheduled for the same day
E) The surgeon has only been practicing independently for 3 years

A

The correct response is Option D.

The term ?wrong-site surgery? includes surgery on the wrong organ or extremity, the wrong patient, or the wrong vertebral level. This error can result in disastrous outcomes for the patient, as well as the institution and professionals involved. Traditionally, these errors have been considered ?sentinel events? that require a root cause analysis to define the hazards that triggered the event. Fortunately, wrong-site surgery is rare; however, the true incidence is unknown and appears to be increasing. Attempts to quantify the true incidence of wrong-site surgery are limited by underreporting to The Joint Commission and the often covert nature of these events caused by liability concerns. One review has estimated an incidence rate of one wrong-site surgery per 100,000 operations. This rate was 4 times higher among hand surgeons, however, with an estimated one in five hand surgeons predicted to perform a wrong-site surgery in their career.

A Joint Commission review of a series of sentinel events identified a number of factors contributing to the increased risk of wrong-site surgery, such as emergency cases; unusual physical characteristics, including morbid obesity or physical deformity; unusual time pressures to start or complete the procedure; unusual equipment or setup in the operating room; multiple surgeons involved in the case; and multiple procedures being performed during a single surgical visit. A large series of wrong-site hand surgeries showed an increased rate of wrong-site surgery with increasing surgeon age and experience, and a direct correlation with increasing surgical case volumes.

While more than one factor was often identified after a root cause analysis, the majority involved a breakdown in communication between the surgical team and the patient and his or her family.

While it seems that something as simple as operating on the correct side would be intuitive, in reality, the enormous pressures of time and patient volume in the current health care environment set up surgeons for an eventual system failure. Review of the factors noted above demonstrates that the major component in preventing wrong-site surgery is effective communication between surgeon and patient, and the surgeon bears the ultimate responsibility of assuring that this level of communication exists.

Current Joint Commission requirements include a preoperative verification, site marking, and a ?time out? in the operating room. One recent study of malpractice claims after wrong-site surgery showed that two thirds of wrong-site surgeries could have been prevented by an effective site-verification protocol. Recommendations for effective site verification include the following:

Site marking by the surgeon with initials or “yes.”

Preoperative verification process: verification of patient identity, site, side, and procedure confirmed by two members of the health care team, including the surgeon. The informed consent should be compared with the operating room schedule. A “time out” before incision will then provide a final confirmation of the appropriate procedure and site. A specific, detailed protocol to define this process that involves clear recommendations for specific behaviors is more likely to be followed than general or ambiguous recommendations.

Inconsistencies: any inconsistencies or uncertainties about the correct operative site should be resolved by the surgeon with agreement by the patient and nursing staff. Specific and explicit protocols should be in place to address the manner in which these uncertainties are resolved.

The informed consent must specify laterality and attempt to localize multiple structures if surgery will be performed in multiple locations.

88
Q

A 34-year-old man is brought to the emergency department 2 hours after sustaining injuries to the right wrist when he punched a glass window. Surgical exploration shows a complete laceration of the median nerve at the level of the wrist. A 1-cm gap between the proximal and the distal stumps of the nerve is noted. Which of the following treatments is most likely to provide the best functional outcome?

A) Multistrand nerve grafting
B) Nerve transfer
C) Nerve transposition
D) Primary epineurial repair
E) Single-strand nerve grafting

A

The correct response is Option D.

The need for nerve grafting is dependent upon many parameters, such as the length of the gap, the excursion of the nerve, the wound bed, and vascularity, among others. For clean, sharp injuries with nerve gaps measuring less than 1 cm in a large peripheral nerve such as the median, most authors agree that primary repair of the nerve results in the best outcome. Autologous nerve grafting should be reserved for cases in which there is tension on the nerve ends with primary repair. Both multistrand nerve grafting and single-strand nerve grafting produce similar outcomes and are inferior to primary repair. Nerve transfer would only be considered if there were no proximal nerve to repair to the distal nerve. Nerve transposition would only be appropriate for gaining length in the ulnar nerve, where the switch from the extensor side of the elbow to the flexor side results in increased relative length in the nerve.

89
Q

A 40-year-old man who is an avid cyclist comes for evaluation of a 5-week history of numbness and tingling of the ring and little fingers of the left hand. Conservative management has failed. Physical examination shows weakness of grip strength. Sensation over the dorsal ulnar aspect of the hand is intact, and results of elbow flexion testing are negative. Which of the following structures are most appropriate for surgical release?

A ) Osborne ligament and arcade of Struthers

B ) Osborne ligament and volar carpal ligament

C ) Pisohamate ligament and volar carpal ligament

D ) Transverse carpal ligament and arcade of Struthers

E ) Transverse carpal ligament and pisohamate ligament

A

The correct response is Option C.

The patient described has ulnar tunnel syndrome or compression of the ulnar nerve in the Guyon canal. Sensory sparing on the dorsal ulnar aspect of the hand suggests a lesion distal to the origin of the dorsal cutaneous branch of the ulnar nerve. Nerve conduction studies and electromyography can be used to confirm the diagnosis. Conservative treatment includes activity modification, splinting, and administration of a nonsteroidal anti-inflammatory drug.

The most appropriate management of this condition is exploration of the entire ulnar tunnel and release of the pisohamate and volar carpal ligaments. The ulnar tunnel, which is 4 to 4.5 cm in length, begins at the proximal volar carpal ligament and ends at the fibrous edge of the hypothenar muscles. Zone I is the region of the tunnel proximal to the bifurcation of the nerve. Zone II is the area around the deep motor branch, which ends at the pisohamate ligament, and Zone III is the area surrounding the superficial branch. The ulnar nerve courses between the volar carpal ligament and the transverse carpal ligament.

The Osborne ligament and the arcade of Struthers are potential sites of compression of the ulnar nerve at the elbow, or cubital tunnel syndrome. The fascia of Osborne, or the cubital tunnel retinaculum, is a band bridging the two origins of the flexor carpi ulnaris muscle and the medial epicondyle. The arcade of Struthers is 8 to 10 cm proximal to the medial epicondyle and extends from the medial intermuscular septum to the medial head of the triceps.

Release of the transverse carpal ligament is appropriate for management of compression of the median nerve at the wrist, or carpal tunnel syndrome.

90
Q

A 45-year-old man is brought to the emergency department immediately after sustaining injuries to the right upper extremity during a motorcycle collision. On admission, physical examination shows a flaccid and insensate right upper extremity, and x-ray studies show no abnormalities. Follow-up nerve conduction studies 4 weeks later show preservation of sensory nerve action potentials and evidence of fibrillations and denervation in the motor action potentials from the paravertebral muscles, biceps, triceps, and deltoid. The nerve injury is most likely located at which of the following levels of the nerve?

A ) Anterior division

B ) C5 and C6 trunk

C ) Lateral cord

D ) Preganglionic root

E ) Subscapular nerve

A

The correct response is Option D.

Following a traction injury to the brachial plexus, the nerves may rupture, be avulsed at the level of the spinal cord, or significantly stretch but remain intact. There are five possible levels where the nerve can be injured: (1) root, (2) anterior branches of the spinal nerves, (3) trunk, (4) cord, and (5) peripheral nerve. Root injuries may be further localized with respect to the dorsal root ganglion. Infraganglionic (postganglionic) injuries are located distal to the dorsal root ganglion, and supraganglionic (preganglionic) lesions are located proximal to the ganglion. With both types of lesions, patients will have the symptom of loss of muscle function. In supraganglionic injuries, the nerve has been avulsed from the spinal cord, separating the motor nerve fibers from the motor cell bodies in the anterior horn cells. The sensory fibers and cell bodies are still connected at the dorsal root ganglion; however, the efferent fibers entering the dorsal spinal column have been disrupted. Thus, sensory nerve action potentials will be preserved in patients with supraganglionic injuries, while motor nerve action potentials will be absent. In infraganglionic injuries, both the motor and sensory nerve cells have been disrupted, so there will be abnormalities in both motor and sensory action potentials. An injury to the C5 and C6 roots would show preservation of triceps function, as would an injury to the medial or lateral cord.

91
Q

A 48-year-old woman has numbness and paresthesia of the right hand 2 years after mastectomy and radiation therapy. She reports no pain or night waking. Symptoms have not improved with cock-up wrist splints or injection of a corticosteroid into the carpal tunnel. Physical examination shows swelling and weakness of the right arm, most prominently in the C5-C6 distribution; no varicosities, stasis ulcers, dermatitis, or symptoms of Horner syndrome are noted. Allen test is normal. Electromyography shows myokymia. CT scan shows diffuse swelling but no mass. Which of the following is the most likely diagnosis?

A ) Acute ischemic injury

B ) Carpal tunnel syndrome

C ) Chronic venous insufficiency

D ) Radiation-induced brachial plexopathy

E ) Tumor recurrence

A

The correct response is Option D.

The most likely diagnosis is radiation-induced brachial plexopathy, which can occur when radiation therapy is directed at the chest, axillary region, thoracic outlet, or neck. The incidence is 1.8 to 4.9% of those patients receiving radiation therapy to the above areas and is most common in patients with underlying breast or lung carcinoma. Patients often have sensory symptoms, with swelling and a generalized weakness of the arm. Eighteen percent of patients have pain in the shoulder, wrist, or hand. The neurologic findings are most prominent in the C5-C6 distribution. The lymphatic-vascular system may show prominent lymphedema of the involved extremity without cyanotic or dusky features. There should be no disturbance of arterial or venous circulation in the involved extremity and no changes in the limb to suggest venous insufficiency (ie, varicosities, stasis ulcers, or dermatitis). The Allen test should be normal. Horner syndrome is not present in patients with radiation-induced brachial plexopathy.

Eighty percent of patients with tumor infiltration into the brachial plexus come to the office because of pain in the shoulder, upper arm, elbow, and ring and little fingers. Symptoms progress to atrophy and weakness of the C7-T1 distribution with persistent pain and occasional Horner syndrome. CT scan shows a discrete mass with circumscribed tissue infiltration. Electromyography shows segmental slowing.

Patients with acute ischemic injury have symptoms of paresthesia in the C5-C6 nerve distribution and acute, nonprogressive weakness and sensory loss. CT angiography would

demonstrate subclavian artery segmental obstruction. Electromyography shows segmental slowing.

Patients with carpal tunnel syndrome often have night waking and experience a period of symptomatic relief after injection of a corticosteroid into the carpal tunnel.

Chronic venous insufficiency does not typically have neurologic sequela.

92
Q

A 54-year-old woman comes to the office because of a 6-year history of weakness and numbness of the left hand. Physical examination shows decreased sensation in the thumb, index, long, and ring fingers. No other sensory abnormalities are noted. Examination of which of the following muscles is most likely to confirm a diagnosis?

A ) Abductor pollicis brevis

B ) Adductor pollicis

C ) First dorsal interosseous

D ) Flexor digiti minimi

E ) Flexor pollicis brevis

A

The correct response is Option A.

The examination of the patient described suggests an injury or compression neuropathy of the median nerve. The only intrinsic muscle innervated by the median nerve (recurrent branch) that can be reliably tested separately from the ulnar intrinsic muscles is the abductor pollicis brevis. The adductor pollicis, first dorsal interosseous, and flexor digiti minimi are all completely innervated by the ulnar nerve. The flexor pollicis brevis muscle has dual innervation from both the ulnar (deep head) and median (superficial head) nerves.

93
Q

A 30-year-old woman comes to the emergency department after cutting herself with a kitchen knife. Surgical exploration shows that the median nerve had been cut at the distal forearm. The distal nerve is relatively fixed, but the proximal nerve has been lifted and twisted. Immediate repair is planned. The motor fibers of the proximal nerve end are most likely in which of the following locations relative to the sensory fibers?

A ) Dorsal and radial

B ) Dorsal and ulnar

C ) Volar and radial

D ) Volar and ulnar

A

The correct response is Option C.

At the level of the distal forearm, the median nerve is a mixed nerve comprised of both motor (20%) and sensory (80%) fibers. The motor fibers become the thenar branch, which innervates the abductor pollicis brevis (AbPB), opponens pollicis (OP), and flexor pollicis brevis (FPB). The FPB is located distal to the AbPB and OP and also has innervation from the ulnar nerve, which is why patients with median nerve injuries can sometimes still bring the thumb to the little finger. The thenar nerve can be injured with a carpal tunnel release procedure. After coursing into the carpal tunnel, these median nerve motor fibers leave the median nerve volar and radial to the sensory fibers of the median nerve through a variety of branching patterns.

When patients have clean-cut injuries to nerves, the nerves can be repaired primarily. In a contaminated, dirty wound caused by a crush and mutilating injury, it can be difficult to determine the nonviable nerve required for debridement before repair. In the scenario described, the nerve ends are labeled with a permanent suture for later identification and repair at a second stage. In general, sensory-only nerves can be repaired by epineurial approximation, and group fascicular repair can be considered for mixed nerves.

94
Q

A 68-year-old woman comes to the office because of a 4-year history of numbness and tingling in the tips of the thumb, index, and long fingers. The symptoms have become progressively severe and now wake her at night. Physical examination shows decreased strength of thumb opposition and thenar atrophy. Sensation to pinprick is diminished over the median nerve distribution. Which of the following findings is most likely on electrodiagnostic testing?

Median Sensory Latency | Median Motor Latency | Fibrillations

A ) Decreased decreased absent

B ) Decreased decreased present

C ) Unchanged unchanged absent

D ) Increased increased absent

E ) Increased increased present

A

The correct response is Option E.

The most likely finding on electrodiagnostic testing is increased median sensory latency, increased median motor latency, and presence of fibrillations.

For the patient described, compression of the median nerve results in numbness and paresthesias in the median nerve distribution. The disease has progressed to the point where physical findings include decreased strength and thenar atrophy, indicating the presence of denervation and median nerve damage.

On nerve conduction studies (NCS), the sensory and motor latency measurements detect the time necessary for a signal to travel across a segment of nerve from a stimulating electrode to a recording electrode. In the presence of nerve compression, demyelination results and conduction is slowed, resulting in a longer time and increased latency (measured in milliseconds).

On electromyography (EMG), the needle examination will assess the presence of denervation changes, manifested as fibrillations and positive sharp waves. These are signs of ongoing nerve injury and muscle denervation. Spontaneous muscle activity with fibrillation potentials is the earliest sign of denervation. In severe carpal tunnel syndrome, examination of the median-innervated abductor pollicis brevis muscle will reveal changes. There may also be alteration in the motor unit potentials caused by chronic episodes of denervation and reinnervation.

Carpal tunnel syndrome remains a clinical diagnosis based on history and physical examination. Clinical suggestion may be confirmed by electrophysiologic testing, but false-negative and false-positive results may occur.

95
Q

Volkmann ischemic contracture occurs when forearm compartment pressure is above 30 mmHg for a minimum of approximately how many hours?

A ) Less than 2

B ) 3-5

C ) 6-12

D ) 18-24

E ) Greater than 24

A

The correct response is Option C.

Permanent neuromuscular damage occurs at 12 hours, leading to subsequent Volkmann contracture. Timeline of ischemia, shown experimentally, is capillary endothelial damage at 3 hours; partially reversible muscle and nerve injury occurs at 6 hours.

96
Q

A 30-year-old man who works as an ironworker is brought to the emergency department after falling 20 ft and impaling his right upper arm on a picket fence. Surgical exploration shows a complete transection of the ulnar nerve 6 in

(15.2 cm) above the elbow with a 2-cm nerve gap. The median and radial nerves are intact. Which of the following procedures is most likely to result in maximal recovery of intrinsic muscle function?

A ) Anterior transposition of the ulnar nerve at the elbow followed by primary repair

B ) Interposition nerve grafting using the lateral antebrachial cutaneous nerve

C ) Nerve transfer of the distal anterior interosseous nerve to the deep motor branch of the ulnar nerve

D ) Tendon transfer of the extensor digiti quinti to the extensor digitorum communis

E ) Vascularized radial nerve grafting

A

The correct response is Option C.

High ulnar nerve injuries, ie, above the elbow, have routinely yielded unsatisfactory results with minimal return of intrinsic function and resultant claw hand deformity. This poor recovery of motor function of the intrinsic muscles of the hand occurs despite meticulous neurorrhaphy technique and regardless of whether nerve graft or primary repair is performed. The principal reason for poor results is the long distance between the site of injury and the muscle motor endplates. During the many months required for the regenerating axons to traverse this gap, the target muscles are undergoing atrophy and fibrosis. The rate of nerve regeneration is approximately 1 inch per day. Motor endplates become refractory to reinnervation after about 15 to 18 months. In contrast, sensory return from high nerve injuries is usually adequate.

The key to nerve transfer is to convert a high nerve injury to a low nerve injury. In this case, the original injury is over 15 inches from the ulnar nerve innervations of the intrinsic muscles. The likelihood of any intrinsic muscle function is poor. By transferring the distal anterior interosseous nerve at the level of the pronator quadratus to the deep motor branch of the ulnar nerve, the distance that the reinnervating nerve has to travel is much shorter, thus preserving the muscle structure. The loss of pronator quadratus function is insignificant. When determining the need for nerve transfer, the time vs. distance constraints can be guided by the €œRule of 18. € The Rule of 18 states: €œThe number of inches from the site of nerve injury to the supplied muscle added to the number of months the muscle has been denervated should be less than the number 18 in order for the primary nerve reconstruction to be attempted successfully. € In other words, the nerve must be able to get to the muscle before 18 months from the initial injury.

The tendon transfer of the extensor digiti quinti to the extensor digitorum communis is inadequate to recover intrinsic function. The other options presented are all inappropriate because they do not address the length of time it will take the nerve to grow to the motor endplates.

97
Q

A newborn who sustained a traction injury to the shoulder during delivery because of the use of forceps has complete palsy of the left upper extremity. Surgical intervention is indicated if there is no spontaneous biceps recovery by what age?

A ) 6 weeks

B ) 6 months

C ) 18 months

D ) 24 months

E ) 36 months

A

The correct response is Option B.

Most experts will perform surgery on infants who are between 3 and 6 months of age. At 3 months of age, if biceps function is absent, there is a poor prognosis. Early on, 92% will recover spontaneously. If biceps function is noted, surgery is not recommended. At 3 months of age, the child is retested by electromyography if there is no biceps function, although there is poor correlation with final results. The final evaluation is cervical myelography.

98
Q

A 23-year-old man is brought to the emergency department after sustaining a single gunshot wound to the right upper arm. Physical examination shows a high radial nerve palsy. Which of the following is the most appropriate first step in management of potential nerve injury?

A ) Immediate surgical exploration and primary nerve repair if nerve is lacerated

B ) Immediate surgical exploration and repair with a nerve graft if nerve is lacerated

C ) Immediate surgical exploration, resection of devitalized nerve, and suture tagging of nerve ends for delayed repair

D ) Observation with electromyography six weeks after injury followed by exploration and repair if no return of function

E ) Observation with electromyography six months after injury followed by exploration and repair if no return of function

A

The correct response is Option D.

In general, nerve injuries associated with open wounds should be explored and repaired early. If the nerve injury resulted from a relatively clean laceration or wound, it should be explored and repaired immediately. Crush or significant soft-tissue injury prohibits early nerve repair until the extent of devitalized tissue can be determined and the soft-tissue repair is stable.

Although gunshot wounds are technically open injuries, they should be treated as closed or blunt nerve trauma because the etiology of the trauma is predominately heat and shock. The majority of nerve dysfunction spontaneously recovers after gunshot wounds. Thus, the most logical first step in treatment is observation for at least six weeks at which time electrodiagnostic studies can be undertaken.

If there is not complete clinical return of function within six weeks, electrodiagnostic studies should be performed for baseline function and repeated at 12 weeks if clinical return of function is still not complete. If electromyography does demonstrate motor unit potentials, expectant management should be continued as full function should return. Electrodiagnostic studies should not be delayed for six months because long-term function is significantly decreased if nerve repair is delayed longer than three months.

Lack of clinical or electrical evidence of reinnervation at three months requires surgical exploration. A nerve that has been completely divided or fails to conduct intraoperatively should be managed with resection of scar tissue and repair of the nerve gap with an interpositional nerve graft. An in-continuity lesion should undergo neurolysis of nonfunctioning nerve units and nerve grafting. The functioning units can remain intact.

99
Q

A 52-year-old man comes to the office because he has had a clinical diagnosis of carpal tunnel syndrome including numbness and dysesthesia of the thumb, index, and long fingers of both hands for the past four months. He says his symptoms worsen at night and when his wrist position changes while driving. Electrodiagnostic studies show delayed motor and sensory latencies and slowed conduction velocity at the wrist, indicating a conduction block of the median nerve. This block is most likely caused by localized nerve ischemia that results in which of the following?

A ) Blocking of calcium channels

B ) Failure to maintain the resting nerve potential at −30 mV

C ) Opening of sodium channels

D ) Prevention of nerve membrane depolarization

E ) Thickening of the myelin sheath

A

The correct response is Option D.

Local nerve ischemia prevents depolarization. A charge is maintained across the axon membrane with the interior of the axon having a charge of −90 mV. When the resting membrane potential reaches −50 mV, the membrane depolarizes. Since there is a relatively greater concentration of K+ ions on the inside of the axon, there is a slow leak of K+ ions, which causes the inside of the axon to become less negative. The ATP-dependent pump imports K+ and exports Na+, maintaining the normal resting membrane potential at −90 mV. Maintenance of this ionic charge separation across the membrane requires energy, and the mechanism stops when the energy supply is interrupted, as with local ischemia. This is one of the mechanisms for conduction block that occurs with nerve compression. Various membrane channels consist of proteins embedded in the phospholipid membrane bilayer that allow passage of Na+, Ca2+, and K+ ions. With chronic nerve compression, the myelin sheath disintegrates and saltatory conduction between nodes of Ranvier ceases.

100
Q

A 25-year-old man who is blind is brought to the emergency department two hours after sustaining a crush injury to the index finger of the dominant right hand at the flexion crease of the metacarpophalangeal joint. Physical examination shows full flexion and extension of the index finger. Sensation to pinprick is absent in the fingertip. Radiographs show no abnormalities. During surgical exploration, disruptions to the radial and ulnar digital nerves are noted. After debridement of the traumatized nerve ends, a 10-mm gap is noted in each digital nerve. Which of the following is the most appropriate management of each digital nerve?

(A) Bone shortening with primary repair

(B) Flexion of the index finger with primary repair

(C) Mobilization of nerve ends with primary repair

(D) Polyglycolic acid conduit between nerve ends

(E) Radial sensory nerve graft between nerve ends

A

The correct response is Option D.

In the first prospective, randomized multicenter evaluation of a bioabsorbable conduit for nerve repair, Weber et al. reported superior sensory recovery when polyglycolic acid conduits were used to repair nerve gaps greater than 8 mm compared to nerve graft repairs.

Although bone shortening has been described as a means of relieving tension on primary nerve repairs, this is clearly not the best choice when less morbid options for nerve reconstruction exist for the patient described. Another conduit available for small nerve gaps is vein, although that offers little mechanical resistance to kinking and collapse in larger defects.

Mobilization of the nerve with primary repair would likely place the nerve repair under some tension, which inhibits nerve regeneration. Management of short gaps by flexing joints to facilitate a tension €‘free repair can be problematic when the digit is mobilized because of scar formation.

Nerve grafts are an improvement over extensive nerve mobilization and limb positioning but are not without sequelae. Nerve graft harvest produces scarring, formation of neuroma, and loss of donor site function, in addition to increasing operating room and anesthesia time. The sural and posterior interosseous nerves are commonly used as grafts. Because of loss of sensitivity and potential for symptomatic neuromas, the radial sensory nerve is not commonly used as a graft.

101
Q

A 42-year-old woman comes to the office for evaluation of a severely painful mass on the upper right arm and numbness in the right forearm six months after undergoing decompression neuroplasty with submuscular transposition. This procedure was performed for progressively worsening compression of the dominant ulnar nerve at the elbow. She developed severe complex regional pain syndrome two weeks after the procedure. Physical examination shows a 0.5-cm mass proximal and central to the medial epicondyle that is adherent to the skin and extremely sensitive. Examination shows decreased sensation to light touch over the ulnar volar forearm. Following excision of the neuroma, which of the following is the most appropriate next step in management?

(A) Medial epicondylectomy

(B) Nerve stump implantation

(C) Stress occupational therapy program

(D) Topical lidocaine patch

(E) Ulnar nerve vein wrap

A

The correct response is Option B.

The ulnar nerve neuroplasty is complicated by complex regional pain syndrome (CRPS) with an associated nerve injury. The medial antebrachial cutaneous nerve (MABC) of the forearm originates from the medial cord of the brachial plexus (shown in the diagram below). The MABC derives from the C8 and T1 roots. In the mid arm, the MABC and the basilic vein pierce the deep fascia and provide sensation to the overlying ulnar volar forearm skin. The MABC can be injured during ulnar nerve at the elbow neuroplasty. If CRPS develops from an identifiable nerve injury, resolution is best achieved by identifying and correcting the nerve injury. In the scenario described, excision and tensionless implantation into adjacent muscle or bone is the most appropriate option.

Medial epicondylectomy would not address the primary MABC neuroma, nor would wrapping the ulnar nerve with a vein graft. A stress occupational therapy program can be useful in CRPS without an identifiable nerve injury. With the injury described, however, the pain more likely would worsen. Topical transdermal anesthetic patches can alleviate but not resolve the nerve pain.

102
Q

A 34-year-old man is brought to the emergency department one hour after sustaining a 7 cm laceration to the left wrist during a suicide attempt. The patient is unconscious but breathing. Physical examination shows a clean, linear radial laceration. Pulsatile arterial bleeding from the wound is noted. A tourniquet is applied to the upper left arm and inflated to 250 mmHg of pressure. Three hours later, the tourniquet is released, and the patient is brought to the operating room. The artery is ligated, and the tendons are repaired. Surgical exploration shows no nerve laceration. The next day, the patient has no sensation or motor function in the left hand. After three days, normal function returns. Which of the following Sunderland grades is the most appropriate classification for this injury?

(A) I

(B) II

(C) III

(D) IV

(E) V

A

The correct response is Option A.

Nerve injuries can be classified with a system that was first described by Seddon and modified by Sunderland. The table below summarizes these nerve injury classifications. The nerve injury described is classified as Sunderland grade I. Rapid nerve recovery is likely. The patient described had the tourniquet in place for longer than the generally accepted recommended period. Studies have shown that tourniquets should be released after two hours of use. Because of the prolonged use in the scenario described, the patient has sustained neurapraxia. When a tourniquet is used, the nerves and muscles are susceptible to injury. The nerves are particularly sensitive to pressure injury. Two-hour use is a guideline; if additional time is needed, it is generally thought that five minutes of reperfusion is necessary for every 30 minutes of tourniquet time.

A Sunderland grade I injury corresponds with neurapraxia. Complete recovery is expected. A grade II injury corresponds with axonotmesis and slow but complete recovery. Sunderland III, IV, and V injuries do not recover spontaneously. Axonotmesis and neurotmesis occur with Wallerian degeneration; therefore, significant regeneration often takes place only after surgery.

103
Q

A 37-year-old woman undergoing flexor tendon repair receives an axillary block. A tourniquet is used. Twenty minutes into the procedure, the patient says she has discomfort. Intolerance of the tourniquet is most likely caused by incomplete block of which of the following nerves?

(A) Axillary

(B) Median

(C) Musculocutaneous

(D) Radial

(E) Ulnar

A

The correct response is Option C.

The musculocutaneous nerve powers elbow flexion and forearm supination. Below the elbow, as the lateral antebrachial cutaneous nerve, it provides sensory function. An incompletely blocked musculocutaneous nerve is most likely to contribute to tourniquet intolerance. The axillary block is administered at the level of the terminal nerves of the brachial plexus. Both the musculocutaneous and axillary nerves leave the axillary sheath proximal to the level of injection. Because of this, they may be inadequately blocked with this anesthetic technique. Because the sensory distribution of the axillary nerve is above the level of the tourniquet application, it is less likely to contribute to tourniquet intolerance than an incompletely blocked musculocutaneous nerve. Multiple injection techniques, paresthesia elicitation, nerve stimulation, and ultrasound guidance increase the probability of a successful block. Administration of additional local anesthetic within the sheath, within the coracobrachialis muscle, or at the elbow can be used to block the musculocutaneous nerve if necessary. The interocostobrachial nerve conveys sensation from the medial aspect of the arm and must also be blocked for tourniquet tolerance.

104
Q

A 35-year-old man sustains a laceration of the volar aspect of the left index finger at the level of the mid phalanx while using a knife. There is smooth flexion and extension of the proximal interphalangeal and distal interphalangeal joints. Examination of the fingertip shows good color and capillary refill. Two-point discrimination is 5 mm on the ulnar aspect of the fingertip but is absent on stimulation of the radial aspect. On exploration, a laceration involving 45% of the cross-sectional area of the flexor digitorum profundus (FDP) tendon is noted. The radial digital nerve and artery are transected. The ulnar neurovascular bundle is intact. In addition to repair of the radial digital nerve, which of the following is the most appropriate management?

(A) Immediate protected mobilization

(B) Repair of the FDP tendon with core and epitendinous sutures and splint immobilization for two weeks followed by protected mobilization

(C) Repair of the FDP tendon with core sutures and splint immobilization for two weeks followed by protected mobilization

(D) Repair of the FDP tendon with epitendinous sutures and splint immobilization for four weeks followed by protected mobilization

(E) Splint immobilization for four weeks followed by protected mobilization

A

The correct response is Option A.

Tendon lacerations of less than 60% of the cross-sectional area of the tendon do not require repair. Attempts to repair the tendon may weaken it. Triggering may occur if the distal or proximal cut edge of the tendon catches on adjacent pulleys. If triggering occurs, trimming the cut edges of tendon, partial or complete pulley release, and repair of the flexor sheath can be performed. In cases of persistent triggering that persists despite these measures, the tendon can be repaired. In the absence of triggering, the tendon does not require treatment. Protected mobilization is performed to promote tendon gliding and strengthening. Despite early recommendations for several weeks of immobilization following digital nerve repair, the outcomes of digital nerve function following immediate protected mobilization are not statistically different than those managed with immobilization.

105
Q

A 17-year-old girl comes to the office because she has persistent wrist drop and weakness of digit extension three weeks after she fractured the intraarticular distal portion of the radius while snowboarding. Open reduction and internal fixation was performed under general anesthesia. A wide-cuff, pneumatic tourniquet was placed over cast padding; inflation pressure was 100 mmHg over the patient’s systolic blood pressure. Reduction was difficult, and tourniquet time was 180 minutes. Use of which of the following interventions is most likely to have decreased the risk of this complication?

(A) Elastic stocking rather than cast padding beneath the tourniquet

(B) Esmarch bandage as the tourniquet

(C) Narrow width tourniquet cuff

(D) Standard inflation pressure of 250 mmHg

(E) Tourniquet deflation/reperfusion period

A

The correct response is Option E.

The intervention most likely to decrease the incidence of nerve palsy in the patient described is providing a period of tourniquet deflation ( breather period ) during which tissue acidosis may reequilibrate.

Despite advances in equipment and technique, complications from tourniquet use in upper extremity surgery still occur. The most common problems involve either neurologic or skin complications. Experimental studies suggest that most neurologic sequelae appear to be related to prolonged ischemia, high local tourniquet pressure, or some combination of the two.

Both elastic stockings and cast padding have been shown to decrease skin complications from tourniquet use; neither has been shown to have any effect on neurologic complications.

Esmarch bandages, although effective as tourniquets, provide unpredictable levels of pressure on the arm and have consequently been replaced by automated cuffs and pumps.

Wide tourniquets, with ratios of cuff width to arm circumference greater than 0.5:1, have been shown to achieve hemostasis at lower pressures than narrow cuffs.

Although many surgeons reflexively set upper extremity tourniquets at 250 mmHg, the optimal inflation pressure to minimize risk and maximize hemostasis appears to vary widely depending on body habitus, type of anesthesia, and comorbidities such as hypertension.

106
Q

A 20 year old man comes to the office after he sustained transection of the ulnar nerve of the left arm during a rollover motor vehicle accident. Physical examination shows a 5-cm transverse laceration in the proximal ulnar aspect of his left forearm. A photograph of the laceration after suture removal is shown. Microscope-assisted epineural repair of the transection is performed. As function is restored over time, which of the following movements will most likely be last to return?

(A) Abduction of the small finger

(B) Adduction of the thumb

(C) Flexion of the metacarpophalangeal joint of the small finger

(D) Flexion of the proximal interphalangeal joint of the small finger

(E) Ulnar €‘sided flexion of the wrist

A

The correct response is Option B.

The ulnar nerve is the terminal portion of the medial cord of the brachial plexus, after the medial head of the median nerve has separated from it, with fibers from C8-T1. It initially lies medial to the axillary artery and then to the brachial artery as it travels distally in the upper arm. It pierces the intermuscular septum and then follows the medial head of the triceps muscle to the groove between the olecranon process and the medial epicondyle. It gives off no branches in the arm. After the ulnar nerve passes through the cubital tunnel, it gives off articular branches and branches to the flexor carpi ulnaris (FCU) and the medial half of the flexor digitorum profundus (FDP). It travels between the two heads of the FCU and continues into the forearm between this muscle and the FDP.

In the distal half of the forearm, it is joined on its lateral side by the ulnar artery. Proximal to the wrist, the nerve gives off the dorsal sensory branch, providing sensation to the dorsal wrist and ulnar aspect of the hand. The ulnar nerve continues into the hand via Guyon €™s canal, where it splits into a superficial sensory branch and a deep motor branch. The superficial branch supplies the palmaris brevis and the skin of the hypothenar eminence and digital nerves to the small and ulnar side of the ring finger. The motor branch passes between the abductor digiti minimi (ADM) and the flexor digiti minimi (FDM), with the deep branch of the ulnar artery, perforates the opponens digiti minimi and follows the deep volar arch across the interossei, and finally innervates the adductor pollicis (AD) and the deep head of the flexor pollicis brevis.

Based on its path, ulnar €‘sided flexion of the wrist (FCU) would be expected early, followed by abduction of the small finger (ADM) and flexion of the metacarpophalangeal joint (FDM). Interosseus muscle function would manifest as ability to abduct and adduct the fingers, followed last by adduction of the thumb (AD). Flexion of the proximal interphalangeal joints of the fingers is a function of the flexor digitorum superficialis, which is innervated by the median nerve.

107
Q

A 37-year-old man comes to the emergency department because he has pain and swelling of the right hand as well as numbness of the fingertips two hours after undergoing CT scan of the abdomen because of abdominal pain. Physical examination shows lack of full active extension of the fingers and exquisite tenderness on passive adduction and abduction of the fingers. Pulses are intact. High-pressure infiltration of intravenous radiographic contrast medium into the hand is suspected. Which of the following is the most appropriate management?

(A) Administration of nonsteroidal anti-inflammatory drugs and follow-up evaluation in the office

(B) Elevation of the arm, application of ice, and re-evaluation in six hours

(C) Emergency fasciotomy

(D) Initiation of heparin therapy for anticoagulation

(E) Subcutaneous injection of calcium gluconate into the arm

A

The correct response is Option C.

The condition described is clearly a developing compartment syndrome that should be emergently decompressed in the operating room. The diagnosis should not be delayed until pulses are absent because absence of pulses is a late finding. Re-evaluation in six hours is not appropriate for the patient described because, by that time, irreversible muscle necrosis may occur, resulting in potential long-term disability. Similarly, having the patient follow up in the office is not appropriate. Initiation of heparin therapy for anticoagulation may be indicated in an ischemic event but is not appropriate in this scenario given the history and findings of the patient described. Subcutaneous injection of calcium gluconate is the treatment of choice for hydrofluoric acid burns, not for intravenous contrast infiltration.

In the case of a €œnonalert € patient, compartment pressures may be measured using an intracompartmental pressure monitor. Such devices are commercially available or may be fabricated. Interstitial compartment pressures of > 30 mmHg are consistent with compartment syndrome.

108
Q

A 21-year-old man comes to the office because he has had absence of sensation of the right thumb since he underwent repair of the digital nerve on the opposition side of the digit one year ago. On physical examination, Tinel sign is absent at the surgical scar. After surgical exploration and debridement of the thumb, a 3-cm nerve gap is noted. Which of the following is the most appropriate method of digital reconstruction?

(A) Mobilization of the free nerve ends and primary repair

(B) Polyglycolic acid tube conduit

(C) Posterior interosseous nerve grafts

(D) Radial central nerve grafts

(E) Vein conduit

A

The correct response is Option C.

Repair or reconstruction of a digital nerve requires a tensionless technique. With tension, repair of the nerve will cause scarring and form a neuroma. If the gap is too large, a graft or conduit is required for a tensionless reconstruction. The terminal posterior interosseous nerve graft is ideal because it provides a nerve with similar caliber for the repair. Harvesting of this nerve avoids a donor deficit because at the level of the distal forearm wrist it only provides for wrist sensation.

Large nerve defects should not be reconstructed with vein or polyglycolic acid tube conduits. The radial sensory nerve is too large in diameter. Additionally, the sensory loss would be more problematic and would risk a painful neuroma.

109
Q

A 21 year-old woman who rows regularly with her university’s crew rowing club comes to the office because she has had pain on movement of the right thumb as well as pain and swelling of the distal forearm for the past month. She has not sustained trauma to the hand or arm. Physical examination shows tenderness and crepitus 4 cm proximal to the wrist and over the distal and radial aspects of the forearm. Slight discomfort is noted on ulnar deviation of the wrist with the thumb clasped in the palm. No grinding or crepitance of the thumb carpometacarpal joint is noted. Which of the following is the most appropriate initial step in management of this patient’s condition?

(A) Injection of a corticosteroid into the first dorsal compartment

(B) Injection of a corticosteroid into the third dorsal compartment

(C) Splinting of the carpometacarpal joint of the thumb

(D) Splinting of the wrist in extension

(E) Observation

A

The correct response is Option D.

The patient described has findings consistent with intersection syndrome, a tenosynovitis of the second dorsal compartment (common radial wrist extensors) associated with pain and swelling of the muscle bellies of the abductor pollicis longus (APL) and extensor pollicis brevis (EPB), 4 cm proximal to the wrist joint (ie, where the muscle bellies of APL and EPB cross extensor carpi radialis longus and extensor carpi radialis brevis). As in other stenotic conditions of the hand and wrist, it is associated with repetitive motions of the wrist and is frequently seen in athletes, especially rowers and weightlifters. Initial nonoperative treatment consists of rest, nonsteroidal anti-inflammatory drugs (NSAIDs), splinting of the wrist in 15 degrees of extension, and, in some patients, injection of a corticosteroid into the second dorsal compartment. For those with persistent pain, surgical release of the second dorsal compartment beginning at the wrist and extending proximal to the area of swelling may be necessary.

De Quervain disease, a tenosynovitis of the first dorsal compartment, presents with wrist pain on the radial side aggravated by movement of the thumb, most often in women aged 40 €“60 years. Local tenderness and swelling 1 to 2 cm proximal to the radial styloid and knifelike pain on Finklestein test (clasped thumb in palm while wrist is ulnarly deviated) is diagnostic. In mild cases, rest, NSAIDs, and splinting of the wrist in gentle extension and the thumb widely abducted may be successful. Injection of a corticosteroid into the first dorsal compartment may be successful in 50% €“80% of patients following one or two injections. If nonsurgical modalities are unsuccessful, then surgical release of the first dorsal compartment is successful, if variations of this compartment are appreciated and addressed.

Tenosynovitis of the extensor pollicis longus (third dorsal compartment) is rare but requires early diagnosis and urgent operative treatment to prevent tendon rupture, a complication rarely seen with de Quervain disease, trigger finger, or trigger thumb. It presents with pain, swelling, tenderness, and often crepitus at Lister €™s tubercle. Surgical treatment consists of third dorsal compartment release and tendon transposition radial to Lister €™s tubercle. Injection of a corticosteroid is rarely indicated.

Degenerative arthritis of the thumb most frequently affects the carpometacarpal joint. Diagnosis can be made by a positive grind test (ie, crepitations elicited with axial loading combined with the rotation of the thumb metacarpal), and confirmed with radiographs that show destruction of the articular surface and joint space. Early-stage disease may be treated with rest, splinting, NSAIDs, and thenar strengthening. If this fails to relieve pain, then ligament reconstruction may be performed, based on the extent of joint surface destruction. As the patient progresses to end-stage degeneration of the carpometacarpal joint, carpometacarpal arthrodesis, trapezium hemiprosthesis, ligament reconstruction-tendon interposition arthroplasty, or hemi- or complete trapeziectomy may be appropriate management.

110
Q

A 58-year-old woman who is a pianist comes to the office because she has had increasing clumsiness of the right hand for the past four months. She says the awkward movement of her hand makes it difficult for her to play the piano. Physical examination shows marked atrophy of the first dorsal interosseous muscle. Two-point discrimination is 3 to 4 mm in all fingers. The most likely cause of these findings is nerve compression at which of the following sites?

(A) Arcade of Struthers

(B) Carpal tunnel

(C) Guyon canal

(D) Ligament of Struthers

(E) Osborne ligament

A

The correct response is Option C.

Isolated intrinsic motor weakness is most often a result of compression of the motor branch of the ulnar nerve with the ulnar tunnel, also referred to as the Guyon canal. The ulnar tunnel is divided into three zones. Within zone 2 of the ulnar tunnel, the deep motor branch passes around the hook of the hamate and between the pisohamate ligament and the fibrous arch of the flexor digiti minimi. The motor branch goes on to innervate the abductor digiti minimi, the flexor digiti minimi, the opponens digiti minimi, the small and ring lumbricals, the palmar and dorsal interosseous muscles, the adductor pollicis, and the deep head of the flexor pollicis brevis muscle. The terminal portion of the nerve innervates the first dorsal interosseous muscle. Causes for compression include ganglion, hamate hook fracture, lipoma, and hypothenar hammer syndrome.

Although isolated motor nerve compression can occur, the ulnar nerve is most commonly compressed at the level of the elbow, which is referred to as cubital tunnel syndrome. Cubital tunnel syndrome classically presents with sensory changes within the small and ring finger followed by weakness in the intrinsic muscles and, in severe cases, weakness of the profundus tendon to the small and ring finger. Surgical release in cases of cubital tunnel syndrome involves release of the Osborne ligament as well as the arcade of Struthers.

The carpal tunnel is the most common site of median nerve compression and is not the most common site of compression in this scenario. The ligament of Struthers may be involved in median nerve compression at the level of the elbow.

111
Q

A 13-year-old girl with cerebral palsy is brought to the office by her parents because she has had inability to voluntarily extend the dominant right wrist as well as weak grasp of the right hand since birth. On physical examination, when the fingers are held straight, the wrist can be brought to neutral. During grasp, there is ulnar deviation of the hand, and during extension of the fingers, there is radial deviation of the hand. Active extension of the fingers is intact, but active extension of the wrist is diminished. Dynamic electromyography confirms that function of the flexor carpi ulnaris (FCU) and flexor carpi radialis (FCR) tendons is intact. Which of the following is the most appropriate intervention for improvement in function of the hand?
(A) Fusion of the wrist
(B) Proximal row carpectomy
(C) Transfer of the superficialis tendon to the profundus tendon
(D) Transfer of the FCR tendon to the extensor carpi radialis brevis tendon
(E) Transfer of the FCU tendon to the extensor carpi radialis brevis tendon

A

The correct response is Option E.

In this patient, there is a lack of wrist extension, which severely limits grip strength and hand function. In patients with cerebral palsy, poor grasp is often secondary to weak or absent wrist extension. Returning active wrist extension will increase the mechanical advantage of the digital flexor tendons and increase grasp strength. Active wrist extension may be created with a tendon transfer.

During grasp, this patient=s hand deviates to the ulnar aspect, suggesting a functioning flexor carpi ulnaris (FCU) tendon. Therefore, this tendon may be transferred to augment wrist extension. Muscles that fire during grasp work well when transferred for wrist extension, whereas muscles that act while the patient attempts to release objects work better for finger extension. Because this patient=s wrist tends to deviate radially with finger release, the flexor carpi radialis (FCR) would be better transferred as a finger extensor. It is important to ensure that the FCR is working, if the FCU is going to be transferred.

Evaluation of the fingers and wrist is important before any tendon transfer. If the proximal interphalangeal and distal interphalangeal joints are passively extendable with wrist extension, then the digital flexors are relatively loose and do not require release before transfer. If the finger flexors were very tight, a transfer of the superficialis to the profundus tendon may be required to improve finger function. In cases of severe wrist contracture, a proximal row carpectomy may be necessary to allow extension of the wrist to neutral before tendon transfer. Wrist fusion may be an option in cases in which hygiene or cosmesis are the main concern or when no other tendons are available for transfer, but it must be remembered that this will remove any tenodesis effect obtained by flexing or extending the wrist.

112
Q

A 25-year-old man who operates a jackhammer has had gradual onset of numbness and tingling for the past year. He says the discomfort awakens him every night and he is often clumsy when handling objects. Physical examination shows decreased sensation in the thumb, index finger, long finger, and radial ring finger. Weakness of thumb opposition and weakness of flexion of the thumb, index finger, and long finger are also noted. Which of the following is the most likely diagnosis?
(A) Anterior interosseous nerve palsy
(B) Carpal tunnel syndrome
(C) Cubital tunnel syndrome
(D) Posterior interosseous nerve palsy
(E) Pronator syndrome

A

The correct response is Option E.

Carpal tunnel syndrome, compression of the median nerve at the wrist, is the most common compression neuropathy of the upper extremity. It presents with pain and paresthesias in the cutaneous distribution of the median nerve, i.e., the thumb, index finger, long finger, and radial half of the ring finger, often worse at night and commonly exacerbated by repetitive use of the hand.

Median nerve compression neuropathies of the distal arm and proximal forearm are uncommon compared with carpal tunnel syndrome. The two most common problems of compression of the median nerve in the proximal forearm are anterior interosseous syndrome and pronator syndrome. Compression within different areas of the pronator teres results in either a pure motor deficit (anterior interosseous syndrome) or a mixed sensory and motor loss pattern (pronator syndrome). Forearm pain and numbness in the radial palm along with the thumb, index finger, long finger, and radial ring finger are common symptoms. Median intrinsic and extrinsic motor weakness also can be demonstrated on examination in pronator syndrome. The anterior interosseus nerve innervates the radial half of the flexor digitorum profundus (FDP), the flexor pollicis longus (FPL), and the pronator quadratus with sensory fibers to the radiocarpal, intercarpal, carpometacarpal, and radioulnar joints. Anterior interosseous nerve syndrome is characterized by impaired function of the FPL and FDP of the index finger with occasional involvement of the long finger and pronator quadratus. The thumb and index finger assume a classic position during pinch, with the distal interphalangeal (DIP) joint of the index finger extended and the proximal interphalangeal (PIP) joint of the index finger in compensatory flexion while the interphalangeal (IP) joint of the thumb is hyperextended.

Pronator syndrome presents with pain in the forearm as well as paresthesia and hypoesthesia in the cutaneous distribution of the median nerve. Patients may also complain of extremity weakness secondary to pain. On physical examination, pain is noted on palpation over the proximal forearm and the pronator muscle may be tender, firm, or enlarged.

Cubital tunnel syndrome is caused by compression of the ulnar nerve at the elbow that manifests as numbness along the small and ulnar half of the ring fingers, often accompanied by weak grip, especially in activities in which torque is applied to a tool. On physical examination, percussion test at the cubital tunnel is positive or symptoms are exacerbated by full flexion of the elbow.

Posterior interosseous nerve palsy is caused by compression of the nerve after the radial nerve bifurcates just proximal to the elbow and involves weakness and pain without a sensory component. Onset is often insidious and may not be noticed by the patient until late weakness of finger and wrist extension or a radial drift of the hand with wrist extension develops.

113
Q

A 28-year-old man comes to the office for evaluation of the nondominant left arm 12 weeks after he sustained a traction injury while working with heavy farm machinery. Physical examination shows complete loss of motor function in the ulnar nerve distribution, loss of sensation at the medial arm and forearm, and mild weakness of the left pectoralis major muscle. Nerve conduction testing of the ulnar nerve shows loss of motor conduction, but sensory conduction is intact. Which of the following is the most likely level of brachial plexus injury?
(A) Anterior cord
(B) Posterior cord
(C) Postganglionic root
(D) Preganglionic root
(E) Terminal branch

A

The correct response is Option D.

This patient has an avulsion injury to the lower roots of the brachial plexus. From proximal to distal, the levels of the brachial plexus are determined by its branching pattern and progress from spinal nerve roots (C5-T1), to trunks, to divisions, to cords, and to terminal branches. Loss of ulnar nerve function indicates damage to the ulnar nerve itself, the medial cord, or the contributing C7, C8, and T1 roots. Loss of medial arm and forearm sensation indicate loss of the medial brachial and medial antebrachial cutaneous nerves, and weakness of the pectoralis muscle indicates loss of the medial pectoral nerve, which innervates the sternocostal head. The lateral pectoral nerve, which is intact in this case, innervates both the clavicular and sternocostal heads and maintains function of the pectoralis major muscle. Loss of motor conductivity with maintenance of sensory conductivity on nerve conduction testing indicates a disruption at the preganglionic nerve root level, with preservation of continuity between the sensory cell bodies in the dorsal root ganglion, the peripheral axons and the peripheral sensory organs, thus preventing Wallerian degeneration of the sensory neurons. The motor neuron cell bodies, however, are located in the ventral horn of the spinal cord, and are separated from their peripheral axons, leading to degeneration and loss of conductivity.

Disruption at the postganglionic nerve root level would result in separation of both sensory and motor cell bodies from their respective peripheral axons and subsequent degeneration. This would lead to loss of both motor and sensory conductivity on nerve conduction testing. Disruptions at the trunk level, cord level, or branch level would also lead to the same results. In addition, loss at the terminal branch level (ulnar nerve in this case), distal to the cord level, would preserve the medial brachial nerve and medial antebrachial nerve, and thus, sensation at the medial arm and forearm.

114
Q

A 62-year-old woman with rheumatoid arthritis comes to the office because she is unable to extend the fingers and thumb of her nondominant left hand. The metacarpophalangeal (MP) joints can be extended passively, but the patient is unable to hold the fingers in extension after they are passively straightened. With passive wrist flexion, the MP joints and proximal interphalangeal (PIP) joints extend. Which of the following is the most likely cause of these findings?
(A) Arthritis of the MP joints
(B) Intrinsic tightness
(C) Posterior interosseous nerve palsy
(D) Rupture of the extensor tendons
(E) Subluxation of the extensor tendons

A

The correct response is Option C.

Rheumatoid arthritis can create several problems related to extensor tendon function.

Posterior interosseous nerve palsy is often the result of inflammation and synovitis surrounding the elbow. In such cases, patients lack active extension of all fingers, but the tenodesis effect would remain in place as the tendons are still in continuity. Subluxation of the extensor tendons usually occurs following the rupture of the radial sagittal band overlying the metacarpal heads. Rupture of the radial sagittal band results in subluxation of the extensor tendons into the metacarpal valleys. This prevents finger extension. If the fingers are extended passively, the extensor tendons will be relocated centrally over the metacarpal head. The patient will be able to maintain extension of the fingers if passively positioned.

Recurrent synovitis at the metacarpophalangeal (MP) joint can cause subluxation of the proximal phalanx with eventual development of destructive arthritis. In such cases, the MP joints are difficult to extend. Extensor tendon rupture is also common and is usually the result of destructive synovitis over the dorsum of the wrist and ulnar head. Such ruptures tend to occur in an ulnar to radial direction, starting with the extensor carpi ulnaris and extensor digiti minimi, and may extend to the first or second compartment tendons in severe cases. Extensor tendon rupture would lead to a lack of tenodesis effect when the wrist is flexed.

Intrinsic tightness results from shortening or contracture of the intrinsic muscles of the hand. This results in extension of the proximal interphalangeal (PIP) joints with attempts at MP extension.

115
Q

A 25-year-old man comes to clinic because he has increasing severe pain in the hand three days after undergoing open reduction of a fracture of the distal radius. Physical examination shows extreme swelling of the hand. Which of the following clinical findings is most likely to confirm the suspected diagnosis of compartment syndrome in the hand of this patient?
(A) Diminished sensation on the dorsum of the hand
(B) Pain on active flexion and extension of the fingers
(C) Pain on passive adduction and abduction of the fingers
(D) Pain on passive extension and flexion of the wrist
(E) Pain on passive flexion and extension of the fingers

A

The correct response is Option C.

The diagnosis of compartment syndrome is primarily a clinical one based on muscle and nerve ischemia. It is possible to measure intracompartment pressure, but the decision to perform a fasciotomy should be based on a high degree of suspicion, close observation of the patient, and clear understanding of the etiology.

Compartment syndrome ischemia may be severe and still not affect the color and temperature of the fingers. The distal pulses are rarely obliterated by compartment swelling, even though muscle and nerve circulation is minimal. The hallmark of muscle and nerve ischemia is pain. The pain is persistent, progressive, and unrelieved by immobilization. Accentuation of the pain by passive muscle stretching is the most reliable clinical test for making the diagnosis of compartment syndrome.

In performing the passive stretch test, one simply stretches the muscle or muscles in the compartment in question, and this should cause severe pain. There are ten compartments of the hand. The intrinsic compartments of the hand are tested by passively abducting and adducting the fingers while keeping the metacarpophalangeal joints in full extension and the proximal interphalangeal joints in flexion. If this maneuver causes a marked increase in pain, the test is diagnostically significant. The adductor compartment to the thumb can be tested by simply pulling the thumb into palmar abduction and thereby stretching the adductor muscle. The thenar muscles are tested by radial abduction of the thumb and the hypothenar muscles by adducting the little finger. Passive or active finger motion can elicit pain seen with other conditions such as flexor tendon infections or injuries, and this is a later and less specific finding.

Diminished sensation is the second most important finding and indicates nerve ischemia as it passes through the affected compartment. The third most important finding is weakness and/or diminished muscle function. Also, palpation of the closed compartments in the hand for tenseness and tenderness should confirm the diagnosis.

116
Q

A 40-year-old construction worker comes to the office because he is unable to extend the left thumb (shown) six months after he sustained a saw injury to the dorsal aspect of the left mid forearm. At the time of the injury, the wound was washed and the skin was repaired in the emergency department. On physical examination, the patient is able to extend the wrist and fingers. No contractures are noted and all joints are supple. Which of the following is the most appropriate management at this time?
(A) Direct repair of the extensor pollicis longus tendon
(B) Transfer of the brachioradialis tendon to the extensor pollicis longus tendon
(C) Transfer of the extensor indicis proprius tendon to the extensor pollicis longus tendon
(D) Tenodesis of the extensor pollicis longus tendon to the extensor digitorum communis of the index finger
(E) Repair of the extensor pollicis longus tendon with intercalated tendon grafting

A

The correct response is Option C.

The extensor indicis proprius (EIP) is an independent extensor tendon to the index finger, which provides overlapping function with the extensor digitorum communis (EDC). The EIP can be transferred to provide independent thumb extension. Proximity of the EIP to the extensor pollicis longus (EPL) in addition to tendon length, which allows for a strong Pulvertaft weave, also favors the EIP-to-EPL tendon transfer. The EIP is located ulnar to the EDC of the index finger at the metacarpophalangeal (MP) joint. Its integrity must be verified before transfer by having the patient extend the index finger while holding the adjacent digits in flexion.

Direct repair with an intercalated tendon graft is not possible because of longstanding contracture, fibrosis, and foreshortening of the proximal muscle belly six months after injury. Therefore, direct repair of the EPL tendon and repair of the EPL tendon with intercalated tendon grafting are incorrect.

Tenodesis of the EPL to EDC of the index finger would improve thumb extension; however, thumb extension would not be independent of finger extension. This makes tenodesis of the EPL tendon to the EDC of the index finger less desirable than EIP-to-EPL tendon transfer.
Brachioradialis-to-EPL tendon transfer would also improve thumb extension; however, range of motion would be lost because the excursion of the brachioradialis is only 1 to 2 cm compared with an excursion of 3 to 5 cm for the EIP. This makes transfer of the brachioradialis tendon to the PEL tendon less desirable than EIP-to-EPL tendon transfer.

117
Q

A 69-year-old woman with type 2 diabetes mellitus is referred to the office because she has compression of the ulnar nerve at the right wrist secondary to an extrinsic burn scar contracture (shown). Physical examination shows weakness of the first dorsal interosseous muscle. Allen test shows perfusion of the hand predominantly by the radial artery. Surgical management includes neurolysis of the ulnar nerve at Guyon’s canal, with identification of the sensory and deep motor branches as shown. Which of the following is the most appropriate technique for coverage of the exposed ulnar nerve?
(A) Adjacent tissue rearrangement of the previously grafted skin
(B) Adipofascial turnover flap with repeat skin grafting
(C) Reverse posterior interosseous fasciocutaneous flap
(D) Reverse radial forearm fasciocutaneous flap
(E) Staged, pedicled groin flap

A

The correct response is Option C.

In this patient who is undergoing ulnar nerve decompression in the distal forearm and wrist, coverage with supple, vascularized tissue will decrease the risk of recurrent contracture and nerve compression.

Adjacent tissue rearrangement of the previously grafted skin will not address the underlying pathophysiology of the scar contracture. An adipofascial turnover flap based on the ulnar artery is not technically possible (because of perforator disruption from the neurolysis), whereas an adipofascial turnover flap based on the radial artery would not be reliable because of the zone of injury in the distal forearm. A staged, pedicled groin flap is relatively contraindicated, given the patient’s age and the potential development of severe stiffness of the hand. A reverse radial forearm flap requires sacrifice of the radial artery, jeopardizing perfusion of the hand.

The photographs shown below depict a reverse posterior interosseus flap. Based on the posterior interosseous vessels between the fifth and sixth dorsal compartments of the forearm, a reverse posterior interosseous flap can support a large skin paddle (18 _ 8 cm), easily reach the distal metacarpals, and include muscle and bone if needed. Donor site morbidity is negligible. In a clinical series of 80 patients who underwent distal coverage of hand and wrist defects with a reverse posterior interosseous flap, only four flaps were unsuccessful due to ischemia.

Free tissue transfer would be an acceptable alternative to a pedicled flap, but this was not listed as a possible option.

118
Q

A 37-year-old man is brought to the emergency department five hours after he sustained the injury to the right forearm (shown) when he was mauled by a dog. Physical examination shows complete transection of the median nerve and radial and ulnar arteries. After revascularization, which of the following is the most appropriate management?
(A) Placement of a vacuum-assisted closure device
(B) Escharotomy
(C) Epimysiotomy
(D) Fasciotomy
(E) Split-thickness skin grafting

A

The correct response is Option D.

The photograph above shows a severe injury of the forearm to the radial and ulnar arteries and the median nerve. The most urgent care needed for this patient is to reestablish the blood supply to the extremity. Muscles can tolerate ischemia for up to four hours, but after six hours, the effect becomes irreversible and tissue loss is greater. With this level of arterial occlusion, restoration of the circulation may produce post-ischemia swelling and initiate a compartment syndrome. Prophylactic fasciotomy is recommended if ischemia time of four to six hours is present before arterial repair. Although compartment pressures can be measured with a commercially available tissue pressure monitor or an infusion technique, these monitoring techniques do not replace the need for fasciotomy with prolonged ischemia. The goal of management is to limit reperfusion injury — not to document compartment pressures. Fasciotomies should be performed in the forearm and over the intrinsic muscles of the hand. The serpentine incision starting from the carpal tunnel running proximally is generally made over the volar forearm with a straight-line incision over the dorsal forearm. Four incisions are generally necessary to release the intrinsic muscles and the thenar and hypothenar muscles. Digital fasciotomies are generally not necessary; however, if there is a significant degree of clinical swelling, fasciotomies are performed along the midaxial line of the fingers and thumb. Epimysiotomy is a procedure during which the muscle covering is stripped from the muscle. This is generally not required. Escharotomy is a procedure during which the skin is incised after a burn injury; it has no role in the treatment of vascular compartment syndrome.

119
Q

A 51-year-old truck driver comes to the hand clinic because he has had constant burning pain in the left arm since he injured the left wrist while loading a truck two weeks ago. Since the injury, avoiding use of the wrist and wearing an immobilization splint on the wrist have not relieved the pain. On physical examination, the left hand, wrist, and forearm are swollen and mildly mottled. The wrist is extremely sensitive to touch. Range of motion of the left wrist and forearm is significantly decreased. Which of the following findings is most likely on additional evaluation of this patient?
(A) Decreased resting sweat output
(B) Decreased temperature readings on thermography
(C) Increased leukocyte count
(D) Increased uptake on a three-phase bone scan
(E) Osteoporosis on plain radiographs

A

The correct response is option D.

This patient’s clinical picture is consistent with early complex regional pain syndrome (CRPS) type I, formerly referred to as reflex sympathetic dystrophy (RSD). A reliable adjunctive tool in confirming the clinical diagnosis is the third phase of a three-phase bone scan. For the scan to be considered indicative of CRPS, the third phase typically shows diffuse increased periarticular activity in the involved joints. Increased uptake is variable in children. In time, results of the three-phase bone scan return to normal.

Radiographic evidence of osteoporosis secondary to CRPS, historically referred to as Sudeck atrophy, does not appear until the third to fifth week of the syndrome. Sudeck atrophy typically demonstrates diffuse osteopenia, juxtacortical demineralization, and subchondral erosions or cysts. This Alag@ in radiographic changes is explained by the fact that calcium content must be altered by 30% to 50% before becoming evident on conventional plain radiographs. The radiographic features of rapid bone loss include visible demineralization with patchy, subchondral, or subperiosteal osteoporosis, metaphyseal banding, and profound bone loss. Despite the osteoporosis, fractures are uncommon.

Other available diagnostic tools include thermography and resting sweat output. Thermography is an indirect method of measuring blood flow to an extremity. Patients with CRPS typically show elevated temperature readings on thermography indicative of increased local blood flow. Resting sweat output quantifies abnormal autonomic system activity by measuring the amount of resting sweat that occurs on the extremity. Patients with CRPS have elevated autonomic system activity and, therefore, demonstrate increased resting sweat output.

Leukocyte count is not helpful in confirming the diagnosis of CRPS.

120
Q

A 22-year-old man comes to the office for initial evaluation 24 months after he sustained an injury to the radial nerve secondary to a fracture of the humerus. He has had no improvement in function of the radial nerve during this time. Which of the following is the most appropriate management of this patient=s nerve injury?

(A) Exploration and repair of the radial nerve at the spiral groove
(B) Free innervated muscle flap transfer
(C) Repair of radial nerve with a nerve graft
(D) Tendon transfers
(E) Vascularized nerve graft reconstruction

A

The correct response is Option D.

The onset of radial nerve palsy in this patient was 24 months ago; therefore, the most appropriate management is tendon transfers to the radial innervated structures that extend the wrist, fingers, and thumb. Wrist extension can be improved with transfer of the pronator teres to the extensor carpi radialis brevis. Finger extension can be improved with transfer of the flexor carpi ulnaris or radialis to the extensor digitorum communis tendons. Thumb extension can be improved with transfer of the palmaris longus to the extensor pollicis longus.

Because the patient has waited so long for treatment, nerve repair is unlikely to achieve recovery of muscle function. After 24 months, the motor end plates atrophy and cannot be reinnervated. When repair of a nerve injury is delayed for an extended period, grafting is usually required for repair. Free innervated muscle flap reconstruction can be considered for necrosis of the flexor compartment in a patient who has had compartment syndrome. Vascularized nerve graft reconstruction is not likely to improve this patient’s hand function.

Twenty-four months after nerve injury, loss at the sensory receptors and motor end plate is severe. Additionally, the time course for recovery of the motor units would require another 6 to 12 months.

121
Q

A 68-year-old man who is undergoing gadolinium-enhanced MRI for evaluation of chronic headaches has the onset of excruciating pain in the forearm after extravasation of approximately 250 mL of gadolinium into the forearm. Which of the following is the most appropriate next step?

(A) CT scan
(B) Measurement of compartment pressures
(C) Plain radiographs
(D) Pulse oximetry
(E) Ultrasonography

A

The correct response is Option B.

This patient who has sustained an extravasation injury during administration of radiologic contrast is at increased risk for compartment syndrome; therefore, the most appropriate next step is measurement of compartment pressures. Low-pressure extravasation injuries have been shown to occur with administration of contrast material. Because most contrast solutions contain nonionic, water-soluble materials, toxicity is typically less of a problem than with high-pressure injection injuries, which typically involve industrial chemicals or chemotherapeutic agents. However, high-osmolarity solutions are associated with an increased risk for local toxicity and may contribute to local tissue necrosis, especially if they are administered using automated high-pressure injection pumps. Because extravasation of more than 100 mL of contrast material has been associated with an increased incidence of compartment syndrome, compartment pressures should be measured, particularly because this patient has excruciating pain, which is the hallmark of compartment syndrome. Decompressive fasciotomy is indicated if the patient is normotensive and has compartment pressures greater than 30 mmHg, or if the patient is hypotensive and has compartment pressures greater than 20 mmHg.

The volar, dorsal, and mobile wad compartments are contained in the forearm; these compartments have been shown to be interconnected (in contrast to the compartments in the lower extremity). Because of these interconnections, release of the volar compartment alone may release the dorsal and mobile wad compartments sufficiently, and fasciotomy of the dorsal and mobile wad compartments may not be necessary. However, this is not an absolute occurrence. Release of the dorsal compartment has also been shown to decrease pressure in the mobile wad compartment. The flexor tendons are contained within the volar compartment. The dorsal compartment contains all of the extensor tendons, with the exception of the extensor carpi radialis brevis and longus, which are contained (along with the brachioradialis tendon) within the mobile wad compartment.

Pulse oximetry is useful for evaluating loss of perfusion, which is a late deficit in patients with compartment syndrome. Ultrasonography, plain radiographs, and CT scans are not helpful in establishing the diagnosis of compartment syndrome and would be unnecessary in any patient with symptoms of compartment syndrome who has increased compartment pressures.

122
Q

A 20-year-old man is undergoing nerve grafting for reconstruction of a 2-cm digital nerve gap. During operative exploration, the distal aspect of the posterior interosseous nerve is identified initially beneath the extensor tendons of the fourth compartment of the forearm. As the nerve is dissected proximally, it courses beneath which of the following muscles?

(A) Brachioradialis
(B) Extensor digitorum communis
(C) Extensor digitorum quinti
(D) Extensor indicis proprius
(E) Extensor pollicis longus

A

The correct response is Option E.

The terminal branch of the posterior interosseous nerve is a good source of nerve for grafting of small distal gaps of the digital nerves because it leaves no cutaneous sensory loss at the donor site. This nerve branch can be found at the floor of the fourth compartment, beneath the extensor digitorum communis and extensor indicis proprius muscles, but travels more proximally beneath the extensor pollicis longus muscle. The posterior interosseous nerve does not traverse deep to the extensor digital quinti. The superficial radial nerve travels beneath the brachioradialis muscle and emerges at the wrist.

123
Q

A patient has complete C5 tetraplegia after sustaining a spinal cord injury. During reconstructive procedures in this patient, restoration of which of the following is most useful functionally?

(A) Digit extension
(B) Digit flexion
(C) Elbow extension
(D) Key pinch
(E) Wrist extension

A

The correct response is Option C.

Reconstruction after spinal cord injury is based on the level of injury and the muscles that are available for transfer. Because many spinal cord injuries are not complete, testing of individual muscles is essential to determine the most appropriate management. The International Classification for Surgery of the Hand in Tetraplegia, a 10-point scale ranging from 0 to 9, is the most widely used tool to determine functional reconstruction. In a patient with a complete C5 tetraplegia, the Group 0 designation indicates that no muscles suitable for transfer below the elbow are available. In patients with this classification, the most useful reconstruction involves restoration of the elbow extension. Elbow flexion and extension, along with shoulder strength, can enable the tetraplegic patient to propel a wheelchair, to transfer from bed, and to shift the body weight to prevent pressure ulcers. The most commonly prescribed transfers are the deltoid-to-triceps transfer and the biceps-to-triceps transfer.
Reconstruction procedures that restore digital extension and flexion, key pinch, and wrist extension are useful in patients with lower-level spinal cord injury.

124
Q

During release of the ulnar nerve at the elbow, which of the following structures is the most common site of ulnar nerve compression?

(A) Arcade of Frohse
(B) Arcade of Struthers
(C) Ligament of Guyon
(D) Ligament of Struthers
(E) Osborne’s ligament

A

The correct response is Option E.

Compression of the ulnar nerve at the elbow is the second most common compressive neuropathy of the upper extremity, after carpal tunnel syndrome. This condition is characterized by paresthesia in the ring and small fingers. A variety of anatomic structures can compress the ulnar nerve. The site can range from 8 cm proximal to the epicondyle to 8 cm distal. Structures that can cause ulnar nerve compression, moving proximally to distally, are the arcade of Struthers, anconeus epitrochlearis, medial head of the triceps, fibro-osseous cubital tunnel (Osborne’s ligament), and aponeurosis of the flexor carpi ulnaris. Of these structures, the most common cause of nerve compression is Osborne=s ligament.

The arcade of Frohse can compress the radial nerve. The ligament of Struthers causes compression of the median nerve at a point proximal to the elbow but does not affect the ulnar nerve. The ligament of Guyon involves the ulnar nerve at the level of the wrist. Ulnar compression of the wrist is far less common than ulnar nerve compression at the elbow.

125
Q

A 32-year-old man is brought to the emergency department after sustaining a crush injury to the right forearm when the arm became caught in a machine. On physical examination, there is severe tenseness over the arm. The patient has pain in the forearm with passive motion of the fingers. Radiographs show no evidence of fracture. Doppler ultrasonography shows weak signals over the radial artery. Which of the following is the most appropriate management?

(A) Angiography and infusion of tissue plasminogen activator
(B) Escharotomy in the emergency department
(C) Fasciotomy under general anesthesia
(D) Hyperbaric oxygen therapy
(E) Observation and intravenous administration of fluids

A

The correct response is Option C.

The most appropriate management of this patient is fasciotomy under general anesthesia. These findings are consistent with compartment syndrome, which is compromise of circulation and tissue perfusion resulting from increased pressure within functional compartments. This condition is characterized by pain, pallor, paresthesia, and pulselessness. Because ischemia typically becomes irreversible after eight hours, treatment should not be deferred until these findings are noted because they often occur late. Compartment pressures can be measured using a hand-held pressure-monitoring device, calibrated manometer, or blood pressure cuff. Various incisions can be used for the fasciotomy procedure, but the carpal tunnel must be released, and three compartments of the forearm (volar compartment, dorsal compartment, and mobile wad) must be released superficially and deeply.

Because this patient has findings consistent with compartment syndrome, surgery should be performed urgently to prevent or minimize the complications resulting from increasing compartment pressures. Prolonged ischemia can lead to muscle necrosis and nerve injury. In the alert, competent patient, the diagnosis of compartment syndrome can be made on clinical examination alone. Pain accentuated by passive stretching of the involved muscle compartment is the hallmark of this condition. Pain on passive motion is the most consistent early sign. Swelling or tenseness of the affected compartments may also be noted.

In patients who have sustained head trauma or spinal cord injuries, compartment pressures may need to be measured. Compartment pressures greater than 30 mmHg, or greater than 20 mmHg in a patient with hypotension, are diagnostic. Distal pulses and perfusion are typically unaffected soon after injury because the radial arteries pass adjacent to and between the muscle compartments. Sensation may be normal or diminished. Diminished neurovascular status is often a late finding.

The most appropriate next step is fasciotomy to decompress the involved muscle compartments. Because collateral circulation is retained to a greater extent in the muscle periphery, profound ischemia is more likely to be noted in the central portion of the muscle belly. The flexor digitorum profundus, flexor pollicis longus, flexor digitorum superficialis, and pronator teres will be affected most significantly, and the wrist flexors and extensors and the brachioradialis muscle usually sustain more moderate damage.

Observation and intravenous administration of fluids are inappropriate in a patient with suspected compartment syndrome. Any delay in decompression will only worsen tissue necrosis. Hyperbaric oxygen therapy has been shown to be beneficial in patients with compartment syndrome but does not replace fasciotomy as the treatment of choice. Angiography and infusion of tissue plasminogen activator (TPA) are indicated in patients with arterial thrombosis or embolization. Escharotomies are typically performed in the emergency department for removal of circumferential eschar in burn patients.

126
Q

A 27-year-old nurse has a 2-cm digital nerve gap after sustaining a crush injury to the index finger. A graft from the medial antebrachial cutaneous nerve is to be used for nerve repair. This nerve can be found adjacent to which of the following structures?

(A) Anconeus muscle
(B) Basilic vein
(C) Brachioradialis muscle
(D) Cephalic vein
(E) Median nerve

A

The correct response is Option B.

The medial antebrachial cutaneous nerve can be found adjacent to the basilic vein. In order to expose the nerve, the surgeon should make a longitudinal incision at a point 2 cm anterior and 3 cm distal to the medial epicondyle. The anterior and posterior branches can each be used for nerve grafting.

The anconeus muscle can be found in the area of the radial nerve at the posterolateral aspect of the elbow. The brachioradialis muscle is located on the lateral aspect of the forearm near the superficial radial nerve. The lateral antebrachial cutaneous nerve is a continuation of the musculocutaneous nerve. It can be identified lateral to the biceps tendon and adjacent to the cephalic vein. The median nerve lies deep to the flexor digitorum superficialis tendon and is not in the region of the cutaneous nerves of the forearm. The sural nerve, which is another donor nerve graft option, has become the standard source for large defects, particularly of the median or ulnar nerves, and can be found posterior to the lateral malleolus. The terminal branches of the anterior and posterior interosseous nerves can also be harvested for grafting of smaller defects. The superficial radial nerve and saphenous nerve in the lower extremity may also be considered.

Alternatives to nerve grafting include the use of vein and muscle autograft or artificial conduits composed of resorbable tubing, as well as nerve allografting with immunosuppression.

127
Q

In a patient who has sustained a crush injury to the upper extremity, what is the minimum number of incisions required to adequately decompress the intrinsic muscles?

(A) 2
(B) 3
(C) 4
(D) 5
(E) 6

A

The correct response is Option C.

Patients who sustain crush injuries to the upper extremity are at risk for compartment syndrome, a limb-threatening condition that can also be potentially life-threatening. Compartment syndrome can be caused by external forces, such as pressure on the extremity, or internal injuries, such as hematoma, infection, burn injury, or edema that develops after revascularization. This condition must be diagnosed promptly and the affected muscle compartments must be released immediately to maintain function of the affected extremity.

There are nine muscle compartments in the upper extremity. These include the deltoid compartment, the anterior and posterior brachial compartments, the mobile wad of the forearm, the volar and dorsal forearm compartments, the thenar and hypothenar compartments, and the interosseous compartment of the hand. Although compartment syndrome most commonly affects the forearm, the compartments in the hand must also be evaluated.

In the treatment of compartment syndrome of the hand, management includes release of the following compartments:

  1. The thenar compartment: this compartment contains the abductor pollicis brevis, flexor pollicis brevis, and opponens pollicis muscles, and is released with a single radial incision over the thenar muscles.
  2. The hypothenar compartment: this compartment, composed of the abductor digiti minimi, flexor digiti minimi, and opponens digiti minimi muscles, is released by performing a single ulnar incision over the thenar muscles.
  3. The interosseous compartment of the hand: the dorsal and palmar interossei are contained within the interosseous compartment of the hand. Two incisions, dorsal to the index and ring finger metacarpals, are required to adequately decompress the compartments adjacent to the second and fourth metacarpals.

Therefore, a minimum of four incisions is necessary to release all compartments of the hand. Performing fewer than four incisions will result in inadequate release of the affected compartments.

128
Q

Risk of development of complex regional pain syndrome is increased by injury to each of the following structures EXCEPT

(A) abdomen
(B) brain
(C) peripheral nerves
(D) spinal cord
(E) upper extremities

A

The correct response is Option A.

In patients with complex regional pain syndrome, an initial noxious or painful event results in pain that is disproportionate to the initial event and cannot be linked to any specific pathologic process. Features of the condition include spontaneous pain, allodynia, and hyperesthesia that cannot be accounted for by injury to a single peripheral nerve, edema, changes to the skin or blood flow, or any other condition. Conditions that place patients at risk for complex regional pain syndrome include fractures of the distal radius as well as injuries of the upper extremities, spinal cord, brain, and peripheral nerves.

129
Q

A 34-year-old man has radial nerve palsy six months after undergoing open reduction and plate fixation of a fracture of the humeral diaphysis. The integrity of the radial nerve was confirmed at the time of the initial injury. Which of the following is the most appropriate next step in management?

(A) Dynamic extension splinting
(B) Injection of a corticosteroid
(C) Tendon transfers
(D) Interpositional nerve grafting
(E) Neurolysis

A

The correct response is Option C.

In patients with high radial nerve palsy, the primary goal is restoration of extension of the wrist, fingers, and thumb. If the nerve was intact at the time of the initial surgery and there is subsequently no return of function six months later, further improvement is unlikely, and tendon transfers are indicated at this time.

Transfer of the pronator teres to the extensor carpi radialis brevis is frequently performed to recover wrist extension. To regain finger extension, the flexor carpi radialis, flexor carpi ulnaris, or flexor digitorum superficialis of the long or ring fingers is transferred into the distal extensor digitorum communis tendons. Transfers to regain thumb extension include the palmaris longus or flexor carpi radialis to the extensor pollicis longus. Additionally, some surgeons advocate end-to-side tendon transfers.

The extensor carpi radialis brevis cannot be transferred to the extensor digitorum communis because it is also affected by the radial nerve palsy. Transfer of the flexor digitorum profundus tendon of the long finger is associated with significant loss of function at the donor site.

Splinting is not indicated in a patient who has persistent radial nerve palsy six months after the initial procedure.

Injection of a corticosteroid is inappropriate treatment of radial nerve palsy.

Because the nerve is shown to be intact, the presence of a neuroma in continuity may be inhibiting the reinnervation process. EMG should be performed to determine the potential for nerve grafting or neurolysis in this patient. However, most nerve grafting procedures in adults provide only limited improvement in motor and sensory functions six months after denervation.

130
Q

A 23-year-old man sustained a complete laceration of the ulnar nerve at the level of the elbow four weeks ago when he was stabbed in the nondominant forearm with a knife. After surgical nerve preparation, a 2-cm gap is present. Which of the following is the most appropriate next step in management?

(A) Use of a vein conduit
(B) Tendon transfers
(C) Mobilization of the nerve 15 cm proximally and distally
(D) Sural nerve grafting
(E) Ulnar nerve transposition

A

The correct response is Option E.

In this 23-year-old man who has a 2-cm nerve gap after sustaining a clean laceration of the ulnar nerve at the elbow four weeks ago, the most appropriate next step in management is transposition of the ulnar nerve. Because transposition of this nerve at the elbow provides as much as 4 cm of length, it is recommended in this patient in whom primary coaptation cannot be performed because of nerve retraction.

Ulnar nerve transposition may yield as much as 3 cm of length when performed in the arm, 2 cm of length at the forearm, and 1 to 2 cm of length at the distal forearm and wrist. Transposition is also appropriate for repair of median and high radial nerve injuries.

Vein conduits should only be considered if direct repair and transposition are not options.

Tendon transfers alone do not restore sensory function, and are recommended for late reconstruction only when nerve repair is no longer an option.

Extensive mobilization of the ulnar nerve into the mid forearm may cause devascularization and injury to distal nerve branches and ultimately worsen functional outcome.

Sural nerve grafting may be considered if transposition of the ulnar nerve results in tension following nerve coaptation.

131
Q

In a patient with established Volkmann’s ischemic contracture, reconstruction via free transfer of the gracilis involves decompression of which of the following nerves?

(A) Anterior interosseous
(B) Median
(C) Posterior interosseous
(D) Radial
(E) Ulnar

A

The correct response is Option B.

In a patient with Volkmann’s ischemic contracture of the forearm, the median nerve is most likely compressed, and is thus most likely to require decompression, because it traverses the center of the scarred muscles. After muscle infarction occurs, peripheral nerves can become compressed within a constricting scar at a specific anatomic location. Because improvement in nerve function is related to the severity and duration of compression, early decompression is required to minimize further dysfunction. As a result, the median nerve should be decompressed as soon as the patient’s condition permits. In order to be successful, operative exploration is necessary at all points of compression.

In patients with compartment syndrome, high interstitial pressures are typically measured in the deepest compartments of the forearm, especially those compartments that lie adjacent to bone. The flexor digitorum profundus and flexor pollicis longus muscles are affected most often; in more severe cases, the flexor digitorum superficialis, flexor carpi radialis, and flexor carpi ulnaris are involved also. The median nerve often lies at the center of the a constricting scar in the forearm and is thus at risk for compression at the lacertus fibrosus, the two heads of the pronator teres, and the proximal arch of the flexor digitorum superficialis, as well as within the carpal tunnel.

The anterior interosseous nerve is a branch of the median nerve and thus is not involved in Volkmann’s ischemic contracture of the forearm. The posterior interosseous nerve and the radial nerve are rarely involved. The ulnar nerve is implicated less frequently than the median nerve and is often compressed at or just distal to the elbow.

132
Q

A 15-year-old girl has absence of sensation of the long finger and radial side of the ring finger 18 months after undergoing operative repair of a partial injury to the median nerve in the wrist. Sensation is normal in the thumb, index, and small fingers and in the ulnar side of the ring finger. An intraoperative photograph is shown above.

Which of the following is the most appropriate management?

(A) Internal neurolysis of the median nerve
(B) Excision of the neuroma only
(C) Excision of the neuroma and epineural repair
(D) Excision of the neuroma and sural nerve grafting
(E) Resection of the median nerve and epineural repair

A

The correct response is Option D.

In this patient who has a neuroma in-continuity, the most appropriate management is excision followed by sural nerve grafting. Neuroma in-continuity is often difficult to diagnose and treat. Serial clinical examination and electrodiagnostic testing are essential for diagnosis. Although operative exploration can improve hand function and result in a good outcome, the functional fascicles that lie adjacent to the neuroma are at risk for injury. Nerve conduction velocity studies should be performed intraoperatively to identify the nonfunctioning fascicles that lead into
and out of the neuroma. The surgeon should take great care during excision of the neuroma to avoid damaging the functional fascicles. Following excision, autogenous grafting with a donor nerve such as the sural nerve should be performed.

Internal neurolysis would not re-establish the continuity of the involved fascicles. Simple excision of the neuroma will result in recurrence. Excision and epineural repair would place excessive tension on the neurorrhaphy and potentially lead to the development of another neuroma. Resection of the median nerve is an excessive procedure that would eliminate the functional portion of the nerve.

133
Q

A 50-year-old man is undergoing evaluation because he has had weakness of grip in the right hand for the past two months that is affecting his golf swing. Physical examination shows limited abduction and adduction of the fingers of the right hand. He has difficulty crossing the fingers. Sensation is diminished over the volar aspect of the small finger and volar-ulnar aspect of the ring finger. Sensation is normal over the radial digits and dorsal aspect of the hand.

This patient most likely has an nerve lesion at which of the following sites?

(A) Arcade of Frohse
(B) Arcade of Struthers
(C) Guyon’s canal
(D) Origin of the flexor carpi ulnaris
(E) Vascular leash of Henry

A

The correct response is Option C.

This patient has symptoms consistent with ulnar nerve compression, including weakness of the intrinsic muscles of the hand innervated by the ulnar nerve (which manifests as a loss of finger adduction and abduction) and evidence of muscle atrophy. There is also decreased sensation in the distribution of the ulnar nerve. The ulnar nerve is compressed most commonly in the region of the cubital tunnel; the second most common site of compression is at Guyon’s canal in the wrist. Sensation over the dorsoulnar hand, which is supplied by the dorsal branches from the ulnar nerve arising proximal to Guyon’s canal, is tested to determine the location of compression. If sensation is altered dorsally, the lesion lies proximal to the distal forearm, and is most likely to involve the cubital tunnel. However, if sensation in the dorsoulnar hand is normal, the lesion lies distal to the distal forearm, and most likely involves Guyon’s canal, as in this patient.

Other, less common sites of compression of the ulnar nerve include the arcade of Struthers, which is a thin aponeurotic band extending from medial head of the triceps to the medial intermuscular septum, located approximately 8 cm proximal to the medial epicondyle, and occasionally the origin of the flexor carpi ulnaris. The arcade of Frohse and vascular leash of Henry are potential sites of compression of the radial nerve.

When compressed, peripheral nerves typically cause pain and specific nerve deficits of sensation and strength. Some sensory branches divide from the nerve proximal to the wrist or at the level of the carpal canal. The dorsal sensory branch of the ulnar nerve divides approximately 6 cm proximal to the wrist. The motor branches of the median and ulnar nerves can be separated into extrinsic and intrinsic function.

134
Q

A 27-year-old woman has intense, burning pain in the right arm 10 days after sustaining a fracture of the right radius. Conservative treatment of the pain with oral administration of narcotic agents has not been effective. At the time of injury, a long arm cast was applied in the emergency department.

Which of the following is the most appropriate diagnostic test?

(A) Three-view radiographs of the wrist
(B) Stellate ganglion block
(C) Thermography of the upper extremity
(D) Triple-phase bone scan
(E) MRI of the wrist

A

The correct response is Option A.

This patient has symptoms consistent with complex regional pain syndrome type I, or reflex sympathetic dystrophy (RSD), a complex alteration of the pain response following trauma. In contrast, patients with complex regional pain syndrome type II, or causalgia, have posttraumatic pain resulting from an identifiable nerve injury. While its exact cause is unknown, RSD is characterized by pain, stiffness, limited function, atrophic changes, and vasomotor instability. Early diagnosis and treatment are essential for optimal functional outcome; the surgeon must also differentiate RSD from other, treatable conditions.

Although fractures of the distal radius are a common precipitating factor for RSD, the pain may actually be caused by fracture nonunion or an excessively tight cast. Therefore, appropriate initial management involves removing the cast and obtaining three-view radiographs of the wrist to determine the adequacy of fracture reduction. This should be performed before any of the other diagnostic tests listed.

135
Q

A neonate has C5-6 brachial plexus palsy at birth. Complete recovery of function is most likely in this patient if some activity is demonstrated in the deltoid and biceps muscles by how many months of age?

(A) 2
(B) 4
(C) 6
(D) 8
(E) 12

A

The correct response is Option A.

In neonates who have brachial plexus palsy at birth, neurologic outcome correlates directly with the severity of the brachial plexus injury. The evaluation and management of brachial plexus palsy remains somewhat controversial. A recognition of the natural history of this condition, along with more uniform evaluation procedures, has resulted in better care of affected neonates. One author recommends using the rate of biceps recovery as an indication for early operative exploration of the brachial plexus. However, the course of biceps recovery over time has been shown to incorrectly predict outcome in 13% of patients. A more complete evaluation of elbow flexion and extension of the wrist, finger, and thumb seems to be more predictive of outcome.

Nevertheless, those infants with C5-6 injuries who begin to exhibit some function of the deltoid and biceps muscles by age 2 months will most likely have normal arm function. In contrast, infants who do not exhibit strong biceps contractions by age 6 months will likely not ultimately attain completely normal arm function.

136
Q

A 40-year-old man has marked swelling of the entire left hand and severe pain with passive movement of the fingers six hours after sustaining a crush injury to the left hand and forearm. Radiographs show no evidence of fracture. Which of the following incisions is most appropriate for release of the adductor pollicis and associated interossei?

(A) Dorsal second metacarpal
(B) Radial edge of the first metacarpal
(C) Thenar crease
(D) Third metacarpal
(E) Transverse palmar

A

The correct response is Option A.

Signs of compartment syndrome include pain accentuated by passive muscle stretch, paresthesia, weakness, and swelling, tenderness, and tenseness of the compartments of the forearm. Pallor and pulselessness are rare, late
findings. In the alert, competent patient, pain accentuated by passive stretching of the involved muscle compartment is the most consistent early sign and is considered to be diagnostic. In patients with signs of compartment syndrome, the four dorsal compartments, three volar interossei compartments, adductor pollicis compartment, thenar eminence compartment, and hypothenar eminence compartment all need to be released.

Fasciotomies of the intrinsic muscles of the hand can be performed through four incisions to decompress all ten compartments. All of the interossei and the adductor muscles can be released through two dorsal longitudinal incisions made over the second and fourth metacarpals. The hypothenar and thenar eminences are released through incisions over the ulnar fifth metacarpal and radial first metacarpal, respectively. Palmar incisions are not performed.

137
Q

A 38-year-old woman has had a clenched-fist deformity involving the right hand for the past 18 months. She sustained an anoxic brain injury 14 years ago. Physical examination shows maceration and wounding of the palmar skin caused by the fingernails. The patient has no voluntary motor control of the arm, and passive motion produces pain in the joints and flexor tendons.

Which of the following is the most appropriate initial management?

(A) Application of a cast with the arm in an intrinsic-plus position
(B) Injection of botulinum toxin (Botox) into the profundus and superficialis muscles
(C) Release of the profundus and superficialis tendons
(D) Transfer of the superficialis tendon to the profundus tendon
(E) Z-plasty for lengthening of the profundus and superficialis tendons

A

The correct response is Option B.

Because reconstructive surgery is especially difficult in patients who have spasticity following brain injury, operative procedures should be performed only if conservative treatment is not successful. Injection of botulinum toxin will provide temporary relief of spasticity and facilitate splinting or physical therapy.

If these measures fail, muscle release or transfer of the superficialis tendon to the profundus tendon may be considered in this patient. Tendon transfer would correct the extensor tendon imbalance but would result in hygienic, rather than functional, improvement in a patient with no voluntary motor control.

Casting is not indicated in poorly compliant patients who have severe contractures and painful joints. Although release of the superficialis and profundus tendons would relieve the flexion contractures, it is most likely to result in unopposed action of the extensor tendons. Z-plasty lengthening will not adequately release the tendons in a patient with a clenched-fist deformity because the muscles are severely contracted.

138
Q

A 38-year-old woman has had a clenched-fist deformity involving the right hand for the past 18 months. She sustained an anoxic brain injury 14 years ago. Physical examination shows maceration and wounding of the palmar skin caused by the fingernails. The patient has no voluntary motor control of the arm, and passive motion produces pain in the joints and flexor tendons.

Which of the following is the most appropriate initial management?

(A) Application of a cast with the arm in an intrinsic-plus position
(B) Injection of botulinum toxin (Botox) into the profundus and superficialis muscles
(C) Release of the profundus and superficialis tendons
(D) Transfer of the superficialis tendon to the profundus tendon
(E) Z-plasty for lengthening of the profundus and superficialis tendons

A

The correct response is Option B.

Because reconstructive surgery is especially difficult in patients who have spasticity following brain injury, operative procedures should be performed only if conservative treatment is not successful. Injection of botulinum toxin will provide temporary relief of spasticity and facilitate splinting or physical therapy.

If these measures fail, muscle release or transfer of the superficialis tendon to the profundus tendon may be considered in this patient. Tendon transfer would correct the extensor tendon imbalance but would result in hygienic, rather than functional, improvement in a patient with no voluntary motor control.

Casting is not indicated in poorly compliant patients who have severe contractures and painful joints. Although release of the superficialis and profundus tendons would relieve the flexion contractures, it is most likely to result in unopposed action of the extensor tendons. Z-plasty lengthening will not adequately release the tendons in a patient with a clenched-fist deformity because the muscles are severely contracted.

139
Q

A 36-year-old man has pain in the forearm and paresthesia in the hand that are exacerbated with activity. He also has decreased sensation and paresthesia in the radial side of the palm, at the base of the thenar eminence, in the thumb, index, and long fingers, and along the radial side of the ring finger.

These findings are most consistent with which of the following syndromes?

(A) Anterior interosseus syndrome
(B) Carpal tunnel syndrome
(C) Cubital tunnel syndrome
(D) Pronator syndrome
(E) Radial tunnel syndrome

A

The correct response is Option D.

This 36-year-old man has findings consistent with pronator syndrome, or proximal compression neuropathy of the median nerve. The median nerve may be entrapped beneath the supracondylar process and ligament of Struthers in the distal third of the humerus or at the lacertus fibrosis, the pronator teres muscle, or the arch of the flexor digitorum superficialis muscle.

Functional testing can be used to determine the site of compression. Compression at the ligament of Struthers is indicated by exacerbation of symptoms with flexion of the elbow against resistance. In patients who have compression of the median nerve at the lacertus fibrosis, symptoms are exacerbated by active flexion of the elbow with the forearm in pronation. Patients who have pain with resisted pronation of the forearm during wrist flexion should undergo surgical exploration of the median nerve where it passes through the pronator teres muscle. However, if resisted flexion of the superficialis muscle of the long finger exacerbates symptoms, the superficialis arch should be inspected carefully during surgical exploration.

Electrodiagnostic studies can be used to confirm the diagnosis and determine the level and severity of nerve injury.

Release of the median nerve is appropriate management. During the procedure, the median nerve should be explored starting at a point 5 cm proximal to the elbow and continuing distally. Each of the potential sites of compression should be carefully divided to ensure that the nerve is adequately released.

Patients with anterior interosseous syndrome have poorly defined pain in the proximal forearm that is relieved with rest. They also have weakness or paralysis of the flexor digitorum profundus tendon of the index and long fingers, flexor pollicis longus tendon, and pronator quadratus muscle.

Carpal tunnel syndrome is the most common compression neuropathy of the upper extremity. It is characterized by pain, especially at night, and numbness and weakness in the distribution of the median nerve at the wrist. Numbness in the region innervated by the palmar cutaneous nerve is uncommon.

Cubital tunnel syndrome, or entrapment of the ulnar nerve at the elbow, manifests as pain over the medial aspect of the proximal forearm. Numbness is noted in the dorsoulnar aspect of the hand, small finger, and ulnar aspect of the ring finger. Weakness can be demonstrated in the flexor digitorum profundus tendons of the ring and small fingers and in the ulnar intrinsic tendons, especially the first dorsal interosseous and abductor digiti minimi tendons.

Patients with radial tunnel syndrome have aching pain localized just below the elbow in the extensor mass and along the course of the radial nerve. Although there are four potential sites of compression within the radial tunnel, most patients have entrapment at the arcade of Frohse, which forms a ligamentous band over the deep radial nerve as the nerve enters the supinator muscle. Radial tunnel syndrome manifests as pain rather than specific numbness or weakness.

140
Q

A right-handed 34-year-old man has been unable to flex the distal interphalangeal joint of the right index finger and interphalangeal joint of the thumb for the past six months. There is no history of trauma. Physical examination shows normal two-point discrimination of the right hand; the muscles innervated by the ulnar nerve are unaffected.

Which of the following is the most likely cause of these symptoms?

(A) Anterior interosseus syndrome
(B) Cubital tunnel syndrome
(C) Posterior interosseous syndrome
(D) Pronator syndrome
(E) Radial tunnel syndrome

A

The correct response is Option A.

The most likely diagnosis is anterior interosseous syndrome (also known as Kiloh-Nevin syndrome). This condition is characterized by absence of function of motor units innervated by the anterior interosseous nerve. These motor units include the flexor pollicis longus tendon, the profundus tendons to the index and long fingers, and the pronator quadratus muscle. In affected patients, there is absence of flexion of the interphalangeal joint of the thumb and distal interphalangeal joints of the index and long fingers and weakness of pronation with the elbow in flexion. Anterior interosseous syndrome results in pure motor deficits; there are no sensory abnormalities.

In patients with this condition, the anterior interosseus nerve may be compressed within the tendinous bands (the deep head of the pronator teres tendon, origin of the flexor digitorum superficialis tendon of the ring finger, and origin of the flexor carpi ulnaris tendon) or accessory muscles (anomalous band connecting the flexor digitorum superficialis tendon to the flexor digitorum profundus, accessory flexor pollicis longus, and palmaris profundus tendons). Other causes include vascular anomalies (thrombosis of ulnar collateral vessels or an aberrant radial artery), bicipital bursa, trauma, fractures, or compression resulting from intravenous devices.

Cubital tunnel syndrome, or compression of the ulnar nerve at the elbow, results in sensory deficits of the ring and small fingers and the dorsoulnar aspect of the hand and motor deficits in the intrinsic and extrinsic muscles innervated by the ulnar nerve. Posterior interosseous syndrome is similar to anterior interosseous syndrome in that it also causes pure motor deficits without sensory findings. Affected patients have weakness in the muscles of the wrist and extensors of the thumb and fingers. Compression of the median nerve by the pronator muscle is unlikely because this patient does not have changes in sensation. Compression of the radial nerve in the radial tunnel primarily results in pain in the distribution of the radial nerve.

141
Q

A 48-year-old dock worker has effort-associated carpal tunnel syndrome. Which of the following muscles is the most likely cause?

(A) Abductor digiti quinti
(B) Adductor pollicis
(C) Lumbrical
(D) Palmaris brevis
(E) Third volar interosseus

A

The correct response is Option C.

The lumbrical muscles, which aid in flexion of the metacarpophalangeal joints, typically originate from the radial side of the flexor digitorum profundus tendons, with the exception of the third lumbrical, which originates from the ulnar side of the long finger and radial side of the ring finger. These muscles insert into the radial sagittal band and also assist in extension of the interphalangeal joints. Because the lumbrical muscles rest within the carpal canal during grip functions and can become edematous with prolonged use, carpal tunnel syndrome may be aggravated in patients with muscular hands.

Each lumbrical muscle is innervated by the same source as its flexor digitorum profundus tendon of origin. The median nerve supplies innervation to the first and second lumbricals, and the ulnar nerve supplies the third and fourth lumbricals.

None of the other intrinsic muscles pass within the carpal canal.

142
Q

An 8-year-old boy sustains a near complete amputation through the midportion of the nondominant left arm. Examination shows a significantly comminuted fracture of the humerus. On intraoperative exploration, the median and radial nerves are transected and retracted. The proximal and distal ends of each nerve are visualized; however, following debridement of the affected nerve areas and mobilization of the nerves, there is a 5-cm gap between the nerve ends.

Following reestablishment of arterial and venous flow, which of the following is the most appropriate management of the nerve injuries?

(A) Delayed reconstruction following healing and stabilization of the humerus fracture
(B) Use of absorbable polyglycolic acid conduits to bridge the gap between nerve endings
(C) Sural nerve cable grafting to bridge the gap between nerve endings
(D) Transfer of the ipsilateral intercostal nerves to the distal ends of the radial and median nerves
(E) Humeral shortening with primary repair of the proximal and distal nerve ends

A

The correct response is Option E.

Following reestablishment of arterial and venous flow, the most appropriate next step is humeral shortening with primary repair of the proximal and distal nerve ends. Nerve repair should ideally be performed as a primary end-to-end repair without tension; in patients with bone comminution, the bone can be shortened within limits to alleviate tension. Shoulder abduction will increase median nerve length by 2.5 cm and ulnar nerve length by 2 cm; elbow flexion will increase the length of both the median and ulnar nerves by an additional 4 cm.

Delayed reconstruction will ultimately be more difficult because the injured nerves will have become retracted and scarred, and mobilization will be limited.

Absorbable polyglycolic acid conduits have been used successfully in bridging digital nerve gaps of 4 mm or less. However, success has not been reported with the use of this material for repair of major peripheral nerves.

Non-interfascicular nerve cable grafts do not optimize fascicle-to-fascicle opposition and thus provide poor results; interfascicular nerve grafts are now used instead. Results seen with nerve grafting are typically less satisfactory than primary repair, especially in older patients. Interfascicular sural nerve grafts can be used in patients in whom humeral shortening is not an option.

Nerve transfer is inappropriate in this patient because the proximal, median, and radial nerves can be repaired instead. Likewise, a neurotization procedure, which involves embedding of the distal end of a nerve in continuity with the spinal cord directly into a recipient muscle, is also not indicated because the distal nerve ends are available for use.

143
Q

A 15-year-old boy has a 1-cm defect of the ulnar nerve after sustaining a laceration of the nerve at the distal wrist crease, just proximal to Guyon’s canal. During dissection of the ulnar nerve at the wrist in preparation for nerve repair, the motor fascicular group can be identified at which of the following sites?

(A) Interwoven with the sensory group
(B) Radial and dorsal to the sensory group
(C) Radial and palmar to the sensory group
(D) Ulnar and dorsal to the sensory group
(E) Ulnar and palmar to the sensory group

A

The correct response is Option D.

During dissection of the ulnar nerve at the wrist in preparation for nerve repair, the motor fascicular group can be identified ulnar and dorsal to the sensory group at the wrist. The motor fascicles in the arm are consistently found between the sensory fascicles to the dorsal sensory nerve branch and the ulnar sensory fascicles to the ring and small fingers. Beyond the dorsal cutaneous nerve branch, the fascicles lie ulnar and slightly dorsal to the sensory fascicular group and then pass dorsal and radial to the cutaneous fascicular group in Guyon’s canal.

144
Q

A 65-year-old man with tetraplegia to the level of C5-6 has the forearm supination deformity shown in the photograph above. On examination, the supination deformity can be passively corrected. In order to relieve the deformity and improve arm function, which of the following is the most appropriate management?

(A) Release of the biceps tendon
(B) Redirecting the brachioradialis tendon into the flexor carpi ulnaris tendon
(C) Redirecting the biceps tendon through the interosseous membrane
(D) Transfer of the triceps tendon to the biceps tendon
(E) Transfer of the biceps tendon to the brachialis tendon

A

The correct response is Option C.

In order to permanently correct this patient’s supination deformity and improve arm function, the biceps tendon should be redirected through the interosseous membrane. The supination deformity shown in the photograph is common to patients who have C5-6 tetraplegia, but the resting hand position results in loss of function and a displeasing aesthetic appearance. Functional improvement can be achieved by performing a tendon transfer to place the hand in a pronated position (eg, a palm-down position on a table top). This will improve the current aesthetic appearance and allow further tendon transfers to potentially establish key pinch. Because the biceps tendon is the strongest supinator in the forearm, it can be redirected through the interosseous membrane, then reattached to itself to establish forearm pronation. Redirecting the tendon insertion will convert a supinator tendon into a pronator tendon.

Release of the biceps tendon will impair elbow flexion and further weaken upper extremity strength. The brachioradialis tendon, which is the strongest elbow flexor, inserts into the ulna and does not influence supination and pronation in the forearm. Transferring the triceps to the biceps or the biceps to the brachioradialis will not correct the supination deformity.

145
Q

An otherwise healthy 64-year-old man is unable to extend the left wrist and fingers 24 hours after undergoing release of Dupuytren’s contractures of the ring and small fingers. Anesthesia for the procedure was provided using a brachial plexus block with 1% lidocaine and inflation of the pneumatic tourniquet to 300 mmHg for 70 minutes. On current examination, active flexion of the wrist and fingers is possible. Capillary refill is less than 3 sec in all digits.

These findings are most consistent with which of the following?

(A) Compartment syndrome
(B) Extensor mass ischemia
(C) Persistent brachial plexus block
(D) Post-tourniquet syndrome
(E) Radial nerve palsy

A

The correct response is Option E.

This patient has findings consistent with radial nerve palsy. Complications associated with pneumatic tourniquets have been reported in one of every 5000 to 8000 patients following tourniquet use. Post-tourniquet nerve palsies are more frequently associated with the use of rubber bandages. The most common complication following tourniquet use is nerve injury; the radial nerve is affected most frequently. Such nerve palsies result from pressure beneath the cuff rather than prolonged ischemia; most cases reportedly result from use of faulty pressure gauges. Permanent nerve deficits are rare; most patients have resolution of symptoms within six months. Tourniquet times of two hours at a pressure of 250 mmHg are commonly used in adults.

Although compartment syndrome has been reported as a complication of pneumatic tourniquet use, it typically occurs secondary to prolonged ischemia.

Muscle weakness resulting from intraoperative ischemia is unlikely in this patient because the pneumatic tourniquet was inflated for only 70 minutes.

In a patient who receives a brachial plexus block with 1% lidocaine, numbness should resolve within six hours.

Findings of post-tourniquet syndrome include edema, stiffness, pallor, weakness without paralysis, and subjective numbness without anesthesia. This condition results from prolonged ischemia, not direct tourniquet pressure, and may even be a mild form of compartment syndrome or ischemic reperfusion.

146
Q

A 37-year-old man has severe pain, swelling, and discoloration of the right hand one month after sustaining a crush injury to the dorsal aspect of the hand. A 4-cm laceration was repaired at the time of injury; there were no fractures or tendon injuries. On examination, the hand is swollen and stiff.

These findings are most consistent with which of the following conditions?

(A) Osteomyelitis
(B) Posttraumatic arthritis
(C) Reflex sympathetic dystrophy
(D) Secretan’s disorder
(E) Suppurative tenosynovitis

A

The correct response is Option C.

This patient’s findings are most consistent with reflex sympathetic dystrophy (RSD) or complex regional pain syndrome (CRPS), which manifests as a progressive or complex pain syndrome. Severe pain, swelling, stiffness, and
discoloration of the affected part are characteristic. These changes are thought to result from vasomotor instability of the sympathetic nervous system. Hyperhidrosis, osteoporosis, and trophic changes may also occur. The stages of reflex sympathetic dystrophy are acute, subacute, and chronic.

Osteomyelitis is a bone infection that develops secondary to an adjacent wound, joint, or tenosynovial infection. Osteomyelitis can also arise from blood-borne pathogens.

Patients with posttraumatic arthritis have joint pain, stiffness, and swelling that develop following intra-articular injury; however, the pain is typically less severe than in patients with RSD.

Secretan’s disorder or peritendinous fibrosis is often associated with minor work-related trauma. It is characterized by edema of the dorsal aspect of the hand and factitious lymphedema of the hand. This condition is typically not associated with the severe pain of RSD.

Suppurative tenosynovitis is characterized by fusiform swelling, tenderness along the flexor tendon sheath, and increased pain with passive extension or semiflexed positioning of the finger.

147
Q

A 35-year-old assembly-line worker has pain in the right upper extremity. On examination, there is pain and tenderness over the mobile wad; long finger extension and resisted supination tests are positive. These findings are most consistent with which of the following syndromes?

(A) Carpal tunnel syndrome
(B) Cubital tunnel syndrome
(C) Intersection syndrome
(D) Radial tunnel syndrome
(E) Wartenberg’s syndrome

A

The correct response is Option D.

This patient has symptoms consistent with radial tunnel syndrome, which is characterized by pain and tenderness over the mobile wad (radial tunnel) that occurs with extension, supination against resistance, or passive flexion and pronation of the wrist. Pain is localized just below the elbow in the extensor mass and along the course of the radial nerve. Radial tunnel syndrome has no sensory or motor loss. The compressing structures are the vascular leash, the arcade of Frohse, and the extensor carpi radialis brevis tendon.

Patients with carpal tunnel syndrome have pain at night, numbness, and tingling in the thumb, index, and long fingers and the radial side of the ring finger. Thenar weakness can develop. Associated findings include a positive Tinel’s sign over the carpal tunnel and positive findings on both the Phalen’s and reverse Phalen’s tests.

Cubital tunnel syndrome, or compression of the ulnar nerve at the elbow, is characterized by numbness and tingling in the ring and small fingers, pain over the medial aspect of the elbow, and sensory deficits along the ulnar side of the palm and in the ring and small fingers. Weakness develops in both the ulnar extrinsic and intrinsic motors. In patients with entrapment of the ulnar nerve at the wrist, numbness and tingling in the ring and small fingers are also presenting signs. These patients can also develop ulnar intrinsic weakness.

Intersection syndrome is a pain syndrome localized to the distal forearm at the intersection of the first extensor compartment (which contains the abductor pollicis longus and extensor pollicis brevis muscle bellies) and the second extensor compartment (which contains the extensor carpi radialis longus and brevis tendons). Patients with intersection syndrome have pain, swelling, and crepitus in the distal forearm proximal to the Lister tubercle. Symptoms are caused by tightness of the retinaculum over the first and second dorsal compartments associated with tenosynovitis of the tendons. Corticosteroid injections or tenosynovectomy are recommended for treatment.

In Wartenberg’s syndrome (ie, cheiralgia paresthetica), the radial nerve is compressed beneath the edge of the brachioradialis muscle at the level of the wrist. Affected patients have persistent pain on the dorsoradial surface of the hand and distal aspect of the forearm. Examination is most likely to show sensitivity to percussion over this area; Tinel’s sign is most likely to be positive over the course of the superficial radial sensory nerve along the dorsal edge of the brachioradialis muscle. The patient should first be instructed to avoid wearing tight jewelry or bracelets in the area. Operative exploration may be indicated in patients with persistent symptoms; however, decompression procedures are only associated with moderate success rates.

148
Q

Which of the following sensations is detected by the Pacinian corpuscles located within the skin and subcutaneous tissue?

(A) Burning pain
(B) Moving two-point discrimination
(C) Sharp pain
(D) Static two-point discrimination
(E) Vibration

A

The correct response is Option E.

The Pacinian corpuscles detect vibration to a sensitivity of 250 Hz. These rapidly adapting receptors are found in the subcutaneous tissue and can be identified easily on sight. They relay their signals through small myelinated A-beta fibers. However, each receptor covers a large area of skin and does not localize vibration sensation well.

Burning pain is detected by the free endings of C fibers, while sharp pain is detected by the free endings of A-delta fibers. Specific receptors are not involved.

Meissner’s corpuscles are located at the sides of the intermediate dermal ridge. These rapidly adapting receptors, which have a small area of skin sensitivity, fire at the beginning of a stimulus and occasionally at the end but never in between, and, therefore, are able to detect moving two-point discrimination.

Merkel cells detect static two-point discrimination. These receptors can be found around the sweat ducts on the underside of the intermediate dermal ridge. These slowly adapting receptors continue to fire for the duration of the stimulus. The sensory skin area connected to one Merkel cell is also well circumscribed, measuring from 2 to 4 mm. Meissner’s corpuscles and Merkel cells also relay signals via A-beta fibers.

149
Q

Which of the following is the most likely site of entrapment of the posterior interosseous nerve in the forearm?

(A) Arcade of Frohse
(B) Arcade of Struthers
(C) Band of Osborne
(D) Lacertus fibrosis
(E) Ligament of Struthers

A

The correct response is Option A.

Nerves of the upper extremity are prone to entrapment in areas of tight passage or at sharp inflection points. The site or level of compression defines the sensory and motor findings. The median nerve can become entrapped proximally in the arm by the ligament of Struthers, which is a dense band that can form between the supracondylar humeral process and the medial epicondyle. More distally, the median nerve can become entrapped beneath the lacertus fibrosis, which is a dense fascial sheet that extends proximally to the biceps tendon from the antebrachial fascia.

The ulnar nerve exits the arm in the medial intermuscular septum. Fascial folds in this area are known as the arcade of Struthers and can pinch the nerve, especially following anterior transposition. As the ulnar nerve passes through the cubital tunnel, a constricting band can form. This band, known as the band of Osborne, must be released during neuroplasty.

The radial nerve exits the arm after passing posterior to the humerus. It divides into the superficial and deep branches; the deep branch of the radial nerve, also known as the posterior interosseous nerve, can become entrapped, typically due to a fascial band over the supinator called the arcade of Frohse. The radial nerve here can also be compromised by vascular leashes, the edge of the extensor carpi radialis brevis, the edge of the proximal supinator, and the edge of the distal supinator. Proximal compression leads to weakness of extension of the wrist, finger, and thumb, often with radial deviation of the wrist because the extensor carpi radialis longus tendon is innervated proximally.

150
Q

A 45-year-old woman has pain and numbness in the right hand and forearm. There are no motor disturbances. Semmes-Weinstein monofilament testing shows decreased sensation in the index and long fingers, thumb, and palm. Phalen’s and Tinel’s signs are negative over the carpal tunnel.

These findings are most consistent with which of the following?

(A) Anterior interosseous syndrome
(B) C5 cervical radiculopathy
(C) Carpal tunnel syndrome
(D) Cubital tunnel syndrome
(E) Pronator syndrome

A

The correct response is Option E.

Although the sensory and motor deficits of carpal tunnel syndrome and pronator syndrome are similar, the sensory deficit within the palmar cutaneous region best differentiates pronator syndrome from carpal tunnel syndrome. This patient has findings consistent with pronator syndrome, or compression of the median nerve within the proximal
forearm. Sites of potential entrapment of the median nerve include the ligament of Struthers in the distal arm, the lacertus fibrosis at the level of the elbow, the pronator teres muscle, and the arch of the flexor digitorum superficialis muscle. Although symptoms are similar to carpal tunnel syndrome, sensory findings typically occur more proximally. Symptoms can be reproduced with active elbow flexion with the elbow in pronation, resisted elbow flexion, or resisted pronation with flexion of the wrist. Phalen’s sign is negative in most patients with pronator syndrome.

Anterior interosseous syndrome results from compression of the anterior interosseous branch of the median nerve in the forearm. Affected patients have poorly defined pain in the forearm and weakness of the profundus tendon of the index finger and the flexor pollicis longus tendon but no sensory deficit. Cervical radiculopathy at C5 is characterized by radicular-type pain in the lateral aspect of the upper arm and forearm. Cubital tunnel syndrome is caused by entrapment of the ulnar nerve in the region of the elbow. Affected patients have sensory deficits in the small finger and ulnar aspect of the ring finger.

151
Q

In a patient with late Volkmann’s contracture, which of the following is the most common finding?

(A) Fibrosis of the extensor muscles
(B) Fibrosis of the flexor muscles
(C) Intrinsic plus deformity
(D) Ischemic necrosis of the digits
(E) Radial nerve palsy

A

The correct response is Option B.

Volkmann’s contracture may develop as a result of compartment syndrome occurring secondary to arterial injury or direct muscle trauma. Children who sustain supracondylar fractures are most frequently affected because the brachial artery is compromised. Affected patients have myonecrosis that leads to muscle fibrosis. The fibrosed muscle results in markedly decreased muscle strength and excursion. Ischemic or compressive neuropathy may be seen in surrounding peripheral nerves.

The severity of Volkmann’s contracture correlates with the physiologic depth or location of the affected muscle and nerve. Patients with mild Volkmann’s contracture have involvement of the muscle bellies of the flexor digitorum profundus (FDP) with mild impairment.

Moderate Volkmann’s contracture occurs in most patients and is characterized by involvement of the FDP and flexor pollicis longus muscles; fibrosis of the flexor digitorum superficialis, flexor carpi ulnaris, and flexor carpi radialis muscles may also be seen. Affected patients have contractures of the extrinsic flexor muscles, resulting in claw hand. Neuropathy of the median nerve is most common. The ulnar nerve is less likely to be affected, and the radial nerve is rarely involved.

In patients with severe Volkmann’s contracture, fibrosis of the forearm flexor muscles is most common and tends
to be uniform. Involvement of the forearm extensor muscles varies.

Thus, the typical Volkmann’s contracture deformity is characterized by flexion of the wrist, flexion and adduction of the thumb, extension of the metacarpophalangeal (MP) joints, and flexion of the proximal interphalangeal (PIP) and distal interphalangeal joints.

Intrinsic plus deformities, which can result from contracture of the intrinsic muscles, are characterized by flexion of the MP joints and extension of the PIP joints.

Although untreated severe compartment syndrome can lead to ischemic necrosis of the digits, this finding is not typical of late Volkmann’s contracture.

152
Q

In a patient with undiagnosed compartment syndrome of the forearm, which of the following muscles are at greatest risk for ischemic injury?

(A) Extensor carpi ulnaris and flexor digitorum superficialis
(B) Flexor digitorum profundus and flexor carpi ulnaris
(C) Flexor digitorum superficialis and palmaris longus
(D) Flexor pollicis longus and flexor digitorum profundus
(E) Pronator teres and brachioradialis

A

The correct response is Option D.

An undiagnosed compartment syndrome poses the greatest risk to the flexor digitorum profundus and flexor pollicis longus muscles in the forearm. These muscles lie within the deepest compartments adjacent to bone and thus typically experience the greatest increases in interstitial pressure, leading first to ischemia and then to muscle necrosis.

In patients who develop severe contractures (involving all four fingers) resulting from undiagnosed compartment syndrome, the deep flexor muscles are most often involved as previously mentioned. The superficial flexors (flexor
and finally the superficial extensors (brachioradialis, extensor carpi radialis longus, extensor carpi radialis brevis, extensor digitorum communis, extensor carpi ulnaris). The characteristic deformity in these patients does not develop until weeks or months later and manifests as flexion of the elbow and wrist, pronation of the forearm, adduction and flexion of the thumb, and extension at the level of the metacarpophalangeal joint and flexion at the level of the interphalangeal joint with a claw-type deformity. These patients may also have a loss of sensation in the hand resulting from ischemic injury to the median and ulnar nerves.

153
Q

A 35-year-old woman has pain in the medial elbow and numbness and tingling of the ring and little fingers. Her symptoms are exacerbated by flexing the elbow with the forearm in supination. The most likely cause of this patient’s findings is nerve entrapment within which of the following structures?

(A) Arcade of Frohse
(B) Arcade of Struthers
(C) Lacertus fibrosis
(D) Leash of Henry
(E) Ligament of Struthers

A

The correct response is Option B.

This patient has cubital tunnel syndrome, or compression of the ulnar nerve at the level of the elbow. Characteristic findings include numbness in the dorsoulnar aspect of the hand, in the little finger, and in the ulnar aspect of the ring finger, as well as weakness of the ulnar extrinsic and intrinsic muscles. Although there are several potential sites of nerve entrapment, it is most likely to occur within the arcade of Struthers. This is a group of fascial bands from the medial intermuscular septum that can entrap the nerve, such as following anterior transposition. Another potential site of entrapment is the band of Osborne, which constricts the ulnar nerve as it passes through the cubital tunnel. Release of this band is critical during neuroplasty.

Entrapment of the deep branch of the radial nerve, or posterior interosseous nerve, can occur at the arcade of Frohse, which is a fascial band located along the supinator muscle in the forearm, or at the vascular leash of Henry, which is a sling of radial recurrent vessels that crosses the radial nerve.

The lacertus fibrosus, a dense sheet of aponeurotic fascia that extends from the biceps tendon to the flexor muscle mass, is a potential site of entrapment of the median nerve at the level of the elbow. This nerve can also become entrapped more proximally by the ligament of Struthers, which forms between the supracondylar humeral process and the medial epicondyle.

154
Q

A 34-year-old secretary has difficulty extending the middle and ring fingers of the right hand. On examination, there is weakness with extension of the wrist, fingers, and thumb. There is no sensory deficit. This patient’s findings are most consistent with

(A) C7 nerve root lesion
(B) lateral epicondylitis
(C) posterior interosseous nerve syndrome
(D) radial tunnel syndrome
(E) Wartenberg’s syndrome

A

The correct response is Option C.

This patient has findings consistent with posterior interosseous nerve syndrome, which is initially characterized by weakness and pain in the forearm in the absence of sensory loss. Other findings include weakness of extension of the metacarpophalangeal joints of the fingers and interphalangeal joint of the thumb, as well as weakness of thumb abduction and wrist extension. Because the innervation of the extensor carpi radialis longus tendon lies above the elbow and is thus not affected, the wrist often deviates radially.

A patient with a C7 nerve root lesion would have weakness in the radially innervated muscles (including the triceps), as well as weakness in the muscles with median nerve innervation, such as the pronator teres, flexor carpi radialis, flexor digitorum superficialis, and flexor pollicis longus.

Lateral epicondylitis is characterized by sharp pain at the epicondyle that is exacerbated with passive flexion of the wrist and fingers with the elbow in extension. Injection of a corticosteroid may produce relief. Although patients with lateral epicondylitis may have positive findings on middle finger testing, severe pain with passive stretch is more typical.

Radial tunnel syndrome involves compression of the radial nerve and results in chronic, aching pain in the area of the lateral humerus, elbow, extensor mass, and dorsal wrist. In addition, patients have tenderness over the mobile wad. Severe pain is elicited on middle finger testing. Weakness is not characteristic.

Wartenberg’s syndrome, or radial sensory nerve entrapment, is characterized by pain and/or paresthesias over the dorsoradial aspect of the hand and wrist. Tinel’s sign will be positive along the course of the nerve, and the patient will have paresthesias with the forearm in hyperpronation and the wrist in neutral. Because the motor branch of the radial nerve divides more proximally, weakness is not seen.

155
Q

A 50-year-old woman has paresthesias of the right thumb six hours after undergoing anatomic open reduction and rigid internal fixation of a fracture of the right distal radius. The pain, numbness, and weakness are worsening. Which of the following is the most appropriate next step in management?

(A) Application of ice and elevation of the extremity
(B) Semmes-Weinstein monofilament testing
(C) Open carpal tunnel release
(D) Release of Guyon’s canal
(E) Surgical exploration of the fracture site

A

The correct response is Option C.

In this patient who has acute carpal tunnel syndrome, the most appropriate management is open release of the carpal tunnel. Acute carpal tunnel syndrome can occur following injury, infection, or hemorrhage. Although appropriate reduction and stabilization of the fracture should typically be followed by rehabilitation of the soft tissues and wrist joint, open decompression of the median nerve in the carpal canal is the only logical next step in a patient who has developed acute carpal tunnel syndrome. Limited exposure techniques are contraindicated in patients with fractures.

Although application of ice and elevation of the extremity are part of normal fracture management, this course would be inadequate in a patient who has acute compression of the median nerve. Semmes-Weinstein monofilament testing will provide objective evidence of median nerve pathology but will not relieve the pain, numbness, and weakness. Release of Guyon’s canal is appropriate for those patients who require decompression of the ulnar nerve at the wrist. Surgical exploration is recommended for definitive management of fractures not associated with carpal tunnel syndrome or other complications, and may not even be necessary in patients with simple fractures.

156
Q

A 24-year-old man has a 2.5-cm gap in the distal digital nerve of the dominant right index finger after cutting the finger while using a saw. Which of the following donor nerves is most appropriate for autografting?

(A) Dorsal branch of the ulnar nerve
(B) Medial antebrachial cutaneous nerve
(C) Superficial radial nerve
(D) Sural nerve
(E) Terminal branch of the posterior interosseus nerve

A

The correct response is Option E.

Selection of an appropriate nerve graft depends on the length, diameter, and function required from the graft. In addition, sensory loss at the donor site should not present a functional problem.

In this patient, the terminal branch of the posterior interosseous nerve will best match the required specifications for replacement of the severed digital nerve. The posterior interosseous nerve can be found deep to the extensor tendons at the level of the wrist. It lies in the floor of the fourth extensor compartment on the radial side, ulnar and deep to the extensor pollicis longus tendon and muscle. The terminal branch is frequently harvested for digital nerve defects because one fascicular strand can be transferred to replace a single digital fascicle. Because it is an articular branch of the nerve, there is no associated sensory deficit.

All of the other sources can be used for grafting but are inferior to the terminal branch of the posterior interosseous nerve for the replacement of a digital nerve. Although the dorsal branch of the ulnar nerve can provide approximately 15 cm of nerve for grafting, harvest of this branch is associated with numbness on the dorsoulnar aspect of the hand. Harvest of the lateral and medial antebrachial cutaneous nerves can be associated with significant donor site morbidity. This is a less favored site for grafting due to the large amount of interfascicular tissue surrounding the nerves. The superficial radial nerve provides an excellent source for graft material, with minimal epineural tissue and tightly packed fascicles, and is best used for nerve reconstruction in a patient with a pre-existing lesion of the high
and posterior to the lateral malleolus in the ankle and can provide as much as 40 cm of nerve for grafting.

157
Q

A 42-year-old man has the acute onset of ischemia in the dominant right upper extremity after sustaining a myocardial infarction. The patient undergoes embolectomy followed by infusion of heparin; 24 hours later, he has pain, tenseness, and tingling of the affected extremity. On examination, he has severe pain with passive range of motion of the elbow, forearm, wrist, and hand. Pulses are weak.

Which of the following is the most appropriate next step in management?

(A) Application of a wrist extensor fixator
(B) Incision and drainage
(C) Median nerve decompression
(D) Fasciotomy
(E) Brachial palmar arch bypass

A

The correct response is Option D.

This patient has developed acute ischemia in the upper extremity, a finding most likely caused by the development of cardiac mural thrombi. Cardiac mural thrombi can result from myocardial infarction or atrial fibrillation and can subsequently embolize. Emboli may also develop from the superficial palmar arch or subclavian artery. Embolectomy and anticoagulant therapy are usually recommended. However, because he has had ischemia for more than 24 hours prior to reperfusion, his condition has most likely progressed to compartment syndrome; the findings of pain, tenseness, and tingling are consistent with this diagnosis. Delayed reperfusion can lead to the onset of compartment syndrome. Therefore, the most appropriate next step is to perform fasciotomy, or compartment release, to relieve the progressively worsening muscle necrosis.

158
Q

A 49-year-old man is evaluated 4 months after sustaining an injury from a motorcycle collision. Physical examination shows a C5-C6 brachial plexus palsy. There has been no recovery of function since the time of injury, and there is a stationary Tinel sign. Electrodiagnostic testing shows absence of motor unit action potentials. Nerve transfer is planned. Which of the following is the most appropriate nerve transfer in this patient?

A) Anterior interosseous to ulnar
B) Intercostal to medial pectoral
C)Musculocutaneous to axillary
D)Musculocutaneous to ulnar
E) Spinal accessory to suprascapular

A

The correct response is Option E.

In a patient with C5-C6 brachial plexus palsy, there will be limitation in shoulder function and elbow flexion. These functions can be restored with nerve transfer if the patient is treated before too much time has elapsed. Typically, the goal for reinnervation of muscle is within 12 to 18 months, and if there has not been any functional recovery or development of motor unit action potentials by 3 to 6 months, surgery is indicated. If the time frame is too long, then nerve surgery cannot be performed and tendon transfers will be the procedure of choice. Spinal accessory to suprascapular nerve transfer restores shoulder abduction and external rotation via rotator cuff function. This is often combined with deltoid reinnervation via triceps branch to axillary nerve transfer to gain additional strength. Elbow flexion can be achieved with the double fascicular nerve transfer, which involves fascicles from the ulnar nerve to the biceps branch and fascicles from the median nerve to the brachialis branch. Phrenic to musculocutaneous nerve transfer may be used to regain elbow flexion when other nerves are not available (such as ulnar, median, or intercostal) as in a total plexus injury (C5-T1). Anterior interosseous to ulnar motor branch nerve transfer is performed to provide intrinsic function of the hand and is used for high ulnar nerve injuries.

Intercostal to medial pectoral nerve transfer would not restore function in this case. Both intercostal nerves and the medial pectoral nerve can be used as donor nerves, as the medial pectoral nerve arises from the lower trunk and is intact with upper trunk or C5-C7 plexus injuries.

Musculocutaneous to axillary nerve transfer would not be appropriate, as the biceps is innervated by the musculocutaneous nerve (C5-C6) and would not be functioning in this case.

Musculocutaneous to ulnar nerve transfer would not be appropriate either. The triceps branch to axillary nerve transfer may be employed to restore deltoid function in upper plexus injuries.

159
Q

A 65-year-old man with bilateral carpal tunnel syndrome is scheduled to undergo release of the more symptomatic right hand. Before the procedure, the patient reports the recent onset of mild heart failure. During the procedure, synovial biopsy is performed to evaluate for amyloid disease. Which of the following staining procedures is most appropriate for this patient’s biopsy specimen?

A) Antinuclear antibody
B) Congo red
C) Giemsa
D) Periodic acid-Schiff
E) Van Gieson

A

The correct response is Option B.

The finding of bilateral carpal tunnel syndrome in older patients should make one think about the diagnosis of amyloidosis. One recent study showed that 10% of men over age 50 years and women over age 60 years who undergo carpal tunnel release with bilateral symptoms had a positive tenosynovial biopsy for amyloid (Donnelly et al). Other associated findings in patients with amyloidosis included trigger finger, biceps tendon rupture, spinal stenosis, heart arrhythmias, or heart failure.

Congo red stain makes amyloid protein a salmon pink color, and when placed under polarized light, the amyloid proteins have an apple green birefringence. Antinuclear antibody (ANA) stain shows a speckled pattern throughout the nucleus of cells in various diseases like systemic lupus erythematosus and Sjgren syndrome. Periodic acid-Schiff stains detect polysaccharides such as glycogen in glycogen storage disease of the liver. Van Gieson stain is used to differentiate collagen and connective tissue. Giemsa stain detects spirochetes and blood parasites.

Following the discovery of Congo-red-positive amyloid deposits on initial histology review, specimens are typically sent for more advanced testing to determine the type of amyloid present, since not all types of amyloid cause the same spectrum of disease. Depending on the type of amyloid protein present, further specialty consultation with cardiology, hematology, nephrology, or other specialists might be indicated.

160
Q

A 45-year-old man comes to the office because of a 3-month history of aching pain of the proximal forearm, weakened grip, and a burning sensation near the thumb. Radial tunnel syndrome is suspected. During physical examination, at which of the following locations is the point of maximal tenderness most likely to be found?

A) 4 cm distal to the lateral epicondyle
B) 4 cm proximal to the lateral epicondyle
C) At the lateral epicondyle
D) At the medial epicondyle
E) At the radial styloid

A

The correct response is Option A.

Radial tunnel syndrome is an uncommon nerve compression characterized by lateral forearm pain without motor symptoms.

Tenderness is localized over the route of the posterior interosseous nerve, 4 to 5 cm distal to the lateral epicondyle at the supinator muscle. Other described examination findings include pain with resisted extension of the middle finger and pain with resisted supination of the extended elbow. Electrodiagnostic studies are usually not helpful in making the diagnosis. MRI has been shown to be of benefit, however, and can help diagnose other syndromes which may produce similar complaints, including tendinopathy at the elbow.

161
Q

A 71-year-old man presents with progressive numbness of the thumb and index finger, along with weakness of the thumb. Electromyography and nerve conduction studies show severe median nerve compression at the wrist. Which of the following changes of the median nerve are most likely?

Cross-sectional
area of median
nerve proximal
to the carpal tunnel Myelination
A) Decreased               decreased
B) Decreased                increased
C) Increased               decreased
D) Increased               increased
E) No change              no change

A

The correct response is Option C.

This patient presents with median nerve compression at the wrist consistent with a diagnosis of carpal tunnel syndrome. With progressive nerve compression, there is frequently edema of the nerve, evidenced by increased cross-sectional area, and this radiographic parameter is increasingly used for the diagnosis of carpal tunnel syndrome. Demyelination can occur with severe nerve compression and can ultimately be accompanied by loss of axonal integrity in advanced cases.

162
Q

A 46-year-old woman has a 5-month history of burning pain and intermittent numbness and tingling of the tips of the thumb, index finger, and middle finger. The symptoms occasionally wake her at night, causing her to shake her hands to make them stop. Physical examination shows no evidence of thenar wasting; thumb abduction and thenar strength are normal. Electrodiagnostic testing is most likely to show which of the following findings in this patient?

Median sensory     Median motor    Fibrillations
latency        latency
A) Increased  /    increased  /    absent
B) Increased  /    increased  /    present
C) Normal  /     normal  /     absent
D) Normal  /     increased  /    present
E) Normal  /     normal  /     present

A

The correct response is Option A.

This patient demonstrates signs and symptoms consistent with carpal tunnel syndrome. Compression of the median nerve results in numbness and paresthesias of the median nerve distribution, which includes the thumb, index finger, middle finger, and radial half of the ring finger. Her symptoms are only intermittent, and the absence of physical findings of thenar atrophy would suggest that there is no permanent denervation or muscle changes that have occurred.

On nerve conduction studies, latency reflects the measurement of the time necessary for a signal to travel across a segment of nerve from a stimulating electrode to a recording electrode. In the presence of nerve compression, demyelination will result in slowing of nerve conduction, resulting in a longer time and increased latency (measured in milliseconds). Median sensory and motor latency measurements can be increased in carpal tunnel syndrome. In some mild cases, the latencies may be normal (unchanged) but would not be paired with fibrillations. Decreased latency means that nerve conduction is faster than usual and would not be the case in nerve compression.

On electromyography, the needle examination will assess the presence of denervation changes, manifested as fibrillations and positive sharp waves. These findings indicate signs of ongoing nerve injury and muscle denervation, and they are present in more advanced disease. Spontaneous muscle activity with fibrillation potentials is the earliest sign of denervation, which occurs after the changes in conduction velocity. In severe carpal tunnel syndrome, examination of the median-innervated abductor pollicis brevis muscle will disclose electromyography changes. There may also be alteration in the motor unit potentials due to chronic episodes of denervation and reinnervation.

The diagnosis of carpal tunnel syndrome remains a clinical diagnosis based on history and physical examination. Clinical suspicion may be confirmed by electrophysiologic testing, but clinical practice guidelines do not recommend routine testing since there may be instances of false-negative and false-positive results.